[obm-l] Re: [obm-l] Re:[obm-l] dúvida??

2002-09-30 Thread leonardo mattos

  Ola,
  Quanto a letra a).Pq nao seria 12*(2/3)^3?!Considerando que no 1ºpulo ela 
alcança 12*2/3,2ºpulo 12*(2/3)^2 e no 3º12*(2/3)^3...
   Um abraço,Leonardo


>From: "Wagner" <[EMAIL PROTECTED]>
>Reply-To: [EMAIL PROTECTED]
>To: <[EMAIL PROTECTED]>
>Subject: [obm-l] Re:[obm-l] dúvida?? Date: Mon, 30 Sep 2002 20:31:27 -0300
>
>Oi pessoal !
>
>
>Mário wrote:
>
>Amigos de lista, peço ajuda:
>
>
>"Uma bola pula cada vez que bate no chão 2/3 da altura de onde caiu.
>Deixando-a cair da altura de 12 metros, pergunta-se:
>a) qual será a altura do terceiro pulo?
>b) Quanto percorreu ao bater no chão pela terceira vez?
>
>
>  As alturas máximas a cada pulo estão em progressão geométrica de razão 
>2/3 e termo inicial 12, assim como a distância percorrida a cada pulo 
>também forma uma PG de razão 2/3 e termo inicial 24. Logo:
>
>a) x = 12.(2/3)^2 = 16/3 metros
>b) Considerando que a pergunta seja quanto ela percorreu desde que é 
>abandonada da altura inicial :
>y = 24.((2/3)^3 - 1)/(2/3 -1) = 24.(-19/27)/(-1/3) = 24.3.19/27 = 152/3 
>metros




_
Converse com seus amigos online, faça o download grátis do MSN Messenger: 
http://messenger.msn.com.br

=
Instruções para entrar na lista, sair da lista e usar a lista em
http://www.mat.puc-rio.br/~nicolau/olimp/obm-l.html
O administrador desta lista é <[EMAIL PROTECTED]>
=



[obm-l] Re: [obm-l] Re: [obm-l] dúvida

2002-09-30 Thread Henrique Branco

Pergunta besta... Mas se 610 = (1001100010)_2, por que ele acertou as
perguntas 2, 6, 7, 10?
Desculpem pelo "nivel primario" da pergunta... Mas...
Grato,
Henrique.

- Original Message -
From: Rodrigo Villard Milet
To: [EMAIL PROTECTED]
Sent: Saturday, September 28, 2002 9:00 PM
Subject: [obm-l] Re: [obm-l] dúvida

Escreva 610 na base 2 : 610 = (1001100010)_2. Como sabemos que a
representação na base 2 é única, ele acertou as perguntas 2,6,7 e 10.
Villard


-Mensagem original-
De: Mário Pereira <[EMAIL PROTECTED]>
Para: [EMAIL PROTECTED] <[EMAIL PROTECTED]>
Data: Sábado, 28 de Setembro de 2002 11:22
Assunto: [obm-l] dúvida

Olá, se alguém puder, me dê uma dica:

Em um jogo de televisão, um candidato deve responder a 10 perguntas. A
primeira vale 1 ponto, a segunda vale 2 pontos, e assim, sucessivamente,
dobrando sempre. O candidato responde a todas as perguntas e ganha os pontos
correspondentes às respostas que acertou, mesmo que erre algumas. Se o
candidato obteve 610 pontos, quantas perguntas acertou?

Obrigado,

Mário.


=
Instruções para entrar na lista, sair da lista e usar a lista em
http://www.mat.puc-rio.br/~nicolau/olimp/obm-l.html
O administrador desta lista é <[EMAIL PROTECTED]>
=



[obm-l] RE: [obm-l] Re: [obm-l] DÚVIDA

2002-10-22 Thread leandro









Andre,

 

O Augusto usou um
artificio muito utilizado em calculo para voce derivar funcoes desse tipo. A
notacao exp(x) siginifica o mesmo que e^x. Na verdade, ele escreveu y =
e^(ln(y)) e usou a regra da cadeia pra obter a derivada desejada. 

 

Leandro.

 

-Original Message-
From: [EMAIL PROTECTED]
[mailto:[EMAIL PROTECTED]] On
Behalf Of Wagner
Sent: Tuesday, October 22, 2002
10:59 AM
To: [EMAIL PROTECTED]
Subject: [obm-l] Re: [obm-l]
DÚVIDA

 



Oi para todos!





 





Não entendi direito a notação (a exp
principalmente) e os parênteses, chaves e colchetes ficaram confusos.





O que você quis dizer por acaso é : 





f '(x) = {(b^x +
c^x)^(1/x)}.{[(-1/x^2).ln (b^x + c^x)] + ((1/x).[(ln b).(b^x) + (ln c).(c^x)])
/ ( b^x + c^x) }   ?





Por favor esclareça





 





André T.





 





 







- Original Message - 





From: Augusto
César Morgado 





To: [EMAIL PROTECTED]






Sent: Sunday,
October 20, 2002 10:41 PM





Subject: Re: [obm-l]
DÚVIDA





 





Wagner wrote:
f(x) = (b^x + c^x)^(1/x) = exp [(1/x) ln (b^x + c^x)]
f'(x) = exp [(1/x) ln (b^x + c^x)] * 
{(- 1/x^2) ln (b^x + c^x) + (1/x)  [(b^x * lnb + c^x * ln c)/ (b^x + c^x)]






 Oi para todos!





 





    Estava resolvendo
um problema e me deparei com isso:





 





Se f(x) = (b^x + c^x)^(1/x), quanto
vale f '(x) em função de b e c?





 





A quem conseguir me ajudar agradeço





 





André T.



 










[obm-l] Re: [obm-l] Re: [obm-l] Dúvida???

2003-01-19 Thread ghaeser
Seja S={1,2,..,2n}

em qualquer subconjunto T de S, tq |T|=n+1, existem x e y tq 
mdc(x,y)=1

dem: indução em n.

base:
n=1
S={1,2}
T={1,2}

x=1, y=2

hip: existe x e y , se S={1,2,..,2n} e |T|=n+1, T subconj. de S.

passo:

seja S={1,2,..,2n,2n+1,2n+2}

se 2n+1 e 2n+2 nao pertencem a T, |T|=n+2 então basta remover um elemento
qualquer de T e aplicar a hipótese de indução.

se 2n+1 xou 2n+2 pertencem a T, |T|=n+2 então basta remover este elemento
e aplicar a hipótese de indução

se 2n+1 e 2n+2 pertencem a T então x=2n+1, y=2n+2.

>>   Por que podemos garantir que em qualquer subconjunto com n + 1
>> elementos do conjunto {1, 2, 3, ..., 2n} existem pelo menos
>> dois elementos que são primos entre si?
>>
>
>

"Mathematicus nascitur, non fit"
Matemáticos não são feitos, eles nascem
---
Gabriel Haeser
www.gabas.cjb.net


--
Use o melhor sistema de busca da Internet
Radar UOL - http://www.radaruol.com.br



=
Instruções para entrar na lista, sair da lista e usar a lista em
http://www.mat.puc-rio.br/~nicolau/olimp/obm-l.html
O administrador desta lista é <[EMAIL PROTECTED]>
=



[obm-l] Re: [obm-l] Re: [obm-l] Dúvida???

2003-01-27 Thread Wagner



Oi para todos !
 
Desculpe o descuido, faltou dizer que x deve ser 
primo. Para n > 1 oconjunto tem pelo menos um número primo.
Mas me ocorreu uma dúvida, a afirmação vale para 
n=1?
 
André T.
 
 

  - Original Message - 
  From: 
  A. C. 
  Morgado 
  To: [EMAIL PROTECTED] 
  Sent: Sunday, January 19, 2003 1:04 
  PM
  Subject: Re: [obm-l] Re: [obm-l] 
  Dúvida???
  Por exemplo, se n=5, o conjunto eh {1, 2, ...,10}. Considere o 
  subconjunto {1, 2, 3, 4, 5, 6}. Se x=4, um elemento do subconjunto que não eh 
  multiplo de x eh o 6. Logo, de acordo com a prova dada 4 e 6 sao primos entre 
  si!MorgadoWagner wrote:
  



Oi para todos !
 
Sim. Aqui vai a prova :
Suponha que m elementos do subconjunto sejam 
múltiplos de x.
Para x > 1, temos m < n + 1 . Logo existe 
pelo menos um elemento do subconjunto que não
é múltiplo de x . Seja y esse elemento . 
Como x =< 2n , então pelo menos x e y
são primos entre si.
 
André T.
 
 

  - 
  Original Message - 
  From: 
  Danilo Artigas 
  To: obm-l 
  
  Sent: 
  Sunday, January 19, 2003 2:24 AM
  Subject: 
  [obm-l] Dúvida???
  
    Por que podemos garantir que em 
  qualquer subconjunto com n + 1 elementos do conjunto {1, 2, 3, ..., 
  2n} existem pelo menos dois elementos que são primos entre 
  si?


[obm-l] Re:[obm-l] Re: [obm-l] DÚVIDA

2003-08-17 Thread luis-cu
me descupe, mas axu q seu jeito esta errado, pq veja so:

se x=-3, temos -3²+|-3|-6=> diferente de zero

e, do mesmo jeito q pode ser 2, pode ser -2, pode caucular

-2²+|-2|-6=2²+2-6=0

logo resposta letra c, 2.(-2)=-4

pelo menos é oq penso
grato

ZANFORLIN

> Um método simples é reescrever as equações com os valore
s do módulo:
> 
> Para x>0, X^2 + X - 6 = 0  -->x'= -
3   e   x" = 2
> Para x<0  X^2 +|-X| - 6 = 0   -->x' = -
3   e   x" = 2
> 
> Observe que o módulo não deixa o segundo termo ser negat
ivo
> 
> Ex: |2| = |-2| = 2
> 
> RESPOSTA: B
> 
> - Original Message -
> From: <[EMAIL PROTECTED]>
> To: <[EMAIL PROTECTED]>
> Sent: Saturday, August 16, 2003 10:31 PM
> Subject: [obm-l] DÚVIDA
> 
> 
> > A equação  |X|²+|X|-6 =0
> > a) só tem uma solução.
> > b) tem duas soluções, tais que seu produto é = - 6.
> > c) tem duas soluções, tais que seu produto é = - 4.
> > d) tem duas soluções, tais que seu produto é igual a 0
> >
> > __
___
> > Voce quer um iGMail protegido contra vírus e spams?
> > Clique aqui: http://www.igmailseguro.ig.com.br
> > Ofertas imperdíveis! Link: http://www.americanas.com.b
r/ig/
> >
> > ==
===
> > Instruções para entrar na lista, sair da lista e usar 
a lista em
> > http://www.mat.puc-rio.br/~nicolau/olimp/obm-l.html
> > ==
===
> 
> 
=
> Instruções para entrar na lista, sair da lista e usar a 
lista em
> http://www.mat.puc-rio.br/~nicolau/olimp/obm-l.html
> 
=
> 

 
__
Acabe com aquelas janelinhas que pulam na sua tela.
AntiPop-up UOL - É grátis!
http://antipopup.uol.com.br/


=
Instruções para entrar na lista, sair da lista e usar a lista em
http://www.mat.puc-rio.br/~nicolau/olimp/obm-l.html
=


[obm-l] Re: [obm-l] Re: [obm-l] DÚVIDA

2003-08-18 Thread Bernardo Vieira Emerick
Oi Henrique,

Eu solucionei o problema da mesma forma que você, exceto por um ponto. 
Quando você põe que x>0, a raiz negativa deve ser abandonada, assim como 
quando se coloca a condição de x<0, a raiz positiva deve ser omitida. As 
raízes que sobram são 2 (para x>0) e -2 (para x<0).
Abraços,
Bernardo


From: "Henrique Patrício Sant'Anna Branco" <[EMAIL PROTECTED]>
Reply-To: [EMAIL PROTECTED]
To: <[EMAIL PROTECTED]>
Subject: [obm-l] Re: [obm-l] DÚVIDA
Date: Sun, 17 Aug 2003 15:23:30 -0300
> A equação  |X|²+|X|-6 =0
> a) só tem uma solução.
> b) tem duas soluções, tais que seu produto é = - 6.
> c) tem duas soluções, tais que seu produto é = - 4.
> d) tem duas soluções, tais que seu produto é igual a 0
Não sei se esse é o jeito certo de resolver, mas...
|x|^2 = (sqrt(x^2))^2 = x^2
Então x^2 + |x| - 6 = 0
Agora temos x > 0 e x < 0
(1) x > 0 ==> x^2 + x - 6 = 0 ==> x = 2 e x = -3
(2) x < 0 ==> x^2 - x - 6 = 0 ==> x = -2 e x = 3
Como tem o módulo, |-x| = x e, portanto, só precisamos testar 2 e 3 ou -2
e -3.
2^2 + 2 - 6 = 0 ==> 0 = 0
3^2 + 3 - 6 = 0 ==> 6 = 0
Portanto, as raízes são -2 e 2.
Letra c)
Abraços,
Henrique.
=
Instruções para entrar na lista, sair da lista e usar a lista em
http://www.mat.puc-rio.br/~nicolau/olimp/obm-l.html
=
_
MSN Hotmail, o maior webmail do Brasil.  http://www.hotmail.com
=
Instruções para entrar na lista, sair da lista e usar a lista em
http://www.mat.puc-rio.br/~nicolau/olimp/obm-l.html
=


[obm-l] Re:[obm-l] RE: [obm-l] Dúvida

2004-06-24 Thread claudio.buffara

Oi, Paulo:
 
Acho que esta sua demonstracao do teorema de Cauchy soh eh valida se G for abeliano, pois no fim, quando voce fala na projecao canonica p: G -> G/H, voce estah implicitamente supondo que G/H eh um grupo e, portanto, que H eh um subgrupo normal de G. Mas isso soh eh verdade para todo H se G for abeliano.
 
Por outro lado, existe uma demonstracao desse teorema que eh um dos meus exemplos favoritos de beleza matematica:
 
Seja G um grupo e p um primo que divide |G|.
 
Considere todos os produtos da forma x_1*x_2*...*x_p que sao iguais a "e", onde os x_i sao elementos nao necessariamente distintos de G.
 
Eh facil ver que existem |G|^(p-1) tais produtos pois, escolhendo-se livremente os valores de x_1, x_2, ..., x_(p-1), o valor de x_p fica unicamente determinado (igual ao inverso de x_1*x_2*...*x_(p-1))
 
Agora vamos dividir estes |G|^(p-1) produtos em classes de equivalencia de forma que dois produtos pertencem a uma mesma classe se e somente se um deles for uma permutacao circular do outro. Teremos dois casos a considerar:
 
Caso 1: todos os x_i sao iguais.
Nesse caso, a classe vai conter apenas um produto, pois x_1*x_2*...*x_p = a*a*...*a e existe apenas uma permutacao dos x_i.
 
Caso 2: pelo menos dois dos x_i sao distintos.
Nesse caso, a classe vai conter exatamente p produtos:
x_1*x_2*...*x_(p-1)*x_p; 
x_2*x_3*... x_p*x_1;
x_3*x_4*...*x_1*x_2;
...
x_p*x_1*...x_(p-1)*x_(p-1).
 
Sejam N1 e N2 os numeros de classes de equivalencia de cada tipo.
Entao, teremos que:
numero de produtos =  1*N1 + p*N2 = |G|^(p-1).
 
Por hipotese, p | |G|^(p-1) e obviamente p | p*N2. 
Logo, p | N1.
 
Alem disso, o produto e*e*...*e obviamente eh do tipo 1, de modo que N1 > 0.
 
Ou seja, o numero N1 de produtos da forma a*a*...*a = a^p = e eh um multiplo positivo de p.
Em outras palavras, existem pelo menos p-1 elementos em G de ordem p.
Naturalmente, se a eh um tal elemento, entao , o subgrupo ciclico gerado por a, terah ordem p.
 
[]s,
Claudio.
 




De:
[EMAIL PROTECTED]




Para:
[EMAIL PROTECTED]




Cópia:





Data:
Thu, 24 Jun 2004 14:20:38 +




Assunto:
[obm-l] RE: [obm-l] Dúvida




 
 
> Ola Eder,
> 
> Ok !
> 
> Vamos fazer o seguinte. Vou provar um resultado classico que voce podera 
> usar na solucao.
> 
> TEOREMA DE CAUCHY : Se G e um grupo finito e "p" e um numero primo que 
> divide
> a ordem de G entao existe um elemento "g" de G de ordem "p".
> 
> PROVA : Vamos usar inducao sobre a ordem de G. Mais especificamente vamos 
> mostrar que
> ( HIPOTESE DE INDUCAO ) se todos os grupos com ordem menor que G satisfazem 
> o TEOREMA DE CAUCHY entao G satisfaz o TEOREMA DE CAUCHY.
> 
> 1) Se a ordem de G for um numero primo, |G| = p, entao a prova e trivial e 
> nem precisamos usar a hipotese de inducao, pois "p" sera o unico numero 
> primo que pode dividir a ordem de G e se "g"
> for um elemento de G entao, pelo teorema de Lagrange, divide |G|, isto 
> e, a ordem de
> "g" e "p". Assim, nao so um, mas todos os elementos de G ( com excecao da 
> identidade ) tem
> ordem "p"
> 
> 2) Se ordem de G nao for um numero primo, seja "p" um numero primo que 
> divide a ordem de G.
> Tomando um elemento "g" pertencente a G, "g" diferente de "e", considere o 
> subgrupo de G : H=. Existem duas possibilidades para H :
> 
> PRIMEIRA : H e igual a G. Neste caso, G e ciclico com G=. Seja N=|G| e 
> considere o elemento g^(N/p). Claramente que g^(N/p) pertence a G e ordem de 
> g^(N/p) e "p". Assim,
> G tem um elemento de ordem "p" e acabou.
> 
> SEGUNDA : H e diferente de G. Neste caso |H| < |G|.
> 
> Se "p" divide |H|, pela HIPOTESE DE INDUCAO, existe "h" pertencente a H tal 
> que ordem de "h" e "p". Como H e subconjunto de G segue que "h" e tambem 
> elmento de G e, portanto, G tem um elemento de ordem "p" e acabou.
> 
> Se "p" nao divide |H| ( mas "p" divide |G|, por hipotese ), pelo teorema de 
> Lagrange |G|=|H|(G:H) teremos que "p" divide (G:H), isto e, "p" divide o 
> indice de H em G. Como (G:H) =| G/H | e
> G/H| < |G|, pela HIPOTESE DE INDUCAO, existe um h_ ( h barra ) em G/H de 
> ordem "p".
> 
> Considere a projecao canonica :
> 
> p : G -> G/H
> 
> Sabemos que trata-se de um homomorfismo e que em todo homomorfismo a ORDEM 
> DA IMAGEM DE UM ELEMENTO DIVIDE A ORDEM DO ELEMENTO, isto e, |h_| divide |h| 
> para algum "h" em G. Como |h_| = p => |h| = kp, para algum k inteiro. 
> Considere o elemento h^k. Claramente que h^k pertence a G e | h^k | = p. 
> Assim, G tem um elemento de ordem "p".
> 
> Vemo que a hipotese de inducao vale ( por vacuidade ) para as ordem 1 e 
> tambem para a
> ordem 2. Segue - pelo que vimos acima - que vale para todas as ordens.
> 
> Um Abraco
> Paulo Santa Rita
> 5,1117,240604
> 
> >From: Lista OBM <[EMAIL PROTECTED]>
> >Reply-To: [EMAIL PROTECTED]
> >To: [EMAIL PROTECTED]
> >Subject: Re: [obm-l] RE: [obm-l] Dúvida
> >Date: Thu, 24 Jun 2004 10:08:22 -0300 (ART)
> >
> >Meu caro Paulo, entendi sua solução, o prblema que esse exercício 
> >encontra-se na seção de um >livro onde ainda

[obm-l] Re:[obm-l] RE: [obm-l] Dúvida

2004-06-24 Thread claudio.buffara

 




De:
[EMAIL PROTECTED]




Para:
[EMAIL PROTECTED]




Cópia:





Data:
Thu, 24 Jun 2004 14:38:52 -0300




Assunto:
[obm-l] RE: [obm-l] Dúvida




 
 
> Você também está usando o fato do grupo ser abeliano, não?
> 
> "Caso 2: pelo menos dois dos x_i sao distintos.
> Nesse caso, a classe vai conter exatamente p produtos:"
> 
> em especial está usando este fato:
> (x_1 * ... x_{p-1}) * x_p = x_p * (x_1 * ... * x_{p-1})
> 
Não. Não estou.
 
Lembre-se de que eu estou apenas considerando produtos de p elementos (não necessariamente distintos) de G cujo produto é a identidade de G.
 
No seu exemplo acima, o que acontece eh que x_p = (x_1*...*x_(p-1))^(-1) de modo que x_p e (x_1*...*x_(p-1)) comutam, uma vez que um elemento e seu inverso sempre comutam, em qualquer grupo, abeliano ou não.
 
[]s,
Claudio.
 

[obm-l] Re: [obm-l] Re:[obm-l] Dúvida

2004-06-25 Thread claudio.buffara

 
> Meu caro Cláudio,
>  
> fiquei me perguntando sobre a seguinte afirmação:
>  
> "Mas A pode ser particionado em pares nao ordenados da forma:
> {x,x^(-1)}"
>  
> O que garante que cada x pertencente a A tem seu inverso em A?
>
 
A eh o conjunto dos elementos de G que sao diferentes dos respectivos inversos.
Assim: 
x pertence a A <==> 
x <> x^(-1) <==> 
x^(-1) <> (x^(-1))^(-1) <==> 
x^(-1) pertence a A.   
 
***
 
No mais, aqui vai uma pequena correcao: a definicao precisa do conjunto B abaixo eh:
B = {x em G | x = x^(-1)  e  x <> e}
 
[]s,
Claudio.
 
"claudio.buffara" <[EMAIL PROTECTED]> wrote:

>
> Oi, Eder:
>  
> O Paulo Santa Rita usou uma bazuca pra matar uma barata.
>  
> Uma solucao mais simples seria a seguinte:
>  
> Particione G nos tres subconjuntos a seguir:
> {e},
> A = {x em G | x <> x^(-1)},
> B = {x em G | x = x^(-1)}.
>  
> Como G tem 2n elementos, A uniao B terah 2n - 1 elementos.
>  
> Mas A pode ser particionado em pares nao ordenados da forma:
> {x,x^(-1)}, jah que cada um dos elementos de A eh distinto do seu inverso.
> Isso significa que A tem um numero par de elementos, digamos 2m.
>  
> Logo, B terah 2n - 1 - 2m elementos, um numero impar e, portanto, >= 1.
>  
> Ou seja, deve existir algum x em G tal que x = x^(-1) <==> x^2 = e.
>  
> []s,
> Claudio.
>  
>



De:
[EMAIL PROTECTED]
>



Para:
[EMAIL PROTECTED]
>



Cópia:

>



Data:
Thu, 24 Jun 2004 07:02:59 -0300 (ART)
>



Assunto:
[obm-l] Dúvida
>



 
 
> > Gostaria que alguém me ajudasse com o problema abaixo:
> >  
> > Seja (G, . ) um grupo contento exatamente 2n elementos, n >=1. Prove que existe x <> e t.q. x^2 = x.x = e.
> >  
> > Obs.: (i) x <> e denota x diferente da unidade de (G, . );
> >     (ii) . é uma operação qualquer que torna G um grupo.
> >  
> > Grato, Éder.


Yahoo! Mail - Participe da pesquisa global sobre o Yahoo! Mail. Clique aqui! 


Yahoo! Mail - Participe da pesquisa global sobre o Yahoo! Mail. Clique aqui! 

[obm-l] Re: [obm-l] Re: [obm-l] Dúvida

2011-10-24 Thread Julio César Saldaña



O mesmo visto de outro modo:

Lucas e Pedro tem a mesma velocidade. Então quando lucas sai da ponte, pedro
percorriou 2/5 da ponta, ou seja falta ainda 1/5 da ponte. Então o trem percorre
a ponte inteira no mesmo tempo que pedro percorre 1/5 da ponte, ouseja o trem é
5 vezes mais veloz que pedro, portanto su velocidade é 5x15=75.

Julio Saldaña


-- Mensaje original ---
De : obm-l@mat.puc-rio.br
Para : obm-l@mat.puc-rio.br
Fecha : Mon, 24 Oct 2011 17:08:45 -0200
Asunto : [obm-l] Re: [obm-l] Dúvida

Lucas corre 2/5 da ponte com velocidade de 15 km/h. Sendo p o comprimento da
ponte, leva (2/5)p/15 = 2p/75 h para sair da ponte.
Pedro corre 3/5 da ponte com velocidade de 15 km/h. Sendo p o comprimento da
ponte, leva (3/5)p/15 = 3p/75 h para sair da ponte.

A diferença entre o momento em que Lucas sai da ponte (momento em que o trem
entra na ponte) e o momento em que Pedro sai dela (momento em que o trem sai
da ponte) é 3p/75 - 2p/75 = p/75. Este é o tempo que o trem leva para
percorrer a ponte, ou seja, para percorrer p km.

Portanto, a velocidade do trem será dada por p/(p/75) = 75 km/h, alternativa
C

Abraços.

Hugo.

Em 24 de outubro de 2011 15:29, Jorge Paulino da Silva Filho <
jorge...@yahoo.com.br> escreveu:


**
Dois amigos, Lucas e Pedro, seguiam o leito de uma ferrovia e começaram a
atravessar uma ponte estreita na qual havia espaço apenas para o trem. No
momento em que completavam 2/5 do percurso da ponte, ouviram o trem que
se aproxima por trás deles. Lucas começou a correr de encontro ao trem,
saindo da ponte praticamente no instante em que o trem entrava. Pedro,
que correu no sentindo oposto ao sentido de Lucas, conseguir sair da ponte
praticamenteno instante em que o trem saía. Sendo 15km/h a velocidade que
Lucas
e Pedro correram, assinale a alternativa que contém a velocidade do trem:

a)60km/h
b)37km/h
c)75km/h
d)30km/h
e)67,5km/h




__
Si desea recibir, semanalmente, el Boletín Electrónico de la PUCP, ingrese a:
http://www.pucp.edu.pe/puntoedu/suscribete/

=
Instruções para entrar na lista, sair da lista e usar a lista em
http://www.mat.puc-rio.br/~obmlistas/obm-l.html
=


[obm-l] Re: [obm-l] Re: [obm-l] Dúvida

2011-11-02 Thread Kleber Bastos
É isso mesmo:
Mostrar que ∀ nº racional a/b>0, M.D.C.(a,b)=1 é válida a sentença:
f(a/b)=f(1)^a/b ( f(1) elevado a a/b)

Em 2 de novembro de 2011 20:57, Victor Hugo Rodrigues <
victorhcr.victorh...@gmail.com> escreveu:

> Como assim? Acho que falta algo aí.
>
> Em 2 de novembro de 2011 17:17, Kleber Bastos escreveu:
>
> Olá grupo,
>> Estou me enrolando nesta prova.
>>
>> Mostre q ∀ nº a/b>0, MDC(a,b) = 1,
>> é válido: f(a/b) = f(1)^a/b .
>>
>> --
>> Kleber.
>>
>
>


-- 
Kleber B. Bastos


[obm-l] RE: [obm-l] Re: [obm-l] Dúvida

2012-03-12 Thread Vanessa Nunes de Souza

Obrigada!Vanessa Nunes

Date: Mon, 12 Mar 2012 17:04:46 -0300
Subject: [obm-l] Re: [obm-l] Dúvida
From: tarsise...@gmail.com
To: obm-l@mat.puc-rio.br

a) Fazendo-se a diferença entre 

(100c+10b+a) - (100a+10b+c) = 396
99c-99a=396
c-a=4

Sendo 2c=a => a=4 e c=8, como a,b,c formando uma PA, temos que b=6

b) A área do triângulo QMN pode ser dada por S1 = [(QP/2)x(QR/2)xsenQ]/2 => 
S1=(QPxQRxsenQ)/8



A área do triângulo QPR pode ser dada por S2 = (QPxQRxsenQ)/2

Donde tiramos que S2/4 = S1

Sendo a área do quadrilátero PMNR dada por S3 = 51, e sendo S3 = S2 - S1

S2 - S2/4 = 51 => S2 = 68m²






On Mon, Mar 12, 2012 at 2:42 PM, Vanessa Nunes de Souza 
 wrote:






 Gostaria da ajuda dos colegas para resolver duas questões:
 
 



  


  


  


  


 
 



  


  


  


  


1- Um número de três algarismos, a, b e c do sistema de numeração decimal, é 
representando por abc, isto é, abc=100a+ 10b+ c. Escrito na ordem inversa(cba), 
esse número aumenta de 396 unidades. Sabendo-se qie (a,b,c) forma, nessa ordem, 
uma progressão aritmética e que c=2a, o algarismo b é igual a:

a)6   b) 5   c) 4   d) 3
2-Considere um triângulo escaleno PQR onde M e N representam, respectivamente, 
os pontos médios dos lados PQ e QR. Se a área do quadrilátero PMNR é igual a 
51m^2, a área do triângulo PQR, em m^2, é igual a: a) 68 b) 54 c)72 
  d) 84


Obrigada
Vanessa Nunes 

  

[obm-l] Re: [obm-l] Re:[obm-l] Dúvida!

2006-02-04 Thread Marcelo Salhab Brogliato



Olá,
primeiramente vamos analisar o seguinte 
problema:
x+y+z = k, x >= 0, y >= 0, z >= 
0
 
Imaginemos que vc tem k palitos de sorvete e 2 
pedras.. de quantos modos vc pode organiza-los?
(k+2)! / (k! 2!), certo?
que é igual a C(k+2, 2) .. combinação de k+2, 
tomados 2 a 2.
 
Agora, considere que cada palito é a unidade, e as 
pedras são os sinais de soma.. então, este tbem eh q quantidade de soluções 
inteiras e não negativas da equação.
Deu pra entender?
 
Analogamente vamos tentar resolver seu 
problema,
ax + by + cz = k, x>= 0, y >= 0, z >= 
0
 
Se pegarmos os casos em que x > 0, então ax 
>= a ... logo: ax - a >= 0
ax - a = X .. logo.. ax = X + a
Analogamente para os outros casos, entao, 
teremos:
X + a + Y + b + Z + c = k
X + Y + Z = k - a - b - c  X >= 0, Y >= 
0, Z >= 0
Bom, ja vimos que o numero de solucoes é: C(k - a - 
b - c, 2)
 
Agora, falta os casos em que eles podem ser zero.. 
entao:
x = 0... y > 0.. z > 0 => by + cz = k 
...
analogamente ao primeiro exemplo, teremos (k - b - 
c + 1)! / (k - b - c)! = C(k - b - c, 1)
 
Assim..
y = 0 => C(k - a - c, 1)
z = 0 => C(k - a - b, 1)
 
Agora, com 2 iguais a zero:
x = 0, y = 0... só terá solução se c | k 
podendo ser 1 ou 0
analogo para os outros..
 
Não tenho certeza da minha solucao.. apenas 
apliquei uma ideia que ja sabia (a que apresentei no comeco da 
mensagem)...
 
Abraços,
Salhab
 
 
 

  - Original Message - 
  From: 
  Luiz H. 
  Barbosa 
  To: obm-l 
  Sent: Saturday, February 04, 2006 9:13 
  PM
  Subject: [obm-l] Re:[obm-l] Dúvida!
  
  
  Como faço para achar o número de soluções de uma 
  equação do tipo ax+by+cz=k, de modo que a,b e c são 
  inteiros não-negativos e k um inteiro maior ou igual a 3?! 
  Para ser mais prático, como acharia o número de soluções de x+2y+3z=7, 
  sendo x,y e z inteiros não-negativos?! Será de suma importância a colaboração 
  dos senhores! 
   
  ===
  Bom , não conheço nenhum método.Mas você pode dar um jeitinho e resolver, 
  veja :
  x+2y+3z=7  (i)
  (x+y)+2(y+z) =7
  Isso quer dizer que a soma tem um multiplo de 2 e como os multiplos de 2 
  menores que 7 são 2,4 e 6 , dividimos o problema em 3 casos:
  1°) 2(y+z) = 2
  2°) 2(y+z) = 4
  3°)2(y+z) = 6
   
  Vou resolver só o primeiro e os demais são semelhantes:
  1°caso:
  y+z = 1 e consequentemente x+y = 5 .Resolvendo tudo em função de uma 
  variável, escolhi y , fica:
  x=4+y
  z=1-y
   
  Substituindo em (i) encontramos y=1 , com isso , x=5 e z=0.
  Faça o mesmo para os outros casos.
   
  []'s
  Luiz H. Barbosa 
  MSN: [EMAIL PROTECTED]


[obm-l] Re: [obm-l] Re: [obm-l] Dúvida

2004-01-29 Thread Eduardo Casagrande Stabel
Oi Platão e demais.

Não querendo corrigir, mas já enriquecendo a mensagem do Platão. Se n é
primo (com exceção a n=2) então Phi(n) = n-1 é par. Se n é potência de primo
n = p^i (com i>=2) então Phi(n) = p^i - p^(i-1) também é par. Já que a
função Phi é multiplicatica, isto é, se mdc(m,n)=1 então Phi(mn) = Phi(m)
Phi(n), então segue a conclusão de que, a menos para n = 2, Phi(n) é um
número par.

Para quem não conhece (a maioria), o Platão é amigo meu, de Novo Hamburgo, e
portanto também gaúcho. Saudações ao mais novo membro da lista, todos
esperamos boas contribuições como essa! Seja bem-vindo!

Abração,
Duda.


From: "Platão Gonçalves Terra Neto" <[EMAIL PROTECTED]>
> Basta ver que se p é primo, ímpar, então phi(p)=p-1, par.
> Para n=b^c, b primo, phi(b^c)=b^c-b^(c-1), que é par, ou seja,  se
> n=a1^p2*a2^p2*...an^pn, sendo ai, todos primos , distintos , n>2 e pi
> expoentes, então phi(n) é par.
> Se n=2^k, phi(n)=2^k-2^(k-1), que é par, exceção, para phi(2)=1.
> phi(1)=1.
> Logo, phi(n) é par , para todo n>2, donde ,N* não é imagem de phi(n)
> - Original Message -
> From: "André Martin Timpanaro" <[EMAIL PROTECTED]>
> To: <[EMAIL PROTECTED]>
> Sent: Thursday, January 29, 2004 8:38 PM
> Subject: [obm-l] Dúvida
>
>
> > A afirmação abaixo é verdadeira?
> >
> > Dado um número natural n não nulo existe algum natural m tal que
phi(m)=n.
> > Onde phi(x) é a função phi de Euler.
> > Em outras palavras, a imagem de phi(x) é N* ?
> >
> > André T.
> >
> > _
> > MSN Messenger: converse com os seus amigos online.
> > http://messenger.msn.com.br
> >
> >
=
> > Instruções para entrar na lista, sair da lista e usar a lista em
> > http://www.mat.puc-rio.br/~nicolau/olimp/obm-l.html
> >
=
> >
>
> =
> Instruções para entrar na lista, sair da lista e usar a lista em
> http://www.mat.puc-rio.br/~nicolau/olimp/obm-l.html
> =
>
>

=
Instruções para entrar na lista, sair da lista e usar a lista em
http://www.mat.puc-rio.br/~nicolau/olimp/obm-l.html
=


[obm-l] Re: [obm-l] Re: [obm-l] dúvida

2004-02-28 Thread Tarcio Santiago
AMIGO RAFAEL OBRIGADO POR SUA AJUDA. A RESPOSTA É 720, MAS EU ACHAVA QUE O
CERTO ERA 120, POIS O LIVRO  O TRIO DE LIVROS A,Be C é igual a B,CeA . estou
errado?
a questão dar 120 ou 720 ?
estou encucado!!?

- Original Message -
From: Rafael <[EMAIL PROTECTED]>
To: <[EMAIL PROTECTED]>
Sent: Saturday, February 28, 2004 5:45 PM
Subject: [obm-l] Re: [obm-l] dúvida


> Tarcio,
>
> Quando a ordem dos elementos envolvidos fizer diferença para a escolha,
> teremos arranjos; em caso contrário, se a ordem for indiferente, serão
> combinações. Vale ressaltar também que todo arranjo pode ser entendido
como
> resultado das permutações de uma dada combinação: P(n,k) = C(n,k)*P(k,k),
> sendo P(n,k) os "arranjos" (em inglês, seriam denominados "permutações",
> haja vista que as chamadas permutações, para nós, são somente um caso
> particular de "arranjos") e C(n,k) as combinações de n elementos tomados k
a
> k, cuja "fórmula" é a mesma dos números binomiais.
>
> Para o seu problema, se você tem 10 livros diferentes e quer formar grupos
> de três livros, não interessa se você escolhe primeiro o de capa marrom,
> depois o de casa azul e por último o de capa vermelha, ou o de capa azul
> primeiro, depois o de capa vermelha e por último o de capa marrom, ou
> qualquer outra ordem que você queira. A ordem não importa para este caso,
> pois o grupo de livros formado será o mesmo, contendo os mesmos livros.
Se,
> por exemplo, você estivesse classificando esses livros numa estante, e
> quisesse colocá-los por assunto, é claro que alguma espécie de ordem
> haveria, e aí teríamos arranjos, que, aliás, advêm do Princípio
Fundamental
> da Contagem (ou Princípio Multiplicativo, como alguns preferem).
>
> Assim, como a ordem não importa, o número de grupos diferentes que podemos
> formar será C(10,3) = 10!/(3!7!) = 120.
>
> Observe também que se fossem 10 livros, mas entre eles houvesse algum
> repetido, teríamos outro resultado: *C(10,3) = C(12,3) = 12!/(3!9!) = 220.
E
> esses 220 - 120 = 100 grupos a mais seriam os grupos formados levando-se
em
> conta 1 ou 2 ou 3 ou ... ou 10 livros repetidos. (Indiquei por *C(n,k) =
> C(n+k-1,k) as combinações completas, i.e., com repetição de elementos, de
n
> elementos tomados k a k.)
>
>
> Abraços,
>
> Rafael de A. Sampaio
>
>
>
> - Original Message -
> From: Tarcio Santiago
> To: [EMAIL PROTECTED]
> Sent: Saturday, February 28, 2004 5:09 PM
> Subject: [obm-l] dúvida
>
>
> olá amigos estou com uma dúvida na questão abaixo;?
> Dispomos de 10 livros diferentes e queremosorganizar grupos de três
livros.
> O número de gruposdiferentes que podemos formar é igual a:
> essa questão é de combinação ou arranjo? quando eu sei que é arranjo ou
> combinação?
>
> =
> Instruções para entrar na lista, sair da lista e usar a lista em
> http://www.mat.puc-rio.br/~nicolau/olimp/obm-l.html
> =
>
>


=
Instruções para entrar na lista, sair da lista e usar a lista em
http://www.mat.puc-rio.br/~nicolau/olimp/obm-l.html
=


[obm-l] Re: [obm-l] Re:[obm-l] dúvida

2004-04-25 Thread Fellipe Rossi
No caso, não entendi o porque do i*(raiz de 1998), visto que ao elevarmos ao
quadrado,  i^2= -1 e a expressão seria -1998

Creio que apenas (raiz de 1998) seja mais correto.

Porém a questão não deve ser apenas isto, a e b devem pertencer a algum
conjunto específico como os Inteiros...

Abraços,
Rossi

- Original Message -
From: "rickufrj" <[EMAIL PROTECTED]>
To: "obm-l" <[EMAIL PROTECTED]>
Sent: Sunday, April 25, 2004 2:40 AM
Subject: [obm-l] Re:[obm-l] dúvida


> -- Início da mensagem original ---
>
>   De: [EMAIL PROTECTED]
> Para: [EMAIL PROTECTED]
>   Cc:
> Data: Sat, 24 Apr 2004 23:31:13 -0300
>  Assunto: [obm-l] dúvida
>
> > como é que eu resolvo este inequação de maneira
> inteligente!!!
> >
> > | (x+1)/(-x)| >=0
> >
> > a expresão acima está em módulo.
> >
> > outra dúvida é:
> > a x b = 1998 .Sabendo que "a" e "b" são tais números
> que a diferença entre eles seja a menor possível.
> >
> ===
> Na desigualdade ,qualquer valor para x é válido .
> Já na segunda questão , se a e b puderem ser
> complexos , então a = b = i*[sqrt(1998)] .Sendo a
> menor difereça igual a zero.
>
> __
> Acabe com aquelas janelinhas que pulam na sua tela.
> AntiPop-up UOL - É grátis!
> http://antipopup.uol.com.br/
>
>
>
> =
> Instruções para entrar na lista, sair da lista e usar a lista em
> http://www.mat.puc-rio.br/~nicolau/olimp/obm-l.html
> =
>


=
Instruções para entrar na lista, sair da lista e usar a lista em
http://www.mat.puc-rio.br/~nicolau/olimp/obm-l.html
=


[obm-l] Re: [obm-l] Re: [obm-l] dúvida

2015-07-09 Thread Ralph Teixeira
Vamos generalizar para R^n: com a noção usual (Euclideana) de comprimento,
o comprimento do segmento que liga (x1,x2,...,xn) a (y1,y2,...,yn) é:

d=raiz((y1-x1)^2+(y2-x2)^2+...+(yn-xn)^2)

Esta é a noção usual de distância entre dois pontos -- confira que é o que
você conhece na reta (n=1) e no plano (n=2).

Abraço, Ralph.

2015-07-09 10:27 GMT-03:00 Pedro José :

> Bom dia!
>
> E o segmento???
>
> Em 8 de julho de 2015 21:48, Israel Meireles Chrisostomo <
> israelmchrisost...@gmail.com> escreveu:
>
>> Como posso encontrar o comprimento de um segmento de reta no espaço
>> tridimensional?Considere a origem da reta no ponto (x_0,y_0,z_0) e o final
>> da reta no ponto (x_1,y_1,z_1)
>>
>> --
>> Esta mensagem foi verificada pelo sistema de antivírus e
>> acredita-se estar livre de perigo.
>
>
>
> --
> Esta mensagem foi verificada pelo sistema de antivírus e
> acredita-se estar livre de perigo.

-- 
Esta mensagem foi verificada pelo sistema de antiv�rus e
 acredita-se estar livre de perigo.



[obm-l] Re: [obm-l] Re: [obm-l] Dúvida

2018-05-24 Thread Pedro José
Boa noite!
Minha primeira tentativa foi tudo 1. Mas aí a soma dos quadrados também é
1001=7*11*13.
As ordens de 10 mod desses fatores são 6, 1 e 6. Mas têm 1001 algarismos e
aí 6 ł 1001não serve.
Tentei outros arranjos com grupos de algarismos iguais, mas sem sucesso.
Mas o que não compreendo é porque não há a divulgação da resposta.
Saudações,
PJMS

Em Qui, 24 de mai de 2018 21:09, Anderson Torres <
torres.anderson...@gmail.com> escreveu:

> Em 23 de maio de 2018 21:41, Pedro José  escreveu:
> > Boa noite!
> > Há algum motivo para não disponibilizarem o gabarito da olimpiada de
> mayo?
> > Gostaria de ver a solução de um problema da XXII olimpiada:
> > Dizemos que um número inteiro positivo é qua-divi se é divisível pela
> > soma dos quadrados de seus dígitos, e além disso nenhum de seus
> dígitos
> > é igual a zero.
> > a) Encontre um número qua-divi tal que a soma de seus dígitos seja 24.
> > b) Encontre um número qua-divi tal que a soma de seus dígitos seja
> 1001.
>
> Só jogando uma ideia solta, eu tentaria calcular para casos como
> 111...11. A soma dos dígitos é N e o número é (10^N-1)/9
>
> Se isso não servir, talvez 111..12222 também possa ser
> útil.
>
> >
> > Grato.
> > Saudações,
> > PJMS
> >
> > --
> > Esta mensagem foi verificada pelo sistema de antivírus e
> > acredita-se estar livre de perigo.
>
> --
> Esta mensagem foi verificada pelo sistema de antivírus e
>  acredita-se estar livre de perigo.
>
>
> =
> Instru�ões para entrar na lista, sair da lista e usar a lista em
> http://www.mat.puc-rio.br/~obmlistas/obm-l.html
> =
>

-- 
Esta mensagem foi verificada pelo sistema de antiv�rus e
 acredita-se estar livre de perigo.



[obm-l] Re: [obm-l] Re: [obm-l] Dúvida

2018-05-25 Thread Pedro José
Boa tarde!
Creio ter conseguido.
Criei um número com fatores congruentes a 1 mod 6, exceto o 5 e o11.
Além disso a ordem de 10 mod desses fatores é sempre 6, exceto o 5 e o 11
que será 1, melhor. Mas o 5 não tem problema.
Então o objetivo é firmar um número da seguinte forma:
...AB...B...C concatenado com o número criado, mencionado
anteriormente.
O número criado foi:
84.259.175 = 5^2*7^2*11*13^2*37
Então a soma dos algarismos desse número é 41 e dos quadrados de seus
algarismos é 265.
No número que pretendo formar o número de algarismos em bloco será múltiplo
de 6.
Então fica o sistema para apenas dois blocos:
ax+by= (1001-41)/6=160
a^2*x +b^2*y=(S2 -265)/6.
Onde x e y é a quantidade de repetições de blocos de 6 algarismos e a e b
são os algarismos e S2 é a soma dos quadrados de todos dígitos.
Agora preciso criar S2 que feche com o problema. Tem que ser 1 mod 6, para
quando subtrair 265, ser divisível por 6. Deve ser um divisor do número
criado no início. 5^2*7^2*11*13^2*37.
Seja S2=5005=5*7*11*13
xa+yb=160
xa^2+yb^2= (5005-265)/6=790.
Como 6| 790 - 160, 1 e 3 formam uma boa escolha, mas infortunadamente, o
número de blocos de 1 dá negativo.
Então introduzi 9 blocos de 8 para acertar, já que há liberdade.

Aí dão 9 blocos de 8, 25 blocos de 1 e 21 blocos de três, concatenação ao
final, 84.259.175.
É o número fica.
10^274*8*(10^54-1)/9+10^124*(10^150-1)/9+10^8*3*(10^126)/9+
5^2*7^2*11*13^2*17.
Como
10^6 =1 mod p, com p=7 ou p=11 ou p= 13 e 5 |10, S2=5*7*11*13, S2 divide
cada parcela e portanto o número.
O número são 54 algarismos 8, seguidos de 150 algarismos 1,seguidos de126
algarismos 3 seguidos de 84259175.
Deve ter um jeito mais elegante de resolver.
Saudações,
PJMS

Em Qui, 24 de mai de 2018 23:51, Pedro José  escreveu:

> Boa noite!
> Minha primeira tentativa foi tudo 1. Mas aí a soma dos quadrados também é
> 1001=7*11*13.
> As ordens de 10 mod desses fatores são 6, 1 e 6. Mas têm 1001 algarismos e
> aí 6 ł 1001não serve.
> Tentei outros arranjos com grupos de algarismos iguais, mas sem sucesso.
> Mas o que não compreendo é porque não há a divulgação da resposta.
> Saudações,
> PJMS
>
> Em Qui, 24 de mai de 2018 21:09, Anderson Torres <
> torres.anderson...@gmail.com> escreveu:
>
>> Em 23 de maio de 2018 21:41, Pedro José  escreveu:
>> > Boa noite!
>> > Há algum motivo para não disponibilizarem o gabarito da olimpiada de
>> mayo?
>> > Gostaria de ver a solução de um problema da XXII olimpiada:
>> > Dizemos que um número inteiro positivo é qua-divi se é divisível
>> pela
>> > soma dos quadrados de seus dígitos, e além disso nenhum de seus
>> dígitos
>> > é igual a zero.
>> > a) Encontre um número qua-divi tal que a soma de seus dígitos seja 24.
>> > b) Encontre um número qua-divi tal que a soma de seus dígitos seja
>> 1001.
>>
>> Só jogando uma ideia solta, eu tentaria calcular para casos como
>> 111...11. A soma dos dígitos é N e o número é (10^N-1)/9
>>
>> Se isso não servir, talvez 111..12222 também possa ser
>> útil.
>>
>> >
>> > Grato.
>> > Saudações,
>> > PJMS
>> >
>> > --
>> > Esta mensagem foi verificada pelo sistema de antivírus e
>> > acredita-se estar livre de perigo.
>>
>> --
>> Esta mensagem foi verificada pelo sistema de antivírus e
>>  acredita-se estar livre de perigo.
>>
>>
>> =
>> Instru�ões para entrar na lista, sair da lista e usar a lista em
>> http://www.mat.puc-rio.br/~obmlistas/obm-l.html
>> =
>>
>

-- 
Esta mensagem foi verificada pelo sistema de antiv�rus e
 acredita-se estar livre de perigo.



[obm-l] Re: [obm-l] Re: [obm-l] Dúvida

2018-05-31 Thread Pedro José
Bom dia!
Corrigindo uma grande bobagem, confirme me alertado.
A ordem de 10 nos 11 é 2 e não 1. Mas como 2|6, não muda nada.

Saudações,
PJMS

Em Sex, 25 de mai de 2018 14:37, Pedro José  escreveu:

> Boa tarde!
> Creio ter conseguido.
> Criei um número com fatores congruentes a 1 mod 6, exceto o 5 e o11.
> Além disso a ordem de 10 mod desses fatores é sempre 6, exceto o 5 e o 11
> que será 1, melhor. Mas o 5 não tem problema.
> Então o objetivo é firmar um número da seguinte forma:
> ...AB...B...C concatenado com o número criado, mencionado
> anteriormente.
> O número criado foi:
> 84.259.175 = 5^2*7^2*11*13^2*37
> Então a soma dos algarismos desse número é 41 e dos quadrados de seus
> algarismos é 265.
> No número que pretendo formar o número de algarismos em bloco será
> múltiplo de 6.
> Então fica o sistema para apenas dois blocos:
> ax+by= (1001-41)/6=160
> a^2*x +b^2*y=(S2 -265)/6.
> Onde x e y é a quantidade de repetições de blocos de 6 algarismos e a e b
> são os algarismos e S2 é a soma dos quadrados de todos dígitos.
> Agora preciso criar S2 que feche com o problema. Tem que ser 1 mod 6, para
> quando subtrair 265, ser divisível por 6. Deve ser um divisor do número
> criado no início. 5^2*7^2*11*13^2*37.
> Seja S2=5005=5*7*11*13
> xa+yb=160
> xa^2+yb^2= (5005-265)/6=790.
> Como 6| 790 - 160, 1 e 3 formam uma boa escolha, mas infortunadamente, o
> número de blocos de 1 dá negativo.
> Então introduzi 9 blocos de 8 para acertar, já que há liberdade.
>
> Aí dão 9 blocos de 8, 25 blocos de 1 e 21 blocos de três, concatenação ao
> final, 84.259.175.
> É o número fica.
> 10^274*8*(10^54-1)/9+10^124*(10^150-1)/9+10^8*3*(10^126)/9+
> 5^2*7^2*11*13^2*17.
> Como
> 10^6 =1 mod p, com p=7 ou p=11 ou p= 13 e 5 |10, S2=5*7*11*13, S2 divide
> cada parcela e portanto o número.
> O número são 54 algarismos 8, seguidos de 150 algarismos 1,seguidos de126
> algarismos 3 seguidos de 84259175.
> Deve ter um jeito mais elegante de resolver.
> Saudações,
> PJMS
>
> Em Qui, 24 de mai de 2018 23:51, Pedro José 
> escreveu:
>
>> Boa noite!
>> Minha primeira tentativa foi tudo 1. Mas aí a soma dos quadrados também é
>> 1001=7*11*13.
>> As ordens de 10 mod desses fatores são 6, 1 e 6. Mas têm 1001 algarismos
>> e aí 6 ł 1001não serve.
>> Tentei outros arranjos com grupos de algarismos iguais, mas sem sucesso.
>> Mas o que não compreendo é porque não há a divulgação da resposta.
>> Saudações,
>> PJMS
>>
>> Em Qui, 24 de mai de 2018 21:09, Anderson Torres <
>> torres.anderson...@gmail.com> escreveu:
>>
>>> Em 23 de maio de 2018 21:41, Pedro José  escreveu:
>>> > Boa noite!
>>> > Há algum motivo para não disponibilizarem o gabarito da olimpiada de
>>> mayo?
>>> > Gostaria de ver a solução de um problema da XXII olimpiada:
>>> > Dizemos que um número inteiro positivo é qua-divi se é divisível
>>> pela
>>> > soma dos quadrados de seus dígitos, e além disso nenhum de seus
>>> dígitos
>>> > é igual a zero.
>>> > a) Encontre um número qua-divi tal que a soma de seus dígitos seja
>>> 24.
>>> > b) Encontre um número qua-divi tal que a soma de seus dígitos seja
>>> 1001.
>>>
>>> Só jogando uma ideia solta, eu tentaria calcular para casos como
>>> 111...11. A soma dos dígitos é N e o número é (10^N-1)/9
>>>
>>> Se isso não servir, talvez 111..12222 também possa ser
>>> útil.
>>>
>>> >
>>> > Grato.
>>> > Saudações,
>>> > PJMS
>>> >
>>> > --
>>> > Esta mensagem foi verificada pelo sistema de antivírus e
>>> > acredita-se estar livre de perigo.
>>>
>>> --
>>> Esta mensagem foi verificada pelo sistema de antivírus e
>>>  acredita-se estar livre de perigo.
>>>
>>>
>>> =
>>> Instru�ões para entrar na lista, sair da lista e usar a lista em
>>> http://www.mat.puc-rio.br/~obmlistas/obm-l.html
>>> =
>>>
>>

-- 
Esta mensagem foi verificada pelo sistema de antiv�rus e
 acredita-se estar livre de perigo.



[obm-l] Re: [obm-l] Re: [obm-l] Dúvida

2018-08-23 Thread Pedro José
Boa noite!
É fato.
Grato,
PJMS.

Em Qua, 22 de ago de 2018 23:00, Ralph Teixeira 
escreveu:

> Acho que nao... Ah, se eu entendi corretamente, (3,6,9) e (3,5,12) seria
> um contra-exemplo.
>
> Abraco, Ralph.
>
>
> On Wed, Aug 22, 2018 at 8:06 PM Pedro José  wrote:
>
>> Boa noite.
>>
>> Sejam duas sequências em ordem crescente com ai,bi >0 e k elementos ambas.
>> se:
>> (a1+a2+a3+...+ak)/(b1+b2+b3+...+bk)=a1a2a3a3...ak/(b1b2b3...bk) podemos
>> dizer que
>> ai=bi para 0>
>> Grato,
>> PJMS
>>
>> --
>> Esta mensagem foi verificada pelo sistema de antivírus e
>> acredita-se estar livre de perigo.
>
>
> --
> Esta mensagem foi verificada pelo sistema de antivírus e
> acredita-se estar livre de perigo.

-- 
Esta mensagem foi verificada pelo sistema de antiv�rus e
 acredita-se estar livre de perigo.



[obm-l] Re: [obm-l] Re: [obm-l] Dúvida

2019-05-19 Thread Pedro José
Bom dia!
Anderson,
obrigado. Porém faltou-me saber se os entendimentos anteriores estão
corretos.

Grato,
PJMS

Em sáb, 18 de mai de 2019 13:27, Anderson Torres <
torres.anderson...@gmail.com escreveu:

>
>
> Em sex, 17 de mai de 2019 às 10:49, Pedro José 
> escreveu:
>
>> Bom dia!
>>
>> Tenho uma dúvida sobre os simbolismos, que aparecem recorrentemente, em
>> artigos sobre teoria dos números, mas que não encontro a definição :
>> Z[i]/(α) - Entendi como o conjunto das classes de equivalências mod α em
>> Z{i}
>> Z[i]/(α)* - Entendi como as classes de equivalência mod α em Z[i], que
>> são inversíveis.
>>
>> Tentando compreender uma demonstração de que todos os números que não
>> podem ser escritos da forma 4^k(8m+7) com k,m>=0, aceitam ser escritos como
>> a soma de três parcelas, todas quadrados, me deparei com  [image:
>> image.png] . O que significa?
>>
>
> Algo como números da forma a+b*sqrt(m) onde a e b são racionais.
>
>
>
>>
>> Os outros entendimentos estão corretos?
>>
>> Saudações,
>> PJMS
>>
>>
>> --
>> Esta mensagem foi verificada pelo sistema de antivírus e
>> acredita-se estar livre de perigo.
>
>
> --
> Esta mensagem foi verificada pelo sistema de antivírus e
> acredita-se estar livre de perigo.

-- 
Esta mensagem foi verificada pelo sistema de antiv�rus e
 acredita-se estar livre de perigo.



[obm-l] Re: [obm-l] Re: [obm-l] Dúvida

2019-12-04 Thread Pedro José
Boa noite!
As retas são cônicas degeneradas. Mas são cônicas.
Definição de cônica :   Dada duas retas g,l concorrentes (cuja interseção é
{V} no |R3 que não sejam perpendiculares e um plano Pi. A interseção desse
plano com o cone K, reto de vértice V e eixo l , obtido pela rotação da
reta g ao redor do ponto V é uma cônica. Podemos ter uma reta, duas retas
ou um ponto como cônicas degeneradas.
Você poderia até ter mencionado o conjunto vazio que não é uma cônica.
x^2+y^2=-1.
Mas na verdade, eu não me expressei com rigor, o que queria dizer é que se
escrevermos a função quadrática F(x,y)= 0, que represente a cônica
(degenerada ou não) F(x,y) é suave? Ou as cônicas suaves devem ser não
degeneradas apenas?
Outrossim, discordo do seu argumento "...geralmente é mais útil que as
definições dos objetos importantes não excluam os casos particulares.."
Geralmente não é o balizador e sim a definição.
1 não é primo. Pois define-se que um primo deve ter dois divisores
positivos e 1 só possui um. Poderia argumentar, na sua linha, os dois
divisores coincidentes (os que afirmam é divisível por si e pela unidade)
O quadrado por definição está claro que é retângulo.
A definição da elipse é de que a soma das distâncias a dois pontos fixos (e
não um) é constante. Aí tem a forçação de se considerar dois como um só.
Não existe dois pontos coincidentes. Se são dois são distintos. Podemos
representar algo de várias maneiras mas se são iguais é só um, representado
de várias maneiras. Qual o cardinal do conjunto de focos de uma elipse, no
caso de você aceitar a elipse com um único foco?
Como é a prova que só existe um vazio. Por hipótese há mais de um vazio,
vazio1 e vazio2 e no fim chega-se a conclusão que vazio1 = vazio2 e
portanto absurdo.Ora, podemos ter vazios coincidentes.
Amigo, você afirma: "*Nunca vi ninguém definir elipse de uma forma que
exclua os círculos*."
Você nem se deu ao trabalho de ler a minha nota, antes de comentar, ou
então me corrija se o círculo atende à:
Lugar geométrico do plano em que a razão entre a distância de um ponto ao
foco direito e a distância entre esse ponto e uma reta (diretriz direita) é
constante e menor que 1 e igual a excentricidade da cônica.
Como a excentricidade da circunferência é zero, teríamos que ter um ponto
fixo em que a distância de cada ponto da circunferência até esse ponto
fosse zero. E se na definição tem foco direito está implícito que há um
esquerdo. Vale a definição para foco esquerdo. Só atenderia se
considerarmos o ponto como uma circunferência de raio zero. E só para esse
caso e ainda aceitarmos que quando há só um foco ele tanto é direito quanto
esquerdo. Grato pelos comentários. Mas as dúvida persistem.

Saudações,
PJMS



Em qua., 4 de dez. de 2019 às 19:59, Pedro Angelo 
escreveu:

> Em matemática, geralmente é mais útil que as definições dos objetos
> importantes não excluam os casos particulares. Um quadrado é um
> retângulo? Se vc quiser que a definição de "retângulo" inclua somente
> quadriláteros com ângulos retos que não sejam quadrados, vc tem que
> explicitar a parte do "não sejam quadrados" na definição. A definição
> mais simples, "retângulo é um quadrilátero cujos ângulos são todos
> retos" (como o nome já diz!) inclui o quadrado como caso especial. Uma
> coisa parecida ocorre com a elipse. Se vc quiser excluir o círculo, vc
> teria que especificar na definição que vc quer focos distintos. A
> definição mais simples, que cita os focos como sendo "dois pontos", ao
> invés de "dois pontos distintos", inclui o círculo como caso especial.
> E é útill que inclua mesmo. Por exemplo, se vc pensar o círculo como
> sendo um tipo especial de elipse, vc pode enunciar o seguinte teorema:
> "A imagem de uma elipse por uma transformação afim é outra elipse."
> Mas se vc achar que um círculo não é uma elipse, então o teorema (da
> forma que foi enunciado) não vale mais. A questão é que praticamente
> qualquer propriedade interessante apresentada por "elipses
> não-circulares" também será compartilhada pelos círculos. É raro em
> matemática vc precisar de uma elipse que seja proibida de ser um
> círculo. Nunca vi ninguém definir elipse de uma forma que exclua os
> círculos.
>
> Sobre a suavidade: da forma que vc escreveu, eu diria que está um
> pouco ruim. Por exemplo, a função
> F(x,y)=x^2-y^2
> é uma função suave (vc consegue calcular dF/dx e dF/dy, por exemplo).
> Mas vc diria que a equação F(x,y)=0 é uma "cônica suave"? Repare que
> essa equação descreve duas retas que se cruzam na origem. Outras
> funções problemáticas são F(x,y)=x^2+y^2 e F(x,y)=0.
>
> Se F(x,y) é um polinômio de segundo grau em x e y, então F(x,y)=0 é
> uma cônica, e eu diria que essa cônica é "suave" se nenhum dos pontos
> dela (pontos (x,y) tais que F(x,y)=0) satisfaz ao mesmo tempo dF/dx=0
> e dF/dy=0. O fato de pelo menos uma das derivadas parciais de F ser
> não-nula garante que não encontraremos problemas como os do parágrafo
> acima.
>
> abraços!
>
>
> Le mer. 4 déc. 2019 à 19:10, Pedro

[obm-l] Re: [obm-l] Re: [obm-l] Dúvida

2021-11-22 Thread Pedro José
Boa tarde!

Grato, pela ajuda!
Não conheço.
Vou abrir um leque de estudo para tentar entender!
Valeu a curiosidade, com o que cheguei consegui matar o problema.
Genericamente, consegui que a solução levaria a uma expressão que era um
quadrado perfeito,esse era o objetivo. Só que me deu curiosidade, quanto a
resolução. Vou me enveredar no tema.

Cordialmente,
PJMS.

Em ter., 16 de nov. de 2021 às 17:29, Prof. Douglas Oliveira <
profdouglaso.del...@gmail.com> escreveu:

> Equação de Pell
>
> Em seg., 15 de nov. de 2021 13:36, Pedro José 
> escreveu:
>
>> Boa tarde!
>>
>> Alguém saberia como resolver a seguinte equação:
>>
>> x^2-7y^2=1, x,y em Z?
>>
>> Fiz a-7b=1 e achei a= 8 +7k e b=1 +K
>> Logo fica fácil que para k=-1 funciona x^2=1 e y^2=0.
>> Também funciona para k=8 x^2=64 e y^2=9.
>> Mas não sei nem como achar mais soluções nem como provar que só são essas.
>> Alguém poderia me dar uma orientação?
>>
>> Cordialmente,
>> PJMS
>>
>> --
>> Esta mensagem foi verificada pelo sistema de antivírus e
>> acredita-se estar livre de perigo.
>
>
> --
> Esta mensagem foi verificada pelo sistema de antivírus e
> acredita-se estar livre de perigo.

-- 
Esta mensagem foi verificada pelo sistema de antiv�rus e
 acredita-se estar livre de perigo.



[obm-l] Re:[obm-l] RE: [obm-l] Dúvida - porcentagem

2003-03-15 Thread efritscher
Eu fui pela lógica, se o preço era 1 e a quantidade 1, o 
novo preço é 0,8 e a quantidade 1,6.
1,6 dividido por 0,8 resulta em 2.
Logo a resposta é c (duplicar). Como mostra o 
companheiro abaixo.


> Originalmente, tínhamos um preço p1 e uma quandidade q1
 vendida no
> período considerado. Na nova sitação, temos um preço p2
= 0,8p1 e uma
> quantidade q2 tal que p2q2 = 0,8 p1q1 = 1,6 p1q1. Logo,
 q2 = 2q1. A
> resposta certa é a c. Trata-
se de um produto de grande elasticidade
> preço. (elasticidade = 100/20 = 5)
> Artur   
> 
> >Subject: [obm-l] Dúvida - porcentagem
> >
> >Por favor alguém tem alguma resolução pra esse exercíc
io:
> >(Unificado - Vunesp -
 2003) Um fabricante de um produto estima que uma
> >redução de 20% no preço ao consumidor implicará um aum
ento de 60% no
> >faturamento. Verificadas estas condições, a quantidade
 vendida do
> produto,
> >em relação à situação anterior à redução de preços, de
verá:
> >
> >a) quadruplicar
> >b) triplicar
> >c) duplicar
> >d) permanecer constante
> >e) cair pela metade
> >
> >Obrigado
> >Rodrigo
> >--
> >__

> >Sign-up for your own FREE Personalized E-
mail at Mail.com
> >http://www.mail.com/?sr=signup
> >
> >==
=
> ==
> >Instruções para entrar na lista, sair da lista e usar 
a lista em
> >http://www.mat.puc-rio.br/~nicolau/olimp/obm-l.html
> >O administrador desta lista é <[EMAIL PROTECTED]>
> >==
=
> ==
> 
> ===
==
> Instruções para entrar na lista, sair da lista e usar a
 lista em
> http://www.mat.puc-rio.br/~nicolau/olimp/obm-l.html
> O administrador desta lista é <[EMAIL PROTECTED]>
> ===
==
> 

 
__
E-mail Premium BOL
Antivírus, anti-spam e até 100 MB de espaço. Assine já!
http://email.bol.com.br/


=
Instruções para entrar na lista, sair da lista e usar a lista em
http://www.mat.puc-rio.br/~nicolau/olimp/obm-l.html
O administrador desta lista é <[EMAIL PROTECTED]>
=


[obm-l] Re: [obm-l] Re: [obm-l] Dúvida-Geo.Esp.

2003-07-01 Thread Nicolau C. Saldanha
On Mon, Jun 30, 2003 at 04:45:18PM -0300, Eduardo Henrique Leitner wrote:
> se o pontos forem coplanares eles determinarao apenas um plano nao eh? tipo, 
> considerem esses pontos contidos no plano do monitor
> 
> 
>  ..
>   ...
> 
> 
> tem exatamente tres colineares e determina um plano soh...

Você tem toda a razão. Tomei os 5 pontos em posição geral em R^3
a menos, claro, da condição de existirem 3 pontos em linha reta.

Isso só deixa mais claro o quanto o enunciado é insatisfatório.

[]s, N.
=
Instruções para entrar na lista, sair da lista e usar a lista em
http://www.mat.puc-rio.br/~nicolau/olimp/obm-l.html
=


[obm-l] Re: [obm-l] Re: [obm-l] Dúvida-Geo.Esp.

2003-07-01 Thread Nicolau C. Saldanha
On Mon, Jun 30, 2003 at 06:30:05PM -0300, Ariel de Silvio wrote:
> Nicolau, nao entendi as igualdades, pq p1p2p4 = p1p3p4 = p2p3p4 e p1p2p5 =
> p1p3p5 = p2p3p5??  na minha cabeca esses sao planos distintos tb... 

O plano p1p2p4 contem a reta p1p2, logo p3, sendo colinear a p1 e p2,
pertence ao plano p1p2p4.

[]s, N.
=
Instruções para entrar na lista, sair da lista e usar a lista em
http://www.mat.puc-rio.br/~nicolau/olimp/obm-l.html
=


[obm-l] Re: [obm-l] Re: [obm-l] DÚVIDA... ajudarr

2003-08-14 Thread Henrique Patrício Sant'Anna Branco
> > 2) Num triângulo retângulo, a hipotenusa é o triplo de um dos catetos.
> > Considerando   o ângulo oposto ao menor lado, podemos afirmar que tg   +
> sec
> >   é igual a  ?
>
> Essa é meio difícil de explicar... Tente fazer o desenho de um triângulo
> retângulo, use trigonometria e o fato de que sec(x) = 1/cos(x).
> Assim, sec(x) + tan(x) = 4B/A.

Esqueci de dizer o que são A e B... A é o cateto adjacente a tal ângulo e B
é o cateto oposto (o menorzinho).

=
Instruções para entrar na lista, sair da lista e usar a lista em
http://www.mat.puc-rio.br/~nicolau/olimp/obm-l.html
=


[obm-l] RE: [obm-l] RE: [obm-l] Dúvida Indução

2012-05-16 Thread Thiago Bersch

Então eu estava tentando fazer mas parava no mesmo ponto, fazia 
2+5+8+...+(3n-1)+[(3n-1)+1], chegando aí eu me perco

From: joao_maldona...@hotmail.com
To: obm-l@mat.puc-rio.br
Subject: [obm-l] RE: [obm-l] Dúvida Indução
Date: Mon, 14 May 2012 15:24:47 -0300




Vamos dizer que para n respeite a formula
Logo 2+4+6+...+2n=n.(n+1)
Somando 2n+2
2+4+6+...+(2n+2=n(n+1)+2n+2=(n+1)(n+2) que respeita a formula
Logo se vale para n, vale para n+1
Como vale para 1, vale para 2, e  entao para 3, 4, 5...
Vale para qualquer natural

Tente fazer o segundo agora
[]s Joao

From: thiago_t...@hotmail.com
To: obm-l@mat.puc-rio.br
Subject: [obm-l] Dúvida Indução
Date: Mon, 14 May 2012 01:09:39 -0300





 2 + 4 + . . . + 2n. 2 + 5 + 8 + . . . + (3n-1).Bem eu sei que o primeiro irá 
dar n(n+1) e o segundo n(3n+1)/2O que em si eu não entendi o resultado O 
primeiro eu tentei fazer assim:2+4...+2n + n+2n+(2n+1),  e fiquei parado nisso 
e o segunda também, gostaria de uma explicação passo-a-passo pois não entendo.  


  

[obm-l] RE: [obm-l] Re: [obm-l] Dúvida Raiz

2005-12-31 Thread Guilherme Neves
ah, completando minha resposta.. no campo dos complexos, utilize as formulas de De Moivre que você obterá as raízes com os dois sinais.


=
Instruções para entrar na lista, sair da lista e usar a lista em
http://www.mat.puc-rio.br/~nicolau/olimp/obm-l.html
=


[obm-l] Re: [obm-l] RE: [obm-l] dúvida fatorial

2006-04-03 Thread Ronaldo Luiz Alonso

Eu havia imaginado vagamente (a tempos atrás)
tudo o que o professor Paulo
colocou nesta mensagem (Show de Bola).
  Só que não tinha exemplos concretos nem clareza
de idéias e também nem citações suficientes
para explicitá-las como as que foram por ele colocadas.

   A moral disso tudo é que devemos sempre
QUESTIONAR aquilo que nos é ensinado e da maneira
como é ensidado, pois podemos frequentemente nos
deparar com situações práticas onde a teoria precisa
ser ligeiramente adaptada e/ou a interpretação IPSIS
LITERIS da teoria pode tornar inviável a sua aplicação.
 O caso das geometrias não euclidianas são um exemplo
prático deste caso.


"WHAT I CAN'T CREATE I CAN'T UNDERSTAND"
-- RICHARD FEYNMAN.



- Original Message - 
From: "Paulo Santa Rita" <[EMAIL PROTECTED]>

To: 
Sent: Monday, April 03, 2006 12:29 PM
Subject: [obm-l] RE: [obm-l] dúvida fatorial



Ola Reginaldo e demais
colegas desta lista ... OBM-L,
( estou escrevendo sem acentos )

Porque e conveniente ... Este postulado ( 0!=1 ) e consistente com outras 
crencas e as implicacoes dele sao uteis na pratica. De fato :


1) ACREDITANDO que a FUNCAO GAMA e a generalizacao do conceito de 
fatorial, e possivel DEDUZIR este postulado.
2) Admitindo este postulado, calculos combinatorios com numeros binomiais 
ficam mais faceis, sinteticos e concordam com a nossa PRATICA habitual de 
contar.


Entretanto :

3) Nao existe nenhuma prova ou razao apoditica de que a funcao gama seja A 
GENERALIZACAO do conceito de fatorial. Muito provavelmente ela e UMA 
GENERALIZACAO, sem nenhuma vantagem logica sobre outras potencialmente 
possiveis ( e potencialmente interessantes ! )
4) Em Matematica, o fato da pratica confirmar a teoria nao e uma razao 
definitiva para nao modificarmos esta teoria. Basta lembrar do nascimento 
das Geometrias nao-euclidianas ...


Quando voce olha para um numero binomial BINOM(N,P) e o define assim :

BINOM(N,P) = N! / ( P! * (N-P)! )

Voce, em verdade, esta ADMITINDO PREVIAMENTE A DEFINICAO DE FATORIAL e 
definindo uma funcao a duas variaveis com certas restricoes. Mas voce 
tambem poderia fazer assim :


Defino :

BINOM(N,0)=1 onde N=0,1,2,...
BINOM(N+1,P)=BINOM(0,P-1) + ... + BINOM(N,P-1) onde P=1,2,...

Neste caso voce vai obter OS MESMOS RESULTADOS sem lancar mao de uma 
definicao previa de fatorial. Mas, o que nos impede de definir :


BINOM(N,0)=2 onde N=0,1,2, ...
BINOM(N+1,P)=BINOM(0,P-1) + ... + BINOM(N,P-1) onde P=1,2,...

No primeiro caso voce obtem o bem conhecido TRIANGULO DE PASCAL, que 
chamaremos doravante de PASCAL. No segundo caso, um "PASCAL DOBRADO" ou 
2*PASCAL :


2
2 2
2 4 2
2 6 6 2
...

E no entanto, o 2*PASCAL mantem formalmente as mesmas propriedades basicas 
( Ex : Teorema das Colunas ) do triangulo tradicional, conforme se ve 
facilmente ...


Quanto vale 0! no 2*PASCAL ?

BINOM(N,0) = 2 = N! / (0! * (N-0)! ) => 0!=1/2

Veja que agora temos maior liberdade. Nao somos mais escravos da 
postulacao 0!=1


Evidentemente, no 2*PASCAL,  nao podemos mais interpretar BINOM(N,P) como 
o numero de combinacoes com P elementos que podemos formar se dispormos de 
N elementos. Bom, isso e decisivo ? E o noumeno sagrado que nao se pode 
tocar ? Sera que o 2*PASCAL nao nos permite fazer interpretacoes 
igualmente interessantes ?


Para um K*PASCAL, defino : NIC=1/0! . Assim, o triangulo de pascal e o 
1*PASCAL=PASCAL com NIC=1. Para cada real NIC ha um triangulo bem 
determinado com potenciais interpretacoes nao menos interessantes.


Finalmente, me permito uma digressao. Pode-se definir a sequencia de 
fibonaci pelo triangulo de pascal. Basta partir da coluna zero e "subir" 
em diagonal, somando os numeros binomiais. Obteremos assim esta sequencia 
tao conhecida.


( Veja : Um ensaio sobre a beleza na Matematica - Huntley - Editora Univ. 
de Brasilia )


Se fizermos NIC=fi, fi = LIM f(n+1)/f(n) onde f(n) e o enesimo termo da 
sequencia de fibonaci, qual e o "triangulo tipo Pascal" correspondente ?


Um Abraco a Todos
Paulo Santa Rita
2,1225,030406





  From: reginaldo.monteiro
  To: obm-l
  Sent: Monday, April 03, 2006 9:49 AM
  Subject: [obm-l] dúvida fatorial


  Bom dia,

  Alguém saberia me informar por que 0! = 1?

  Obrigado

  Reginaldo


_
Seja um dos primeiros a testar o novo Windows Live Mail Beta. Acesse 
http://www.ideas.live.com/programpage.aspx?versionId=5d21c51a-b161-4314-9b0e-4911fb2b2e6d


=
Instruções para entrar na lista, sair da lista e usar a lista em
http://www.mat.puc-rio.br/~nicolau/olimp/obm-l.html
=



=
Instruções para entrar na lista, sair da lista e usar a lista em
http://www.mat.puc-rio.br/~nicolau/olimp/obm-l.html
=


[obm-l] Re: [obm-l] Re: [obm-l] dúvida fatorial

2006-04-04 Thread Ronaldo Luiz Alonso
Qualquer valor diferente de "um" atribuído por "convenção" estaria negando 
a definição de fatorial.


SE considerarmos  a interpretação de fatorial
como número de bijeções de um conjunto com n
elementos em um conjunto com n elementos
e SE considerarmos a definição de números
binomiais em termos de fatorial como usualmente
nos são apresentadas, aí podemos dizer que o que
vc escreveu está correto. Não há como trocar
a definição sem causar conflitos.

Mas todas essas definições são, de fato, convenções.
Então a definição 0! = 1 também (do mesmo jeito)
é uma convenção.
Não consigo ver como não seria com o que
nos foi apresentado até agora  ...




Ojesed.

- Original Message - 
From: "Nicolau C. Saldanha" <[EMAIL PROTECTED]>

To: 
Sent: Monday, April 03, 2006 4:19 PM
Subject: Re: [obm-l] dúvida fatorial


On Mon, Apr 03, 2006 at 09:49:58AM -0300, reginaldo.monteiro wrote:

Alguém saberia me informar por que 0! = 1?


Alguém já respondeu corretamente que isto é uma convenção,
mas acho que há mais para ser dito.

A interpretação combinatória para n! é que este é o número
de permutações de um conjunto A com n elementos. Recapitulando,
uma permutação de A é uma função bijetora f:A->A, ou,
equivalentemente, um subconjunto F de AxA (o gráfico de f)
tal que, para todo a em A:
* existe um único b em A tal que (a,b) pertence a F;
* existe um único c em A tal que (c,a) pertence a F.

Com esta definição, se A = 0 (vazio) então F = 0 é o gráfico
de uma bijeção f:A->A, a função vazia. As condições para verificar
que f é bijetora são satisfeitas por vacuidade. É bem claro
que esta é a única permutação de A, donde 0!=1.

[]s, N.
=
Instruções para entrar na lista, sair da lista e usar a lista em
http://www.mat.puc-rio.br/~nicolau/olimp/obm-l.html
=


--
No virus found in this incoming message.
Checked by AVG Free Edition.
Version: 7.1.385 / Virus Database: 268.3.4/299 - Release Date: 31/3/2006


=
Instruções para entrar na lista, sair da lista e usar a lista em
http://www.mat.puc-rio.br/~nicolau/olimp/obm-l.html
=



=
Instruções para entrar na lista, sair da lista e usar a lista em
http://www.mat.puc-rio.br/~nicolau/olimp/obm-l.html
=


[obm-l] Re: [obm-l] RE: [obm-l] dúvida fatorial

2006-04-12 Thread Ronaldo Luiz Alonso
Um conjunto A de funcoes analiticas, duas a duas distintas,  definidas em 
C ( C e o conjunto dos numeros complexos )  tal que para cada z 
pertencente a C fixado,  o conjunto { f(z), f variando em A}  seja 
enumeravel.


Pergunto : A e um conjunto enumeravel ?

E respondo propondo um exercicio :
Prove que se a HIPOTESE DO CONTINUO e falsa, entao qualquer A sera sempre 
um conjunto enumeravel. Se, porem, a HIPOTESE DO CONTINUO e verdadeira, 
entao existe ao menos um conjunto A não enumeravel.


 Parece uma afirmação legal.   Se a imagem da função é um conjunto 
enumerável então #f(C) = ALEPH0 e a função tem
que ter um número infinito de descontinuidades se a hipótese for verdadeira. 
Logo a quantidade o número de funções
é enumerável e por conseguinte A é enumerável. Se for falsa, é possível 
produzir um conjunto entre ALEPH0 e c e portanto o

número de tais funcões é não enumerável.

Um vazio de afinidades parece ser algo diferente de um vazio de 
cardinalidade. Existiria assim dois conjuntos vazios. Para um dado 
conjunto, existiria o seu numero cardinal e o conjunto das afinidades de 
seus elementos. Exemplo :


conjunto A ={joao,maria}
Afinidades associadas a A ={{ mesma idade, mesma cor, mesma altura }}

conjunto B={pedro,marta}
Afinidades associadas a B = {{ }}( sem afinidades )



Parece interessante.  Os conjuntos neste caso são definidos por propriedades 
intrínsecas

que definem seus elementos
que poderiam ser propriedades topológicas (por exemplo).

No caso do conjunto {joao, maria}
os elementos guardam entre si uma relação de equivalência topológica (mesma 
idade, mesma cor,
mesma altura).  No caso dos naturais e racionais a propriedade topológica 
seria que qualquer ponto
pode ser isolado de outro ponto por uma bola aberta (ou que poderíamos ser 
capazes de

identificar os elementos desse conjunto, "pescando-os com uma agulha")

   No caso dos reais essa propriedade topológica não é verificada: Não é 
sempre possível "pegar

um número real com uma agulha na reta", o que faz com que o conjunto
seja não enumerável.  Mas não é tão simples.  Existem outras propriedades 
(afinidades) que não estamos

identificando nos reais que dizem respeito à cardinalidade desses conjuntos?


Seria necessario agora construir um conjunto de axiomas que regulassem a 
relacao entre estas duas categorias de conjuntos : os conjuntos e suas 
afinidades.  Mas e certo que poderiamos passar a interpretar 
combinatoriamente :


BINOM(N,0)=2


Tah... mas as afinidades formam um conjunto.  O que dizer então da afinidade 
do conjunto de afinidades?





Esta mensagem toca em assuntos dificeis e importantes, mas e absolutamente 
despretensiosa



  É assim que as coisas começam.  Sempre tiramos coisas úteis, por mais 
besta e idiota que a discussão pareça.
Geralmente na cantina do instituto é que surgem as grandes idéias.   Confira 
o link:


http://ctjovem.mct.gov.br/index.php?action=/content/view&cod_objeto=14986

Abraço a todos.
Gandhi. 


=
Instruções para entrar na lista, sair da lista e usar a lista em
http://www.mat.puc-rio.br/~nicolau/olimp/obm-l.html
=


[obm-l] Re:[obm-l] RE: [obm-l] Dúvida (urgente)

2003-11-26 Thread Osvaldo
Está certo, a circunferencia tem raio f(X0).

Estou tentando desenvolvendo um metodo numerico para 
calcular as raizes de uma funcao continua de modo que 
necessite de MUITO MENOS interacoes com relacao as 
necessarias usando o famoso metodo de newton (usando 
derivacoes). Para isto pego um pto. da funcao e traco um 
circunferencia de raio f(Xo), tocando no eixo dos X. 
Encontrando as coodenadas de todas as  interseccoes de f 
com a circunferencias pego a de abssissa menor e traco 
outra circunferencia, agora com centro em (x1,f(x1)) 
tocando o eixo dos x novamente e assim sucessivamente, 
porem tenho que encontrar x1=F(xo,f(xo)), onde F é a 
funcao a ser determinada, e x1 NAO pode estar em funcao 
de f(x1).


Fiz inumeros testes para este processo, funcionou!!!, 
mais nao consigo provar analiticamente. 


Na sua resposta

> f(x1)= f(x0) + sqrt(f(x0)^2 - (x1-x0)^2) ou 
> f(x1)= f(x0) - sqrt(f(x0)^2 - (x1-x0)^2

x1 esta em funcao de f(x1), que nao é a funcao que 
queria.

 
Agradeco sua ajuda.









> Osvaldo,
> 
> Nao sei se entendi direito, me corriga se eu estiver er
rado.
> 
> Considere dois pontos P1 e P2 tais que:
> 
> P1: (X0,F(X0))  -
  Centro da Circunferencia (Why ??? Faca um desenho) 
> P2: (X1,F(X1))  -
  Ponto de intersecao de f com a circunferencia. 
> 
> Note, a circunferencia tem que ter centro P1 e o raio d
ela tem que ser f(X0)
> segundo a descricao do problema
(Faca um desenho). Alem disso, se f
> intersecta a circunferencia em P2, entao temos que ter:
> 
> |P1-P2| = Raio da circunferencia = f(X0) 
> 
> (x1-x0)^2 + (f(x1)-f(x0))^2 = [f(x0)] ^2 
> 
> [f(x1) - f(x0)]^2 = f(x0)^2 - (x1-x0)^2 
> 
> f(x1)= f(x0) + sqrt(f(x0)^2 - (x1-x0)^2) ou 
> f(x1)= f(x0) - sqrt(f(x0)^2 - (x1-x0)^2).
> 
> 
> Regards,
> 
> Leandro
> Los Angeles, CA. 
> 
> 
> -Original Message-
> From: [EMAIL PROTECTED] [mailto:owner-obm-
[EMAIL PROTECTED] On
> Behalf Of Osvaldo
> Sent: Wednesday, November 26, 2003 2:06 PM
> To: lista de discussao de matematica
> Subject: [obm-l] Dúvida (urgente)
> 
> Olá pessoal, tenho um problema que tenho tentado 
> solucionar mas tá dificil. Muitos tem me dito que é 
> impossível, mas eu insisto. 
> 
> O problema é o seguinte:
> 
> "Seja f uma função contínua em seu domínio. Sabe-
se que 
> ela passa pelo centro de uma circunferência que é 
> tangente ao eixo dos X na abscissa Xo. A função não é 
> necessariamente bijetora e seja X1 a abscissa de uma da
s 
> intersecções de f com a circunferencia em questão.
> 
> O problema é determinar f(X1) EM TERMOS DE Xo E/OU F
> (Xo)." - Paradigma de Labaki-Osvaldo
> 
> 
> 
> 
> Eu substitui X1 na equação da circunferência e dirivei-
a 
> com relação a X1 duas vezes consecutivas obtendo, assim
, 
> uma expressão para a derivada segunda em X1 da função 
> dada em termos de Xo e de f
(Xo). Daí teria que encontrar 
> as raízes desta equação diferenciavél, mais não consegu
i 
> encontrar f(0) nem f'(0), o que complica mais.
> 
> 
> 
> Atenciosamente,
> 
> Osvaldo Mello Sponquiado FEIS - UNESP
> Usuário em GNU/Linux
> Futuro Engenheiro Eletricista
> 
> 
> 
>  
> ___
___
> Acabe com aquelas janelinhas que pulam na sua tela.
> AntiPop-up UOL - É grátis!
> http://antipopup.uol.com.br/
> 
> 
> 
> ===
==
> Instruções para entrar na lista, sair da lista e usar a
 lista em
> http://www.mat.puc-rio.br/~nicolau/olimp/obm-l.html
> ===
==
> 
> ===
==
> Instruções para entrar na lista, sair da lista e usar a
 lista em
> http://www.mat.puc-rio.br/~nicolau/olimp/obm-l.html
> ===
==
> 

Atenciosamente,

Osvaldo Mello Sponquiado FEIS - UNESP
Usuário em GNU/Linux




 
__
Acabe com aquelas janelinhas que pulam na sua tela.
AntiPop-up UOL - É grátis!
http://antipopup.uol.com.br/



=
Instruções para entrar na lista, sair da lista e usar a lista em
http://www.mat.puc-rio.br/~nicolau/olimp/obm-l.html
=


[obm-l] Re: [obm-l] RE: [obm-l] Dúvida Simples!!!

2004-01-13 Thread Paulo Santa Rita
Ola Fabio e demais colegas
desta lista ... OBM-L,
Nao precisa inverter, basta usar o sinal <=> no lugar de =>, que e o que eu 
queria fazer.

From: Fábio Dias Moreira <[EMAIL PROTECTED]>
Reply-To: [EMAIL PROTECTED]
> 1) f(x)=2x-5 ( Dominio : R, Contra-Dominio : R )
>
> Suponha que x1=x2. Entao :
> 2*x1 = 2*x2   =>   2*x1 - 5 = 2*x2 - 5   =>   f(x1)=f(x2)
> Portanto : x1=x2 => f(x1)=f(x2) => funcao injetiva.
> [...]
A implicação x=y => f(x)=f(y) vale para qualquer função trivialmente.  A
implicação que prova a injetividade é f(x)=f(y) => x=y (ou, naturalmente, a
sua contrapositiva).
De qualquer forma, basta inverter a cadeia de implicações acima.

[]s,

- --
Fábio "ctg \pi" Dias Moreira
-BEGIN PGP SIGNATURE-
Version: GnuPG v1.2.3 (GNU/Linux)
iD8DBQFAAvaualOQFrvzGQoRAudzAJwKZmwKUbGWJepRhwJbXgzpRl+lhQCcDHUe
lmPqaHD0ss5v6t63HcZqlVE=
=aZJ1
-END PGP SIGNATURE-
=
Instruções para entrar na lista, sair da lista e usar a lista em
http://www.mat.puc-rio.br/~nicolau/olimp/obm-l.html
=
_
MSN Hotmail, o maior webmail do Brasil.  http://www.hotmail.com
=
Instruções para entrar na lista, sair da lista e usar a lista em
http://www.mat.puc-rio.br/~nicolau/olimp/obm-l.html
=


[obm-l] Re: [obm-l] Re: [obm-l] Dúvida persistente!!!

2004-04-10 Thread Rafael
Esta resolução está errada... :-(

Vou tentar consertar e reenvio depois.



- Original Message -
From: "Rafael" <[EMAIL PROTECTED]>
To: <[EMAIL PROTECTED]>
Sent: Saturday, April 10, 2004 9:50 PM
Subject: [obm-l] Re: [obm-l] Dúvida persistente!!!


Eduardo,

Esse exercício é facilitado se você fizer algumas construções.

Primeiramente, vamos subtrair a área de um setor de 90° e raio x da área do
quadrado ABCD:

S1 = x^2 - (Pi * x^2)/4 = x^2 * (1 - Pi/4)

Depois disso, ligue o centro da circunferência inscrita no quadrado ao ponto
médio de dois lados adjacentes do quadrado; construiremos um quadrado de
lado x/2. Da área deste quadrado subtraímos a área de um setor de 90° e raio
x/2:

S2 = (x/2)^2 - (Pi * (x/2)^2)/4 = x^2  * (1 - Pi/4) / 4

Pronto! A área que procuramos é:

S = x^2 - 2 * S1 - 2 * S2 = x^2 - 2 * (S1 - S2)
S = x^2 - 2 * (3/4 * x^2 * (1 - Pi/4))
S = x^2 - 3/2 * x^2 * (1 - Pi/4)
S = x^2 * (1 - 3/2 * (1 - Pi/4)
S = x^2 * (1 - 3/2 + 3*Pi/8)

S = x^2 * (3*Pi - 4) / 8


Abraços,

Rafael de A. Sampaio





- Original Message -
From: "Eduardo de Melo Beltrão" <[EMAIL PROTECTED]>
To: <[EMAIL PROTECTED]>
Sent: Saturday, April 10, 2004 7:02 PM
Subject: [obm-l] Dúvida persistente!!!


Olá pessoal,
Tenho uma dúvida que já perdura por anos. Gostaria de compartilhar com
vocês, e se a resposta já foi lançada na lista, gostaria apenas que
indicassem o caminho para eu poder analisar. Desde já agradeço.
Eduardo Beltrão

Num quadrado ABCD de lado x está inscrita uma circunferência L1. Os vértices
opostos A e C do quadrado são centros das circunferências L2 e L3, de raios
igual ao lado do mesmo. Determinar a área da região formada pela interseção
de L1, L2
e L3 em função de x. (Tente usar apenas conhecimentos de geometria plana).



=
Instruções para entrar na lista, sair da lista e usar a lista em
http://www.mat.puc-rio.br/~nicolau/olimp/obm-l.html
=


[obm-l] Re: [obm-l] Re: [obm-l] Dúvida persistente!!!

2004-04-12 Thread Rafael
Eu desisto...

Tentei encontrar uma solução simples, como pedia o Eduardo, mas a melhor
forma que vejo agora é calcular, por integral, a área verde e só depois
encontrar a área amarela.

Minha idéia é pôr a circunferência de centro A na origem do sistema de
coordenadas; o lado do quadrado não será mais x, e sim R; a equação da
circunferência citada será x^2 + y^2 = R^2. A circunferência inscrita no
quadrado terá equação: (x-R/2)^2 + (y+R/2)^2 = R^2/4. Os pontos de
intersecção das equações são:

( R*(5 + sqrt(7))/8 ; R*(sqrt(7) - 5)/8 )

e

( R*(5 - sqrt(7))/8 ; -R*(5 + sqrt(7))/8 )


A área S amarela será dada por:

S = Pi * R^2/4 - 2*(Integral[- sqrt(R^2 - x^2)] dx -
- Integral[- R/2 + sqrt(x*R - x^2)] dx)

O intervalo das integrais é [R*(5 - sqrt(7))/8 ; R*(5 + sqrt(7))/8].


Depois de muito trabalho algébrico (deixado para o Mathematica),
voltando de R para x, chegamos à expressãozinha anexada a esta mensagem,
por razões óbvias...

Dá para entender o porquê de a questão ser persistente...


Abraços,

Rafael de A. Sampaio





- Original Message -
From: "Rafael" <[EMAIL PROTECTED]>
To: <[EMAIL PROTECTED]>
Sent: Sunday, April 11, 2004 3:12 AM
Subject: [obm-l] Re: [obm-l] Dúvida persistente!!!


Obrigado pelo elogio à figura, Qwert.

Na verdade, o que tornou a minha solução errada foi não ter somado quatro
vezes a área vermelha, pois cada uma acabou sendo subtraída duas vezes. Pelo
que vejo, descobrindo a área vermelha, teremos a área amarela (que foi a que
pretendi calcular) e a diferença da área do círculo menor (de raio x) com
esta área amarela é, precisamente, a área verde.

Descobrir essa área vermelha é que não me parece muito fácil...






FigColor.gif
Description: Binary data


result.gif
Description: Binary data


[obm-l] RE: [obm-l] Re:[obm-l] dúvida chara!

2004-05-23 Thread Rogério Moraes de Carvalho
Olá colegas da lista,

Apesar da resolução apresentada pelo Osvaldo ter seguido um possível
raciocínio correto para resolver esta questão, a análise dele está
incompleta porque omite alguns passos muito importantes, o que pode nos
levar a encontrar soluções inválidas. Neste problema especificamente, a
resposta encontrada está correta, porém, se modificarmos o valor da
diferença de quadrados de 27 para outro valor, então a resolução dele pode
nos levar a resultados errados.

A análise que eu apresento a seguir corresponde a uma crítica de
caráter construtivo com relação à resolução apresentada pelo Osvaldo. O
objetivo desta análise não é depreciar a resolução do Osvaldo, mas sim de
mostrar que é necessário sermos rigorosos nas resoluções de problemas de
Matemática para não chegarmos a resultados incorretos. Muitas vezes podemos
encontrar uma resposta correta para uma questão resolvendo-a de maneira
errada.

Na resolução apresentada abaixo, considere que "=>" significa
"implica" e ">=" significa "maior ou igual a".


QUESTÃO ORIGINAL:

"A diferença entre os quadrados de dois números naturais é 27. UM dos
possíveis valores do quadrado da soma desses dois números:
a)529
b)625
c)729
d)841"


RESOLUÇÃO POSSÍVEL:

Sejam x e y os dois números naturais, então devemos ter:
x^2 - y^2 = 27 <=> (x + y)(x - y) = 27

Adotando a = x + y e b = x - y, teremos:
a.b = 27 (i) (Observe que o produto de a e b é positivo)
Resolvendo o sistema de equações nas variáveis x e y, podemos encontrar x e
y em função de a e b:
a + b = (x + y) + (x - y) <=> a + b = 2x <=> x = (a + b)/2 (ii)
a - b = (x + y) - (x - y) <=> a - b = 2y <=> y = (a - b)/2 (iii)

Como x e y são naturais, então x >= 0 e y >= 0. Portanto:
x + y >= 0 + 0 => a >= 0. De acordo com a igualdade (i), a não pode ser 0,
logo a > 0 (iv)
Como a.b > 0 (i) e a > 0 (iv), então b > 0 (v)
y >= 0 => -y <= 0 => y >= 0 e 0 >= -y => y >= -y => x + y >= x - y =>
a >= b (vi)
Por (v) e (vi), concluímos que: a >= b > 0 (vii)

Sendo assim, devemos encontrar a e b inteiros tais que sejam satisfeitas as
seguintes condições:
a.b = 27 (ii)
a >= b > 0 (vii)
x = (a + b)/2 (ii) seja um número natural.
y = (a - b)/2 (iii) seja um número natural.

Analisando os divisores de 27, podemos concluir que existem apenas dois
pares de valores de a e b que satisfazem as condições (ii) e (vii):
(a = 27 e b = 1) ou (a = 9 e b = 3)

Para a = 27 e b = 1:
x = (27 + 1)/2 = 14 é um número natural.
y = (27 - 1)/2 = 13 é um número natural.
Portanto, x = 14 e y = 13 é uma solução possível.

Para a = 9 e b = 3:
x = (9 + 3)/2 = 6 é um número natural.
y = (9 - 3)/2 = 3 é um número natural.
Portanto, x = 6 e y = 3 é uma solução possível.

Possíveis valores para (x + y)^2:
(x + y)^2 = (14 + 13)^2 = 27^2 = 729
(x + y)^2 = (6 + 3)^2 = 9^2 = 81

Resposta: Alternativa c


Observação: Pode parecer que os passos apresentados para deduzir as
condições são desnecessários, mas são eles que garantem a validade das
soluções encontradas.


EXPLICAÇÃO DO MOTIVO DA RESOLUÇÃO APRESENTADA PELO OSVALDO SER INCOMPLETA:

Na resolução são apresentados 4 valores possíveis para a e b (a,b):
{(1,27),(3,9),(9,3),(27,1)}. Porém, (1,27) e (3,9) não satisfazem a condição
(vii): a >= b > 0. Portanto, somente os pares (9,3) e (27,1) correspondem a
possíveis valores para a e b, restando apenas verificar se eles produzem
valores naturais para x e y. Logo, na lista de valores apresentados para
(x+y)^2 = a^2, {1, 9, 81, 729}, não poderia aparecer os valores 1 = 1^2 e
nem 9 = 3^2. Além disto, não há garantia de que 81 = 9^2 e 729 = 27^2
correspondem a valores de a e b válidos, pois os valores de x e y não são
calculados para verificar se eles são naturais, como foi descrito no
enunciado do problema. Portanto, os valores de a e b encontrados poderiam
não ser válidos. Neste problema específico, os valores de a e b encontrados
são válidos, logo a resposta encontrada está correta. A seguir, eu apresento
uma variação deste problema que mostra de maneira concreta que a resolução
apresentada pelo Osvaldo pode apresentar resultados errados. Para se ter uma
idéia apenas 1 resultado, dos 6 encontrados, é correto!



QUESTÃO MODIFICADA:

"A diferença entre os quadrados de dois números naturais é 68. UM dos
possíveis valores do quadrado da soma desses dois números:
a)16
b)289
c)1156
d)4624"


RESOLUÇÃO DO OSVALDO ALTERADA PARA A VERSÃO MODIFICADA DA QUESTÃO:

sejam x e y tais numeros, dai temos que
x^2-y^2=68

(x+y)(x-y)=68


a=x+y
b=x-y

Possiveis valores para a e b (x,y):

{(1,68),(2,34),(4,17),(17,4),(34,2),(68,1)}

Assim (x+y)^2=a^2

Temos então que todos os valores de (x+y)^2 pertencem a
{1, 4, 16, 289, 1156, 4624)

Logo quatro dos valores possiveis são 16, 289, 1156 e 4624
resposta a, b, c, d



RESOLUÇÃO CORRETA POSSÍVEL PARA A QUESTÃO MODIFICADA:

Sejam x e y os dois números naturais, então devemos ter:
x^2 - y^2 = 68 <=> (x + y)(x - y) = 68

Adotando a = x + y e b = x - y, teremos:
a.b = 68 (i) (Observe que o produto de a e b é positi

[obm-l] Re: [obm-l] Re: [obm-l] Dúvida com questão

2008-09-16 Thread João Luís
Sim, é verdade. Ficou incompleto mesmo.

O que acontece é que eu quis enfatizar que, independentemente da incompletude 
do enunciado, a bicondicional dada será falsa. E, com isso, acabei me 
esquecendo do sinal do termo quadrático.

Obrigado pela observação, Bouskela.
  - Original Message - 
  From: Bouskela 
  To: obm-l@mat.puc-rio.br 
  Sent: Tuesday, September 16, 2008 11:46 AM
  Subject: Re: [obm-l] Re: [obm-l] Dúvida com questão


  João Luís:

  Sua solução está correta! Entretanto repare que ela (sua solução) está 
correta apenas porque "x^2+x+1" é positivo (maior do que "0") para qualquer que 
seja "x" real. É, então, necessário, no âmbito da sua solução, demonstrar isto:

  x^2+x+1 > 0 para qualquer que seja "x" real.

  Sds.,
  AB 


  2008/9/16 João Luís <[EMAIL PROTECTED]>

Bom, faltou um símbolo de desigualdade no primeiro membro (antecedente) 
dessa bicondicional: [(x^2+x+1)/(x-2)] 3. Mas de qualquer forma, a afirmativa é 
falsa, já que o sinal da desigualdade vai mudar de sentido conforme o sinal do 
termo de primeiro grau (x-2): se (x-2) < 0, a segunda desigualdade será o 
contrário da primeira; se (x-2) > 0, será igual à primeira

Um abraço,

João Luís.
  - Original Message - 
  From: Robÿe9rio Alves 
  To: obm-l@mat.puc-rio.br 
  Sent: Tuesday, September 16, 2008 10:05 AM
  Subject: [obm-l] Dúvida com questão


A afirmação: para todo x real, x diferente de 2,[( x^2+x+1) / 
(x - 2 )]  3  <=> x^2+x+1 > 3.(x - 2 ) é verdaeira ou falsa ? Justifique.

Como resolver ???
   


--
  Novos endereços, o Yahoo! que você conhece. Crie um email novo com a sua 
cara @ymail.com ou @rocketmail.com.



  -- 
  Saudações,
  AB
  [EMAIL PROTECTED]
  [EMAIL PROTECTED]


[obm-l] Re: [obm-l] Re: [obm-l] Dúvida de vestibular

2003-02-27 Thread Nicolau C. Saldanha
Eu [ainda] não sei resolver o problema do Okakome mas...

On Wed, Feb 26, 2003 at 04:39:33PM -0300, Domingos Jr. wrote:
> > Oi Pessoal,
> >   Estava estudando análise combinatória por uma
> > apostila de um curso pré-vestibular, e encontrei o
> > seguinte problema, que achei interessante, mas minha
> > solução foi muito longa, e não sei se está certa,
> > porque tinha muitos casos. Se estivesse num
> > vestibular, o que faria?
> >   Num país, as estradas ligam duas cidades e são de
> > mão única (pode haver mais de uma estrada entre duas
> > cidades). O número de estradas que partem de cada
> > cidade é igual ao número de estradas que chegam nessa
> > cidade. Um mapa da cidade C é um conjunto de rotas
> > que: 1) levam C a cada uma das outras cidades do país,
> > sem passar por uma cidade mais de uma vez. 2) Se uma
> > rota parte de C a D passando por E, então a rota que
> > vai de C a E coincide com o começo da rota de C a D.
> > Prove que o número de mapas da cidade C é igual ao
> > número de mapas de qualquer outra cidade.
> 
> Acho que esse enunciado não está completo:
> - se existe uma cidade com nenhuma estrada partindo ou chegando possui um
> número de rotas 0 e não vai ser igual as demais, até aí é um caso idiota que
> pode ser excluído do problema.

Neste caso não existe nenhum mapa pois é impossível ligar C a X,
a cidade sem estradas. O problema fica correto pois o número
de mapas é sempre 0.

> - se existem conjuntos de cidades "disjuntos" ou seja cidades de um conjunto
> A não possuem rota nenhuma para as cidades do conjunto B e vice-versa:
> 
> C1 <---> C2 ( 1 estrada de C1 -> C2 e outra de C2 -> C1  )
> C3 <==> C4 ( 2 estradas de C3 -> C4 e duas de C4 -> C3 )
> 
> neste caso temos que o número de rotas de N(C1) = N(C2) = 1, mas N(C3) =
> N(C4) = 2.

Também neste caso o número de mapas é sempre 0.

> além disso, há um trecho que eu considero confuso:
> 
> "2) Se uma rota parte de C a D passando por E, então a rota que vai de C a E
> coincide com o começo da rota de C a D"
> nessa situação parece que a rota de C a E deve ser única, mas podem haver
> outras rotas de C até E sem que uma cidade seja visitada mais de uma vez...
> mesmo que sempre fossem escolhidos os caminhos que passem por menos cidades
> isso poderia ocorrer já que é permitido haver mais de uma estrada ligando
> duas cidades.

Acho que ajuda considerar o caso em que todas as estradas são de mão dupla:
neste caso um mapa nada mais é do que uma árvore maximal e o problema
fica trivial.

[]s, N.
=
Instruções para entrar na lista, sair da lista e usar a lista em
http://www.mat.puc-rio.br/~nicolau/olimp/obm-l.html
O administrador desta lista é <[EMAIL PROTECTED]>
=


[obm-l] Re: [obm-l] Re: [obm-l] Dúvida sobre limites

2009-10-28 Thread Paulo Barclay Ribeiro
 Luiz.
Creio que o erro que cometi foi ter dividido o calculo desse limite em duas 
partes( produto dos limites 1+n/n+1 com sen(npi/2), nesta parte fiz: 
-1 escreveu:


De: Luiz Paulo 
Assunto: [obm-l] Re: [obm-l] Dúvida sobre limites
Para: obm-l@mat.puc-rio.br
Data: Quarta-feira, 28 de Outubro de 2009, 14:14







Podemos ver Tn da seguinte forma:
T(n)=1+1/[1+1/n]*sen(npi/2).
Tomando n=2k (k inteiro) vemos que daí teremos sen(kpi) que fica sendo zero.
Tomando n=2k+1(k inteiro) teremos sen[(2k+1)pi/2] que oscila entre -1 ou 1 
dependendo do k.
Daí tomando k tendendo ao infinito vemos que o termo em sen oscila entre esses 
valores.
Portanto T(n) diverge.

--- Em ter, 27/10/09, Paulo Barclay Ribeiro  
escreveu:


De: Paulo Barclay Ribeiro 
Assunto: [obm-l] Dúvida sobre limites
Para: obm-l@mat.puc-rio.br
Data: Terça-feira, 27 de Outubro de 2009, 18:35







pessoal , peço uma ajuda para esclarecer a seguinte questão;
 
A sucessão: T_n = 1+(n/n+1)*sin(npi/2) com n sendo um número natural é 
convergente ?
 
A achei que a sucessão acima é convergente , mas conversando,por alto, com um 
colega ele levantou a hipótese dessa sucessão ser divergente. Aí fiquei em 
dúvida.
Gostaria de uma orientação de vocês.
 
Obrigado, mais uma vez
 
Paulo


Veja quais são os assuntos do momento no Yahoo! + Buscados: Top 10 - 
Celebridades - Música - Esportes


Veja quais são os assuntos do momento no Yahoo! + Buscados: Top 10 - 
Celebridades - Música - Esportes


  

Veja quais são os assuntos do momento no Yahoo! +Buscados
http://br.maisbuscados.yahoo.com

[obm-l] Re: [obm-l] Re: [obm-l] dúvida sobre séries

2011-06-06 Thread Rodrigo Renji
Olá!

Uma outra maneira  ( além da que os colegas enviaram antes), para
mostrar que a série  não converge, tem um critério de convergência que
acho legal, Critério de condensação de Cauchy:


 Se x_k é uma sequência decrescente de termos positivos ( como é o caso de 1/k )


então a série [ SOMA de x_k]  converge , se e somente se , a série [
SOMA de 2^k  x_(2^k) ]  converge.

Aplicando isso para a série do email

temos com a_k= 1/k


[ SOMA de 2^k  x_(2^k) ]  =  [ SOMA de 2^k  ,  1/ (2^k)   ] =  [ SOMA 1 ]

que diverge, pois somando de 1 até n resulta em "n", com n indo pro
infinito , diverge  : ) Pode não ajudar muito, mas acho esse critério
legal

abraço

=
Instruções para entrar na lista, sair da lista e usar a lista em
http://www.mat.puc-rio.br/~obmlistas/obm-l.html
=


[obm-l] Re: [obm-l] RE: [obm-l] Dúvida em gabarito

2011-11-27 Thread JOSE AIRTON CARNEIRO
Sua resposta está correta. A área da figura em questão é formada por um
triângulo equilátero + 2 segmentos circulares de arcos 60°. = a^2(4pi -
3raiz(3))/12.
Airton
Em 26 de novembro de 2011 15:19, João Maldonado  escreveu:

>
> Deixa  eu ver se entendi
>
>
> Dado um  quadrado,  construa dois circulos  de centro em  vértices
> adjacentes e raios iguais a a (que é o lado do quadrado).  Calcule a região
> em comum entre os círculos e o quadrado.
>
> Faça assim divida o quadrado no meio,  vai originar duas figuras
> semelhantes.  A área de cada uma pode ser calculada:
>  (Área do setor-Área do triângulo)/2  *  2
> (pois  são duas figuras)
>
> Pi.a²/3 -  a²sqrt(3)/4 = a²(4Pi-3sqrt(3))/12
>
> João
>
>  --
> From: marconeborge...@hotmail.com
> To: obm-l@mat.puc-rio.br
> Subject: [obm-l] Dúvida em gabarito
> Date: Sat, 26 Nov 2011 11:28:50 +
>
>
> seja um quadrado ABCD de lado a.Os arcos AC e BD formam com 3 dos lados do
> quadrado dois setores ciculares(um quarto do círculo de raio a
> cada).Deteminar a área comum aos setores circulares.
> A resposta que o livro dá é a^2(9raiz(3) - 4pi)/12 e a que eu achei foi
> a^2(4pi - 3raiz(3))/12
> Dsculpem a falta da figura.Agradeço a quem puder ajudar.
>


[obm-l] Re: [obm-l] Re: [obm-l] Dúvida de Lógica

2012-02-08 Thread Ralph Teixeira
H nao sei nao. Vou usar "C" para "estah contido" e "E" para
"pertence a".

Concordo que toda inclusao de conjuntos pode ser pensada como uma
implicacao (bom, com um quantificador "para todo"). Afinal:

A C B
eh o mesmo que dizer
para todo x, xEA => xEB

Por isso, concordo que a Teoria dos Conjuntos e a Logica Matematica estao
profundamente emaranhadas Mas nem toda sentenca logica tem um conjunto
-- tem que ter uma variavel na sentenca logica!

Assim, se voce considera a proposicao "x^3+x+1>0", a ela estah associada o
conjunto A dos valores de x que satisfazem aquela proposicao. Mas aquela
proposicao eh ABERTA, isto eh, tem uma variavel x SOLTA nela, entao faz
sentido falar do conjunto dos x que a satisfaz. (Variavel SOLTA eh uma que
pode ser substituida por valores; uma proposicao ABERTA nunca eh nem V nem
F -- mas se voce substituir valores em todas as variaveis soltas de uma
proposicoa aberta, ai ela passa a ser V ou F, dependendo dos valores)

Agora, na proposicao "2>3", nao tem x. Entao nao faz sentido pensar no
conjunto A que voce cita, pelo menos nao diretamente. Voce poderia forcar a
barra e dizer que a proposicao eh r(x): 2>3, e entao eu concordo contigo,
mas tem que dizer algo explicito sobre a variavel que NAO aparece na
proposicao.

(Isto eh mais ou menos equivalente aa diferenca entre escrever f=5 ou
f(x)=5; se voce escreve do primeiro jeito, eu vou pensar que f eh um
NUMERO, igual a 5; do segundo jeito, eu penso que f eh a FUNCAO CONSTANTE e
igual a 5, cuja variavel x calhou de nao aparecer na expressao de f. Sao
objetos distintos.)

Idem para a segunda proposicao: "Todo brasileiro eh desonesto", nao tem
variavel solta, eh V ou F e acabou. Sim, ela eh equivalente a "Para todo y,
se y eh brasileiro entao y eh desonesto", mas note que nesta sentenca y nao
eh uma variavel SOLTA -- o "para todo y" da frente te proibe de substituir
valores em y. A proposicao "se y eh brasileiro entao y eh desonesto" eh
ABERTA e permite susbtituir valores para y, mas ela nao tem o "para todo".
Uma grande parte da confusao eh que a gente costuma pensar numa implicacao
e incluir o "para todo" nela sem escrever...

To sendo confuso e chato, eu sei mas eh que eu acho que o tempo que se
gasta em teoria dos conjuntos no ensino fundamental e medio podia ser MUITO
melhor aproveitado indo direto na logica -- e eu nao sei explixar tudo isso
em poucas linhas.

Abraco,
  Ralph

2012/2/7 Francisco Barreto 

> Saudações a todos!
> Seja A o conjunto dos objetos que satisfazem a propriedade r  de que 2 > 3.
> Seja B o conjunto dos objetos que satisfazem a propriedade s de que "Todo
> brasileiro é desonesto"
>
> Sabemos que A é o conjunto vazio. O conjunto vazio está contido em
> qualquer conjunto, incluindo B. Portanto os elementos de A (se você
> encontrar) devem satisfazer a  propriedade s de que todo brasileiro é
> desonesto.
>
>
>
> 2012/2/7 Pedro Chaves 
>
>>
>> Caros Colegas,
>>
>> Pode-se dizer que "2 > 3 => Todo brasileiro é desonesto"?   (O símbolo =>
>> indica implicação lógica.)
>>
>>
>> Sei que é verdadeira a proposição condicional "Se 2 > 3, então todo
>> brasileiro é desonesto", mas me parece que não existe implicação lógica.
>>
>> Desde já, muito obrigado.
>> Um abraço do Pedro Chaves!
>> =
>> Instruções para entrar na lista, sair da lista e usar a lista em
>> http://www.mat.puc-rio.br/~obmlistas/obm-l.html
>> =
>>
>
>
>
> --
> Sinceramente,
> Francisco Costa D. Barreto
>
>


[obm-l] RE: [obm-l] Re: [obm-l] Dúvida(questão simples)

2014-02-22 Thread marcone augusto araújo borges
Eu pensei assim também.Obrigado!

From: ilhadepaqu...@bol.com.br
To: obm-l@mat.puc-rio.br
Subject: [obm-l] Re: [obm-l] Dúvida(questão simples)
Date: Sat, 22 Feb 2014 00:28:27 -0300








quando ele anda no sentido horário ele anda 380 
graus em 40 minutos porque o ponteiro das horas em 40 minutos andou 20 
graus
 
Assim sua velocidade é de 9,5 graus por 
minuto
 
o ponteiro das horas  anda a 0,5 grau por 
minuto
 
logo falta calcular  quando vão se encontrando 
e a velociadade conjunta é de 10 graus por minuto
 
para percorrer 180 graus eles vão demorar 18 
minutos 
 
40 +18 = 58 minutos
 
abraços 
Hermann
 
 
 
 

  - Original Message - 
  From: 
  marcone augusto araújo borges 
  
  To: obm-l@mat.puc-rio.br 
  Sent: Friday, February 21, 2014 11:33 
  PM
  Subject: [obm-l] Dúvida(questão 
  simples)
  

  Exatamente no  momento em que o ponteiro das horas passa 
  pelo 12, uma formiga começa a andar ao longo
da borda de um relógio no 
  sentido anti-horário,partindo do 6,com velocidade constante.Quando a formiga 
  en-
contra o ponteiro das horas,ela muda de direção e continua a andar na 
  mesma velocidade no sentido horário
Quarenta minutos após o primeiro 
  encontro,a formiga se encontra pela segunda vez com o ponteiro das horas 
e 
  morre.Quanto tempo a formiga andou?

O gabarito dá 54 minutos mas eu só 
  tô achando 58.
Agradeço por esclarecimento.


-- 
Esta 
  mensagem foi verificada pelo sistema de antivírus e 
acredita-se estar 
  livre de perigo. 
--

Esta mensagem foi verificada pelo sistema de antivírus e 

 acredita-se estar livre de perigo.   
-- 
Esta mensagem foi verificada pelo sistema de antivírus e
 acredita-se estar livre de perigo.



[obm-l] RE: [obm-l] Re: [obm-l] Dúvida(questão simples)

2014-02-22 Thread marcone augusto araújo borges
Obrigado!

Date: Sat, 22 Feb 2014 00:31:24 -0300
Subject: [obm-l] Re: [obm-l] Dúvida(questão simples)
From: tarsise...@gmail.com
To: obm-l@mat.puc-rio.br

Erramos juntos. Pq tb achei 58.

--

Esta mensagem foi verificada pelo sistema de antivírus e 

 acredita-se estar livre de perigo.   
-- 
Esta mensagem foi verificada pelo sistema de antivírus e
 acredita-se estar livre de perigo.



Re: [obm-l] RE: [obm-l] Re: [obm-l] Dúvida Raiz

2005-12-31 Thread ricardo.bioni
O valor de x^(1/n) onde n é um número par é apenas o número positivo y tal que y^n = xSe for procurado o número negativo y tal que y^n = x então o valor procurado é y = -x^(1/n)


RE: [obm-l] Re: [obm-l] Re: [obm-l] Dúvida persistente!!!

2004-04-13 Thread Qwert Smith
Sai na geometria mas da umas contas chatas.

na primeira figura:
---
(area em amarelo)  = (area do circulo menor) - 2*(area em verde)
(area em vermelho) = (area do quadrado) -
1/4*{(area do circulo maior) + [(area do quadrado)-(area circulo menor)]} -
(area em verde)
---
na segunda figura:
---
(area em laranja) = (setor circular PBQ) - 2*(triangulo PBO - area em azul)
(area em verde) = (setor circular POQ) - (area em laranja)
--
Como todos os lados do triangulo sao conhecidos (em funcao do lado do 
quadrado).  Agulos e areas sao questao de braco.

A descricao da figura 2 vc encontra no link

http://www.mat.puc-rio.br/~nicolau/olimp/obm-l.200310/msg00574.html

em uma menssagem do grande Claudio BUffara que ainda teve paciencia 
montruosa de me explicar em off o problema.

Em meu email anterior eu tinha feito confusao e atribuido a mensagem do link 
acima a outro fera, o Paulo Santa Rita que mandou uma mensagem sobre "lua 
algebrica", que tb vale a pena conferir.
E' muito genio pra keep track.

Valeu Super Buffara!

[]s,
Auggy








From: "Rafael" <[EMAIL PROTECTED]>
Reply-To: [EMAIL PROTECTED]
To: "OBM-L" <[EMAIL PROTECTED]>
Subject: [obm-l] Re: [obm-l] Re: [obm-l] Dúvida persistente!!!
Date: Tue, 13 Apr 2004 03:20:58 -0300
Eu desisto...

Tentei encontrar uma solução simples, como pedia o Eduardo, mas a melhor
forma que vejo agora é calcular, por integral, a área verde e só depois
encontrar a área amarela.
Minha idéia é pôr a circunferência de centro A na origem do sistema de
coordenadas; o lado do quadrado não será mais x, e sim R; a equação da
circunferência citada será x^2 + y^2 = R^2. A circunferência inscrita no
quadrado terá equação: (x-R/2)^2 + (y+R/2)^2 = R^2/4. Os pontos de
intersecção das equações são:
( R*(5 + sqrt(7))/8 ; R*(sqrt(7) - 5)/8 )

e

( R*(5 - sqrt(7))/8 ; -R*(5 + sqrt(7))/8 )

A área S amarela será dada por:

S = Pi * R^2/4 - 2*(Integral[- sqrt(R^2 - x^2)] dx -
- Integral[- R/2 + sqrt(x*R - x^2)] dx)
O intervalo das integrais é [R*(5 - sqrt(7))/8 ; R*(5 + sqrt(7))/8].

Depois de muito trabalho algébrico (deixado para o Mathematica),
voltando de R para x, chegamos à expressãozinha anexada a esta mensagem,
por razões óbvias...
Dá para entender o porquê de a questão ser persistente...

Abraços,

Rafael de A. Sampaio





- Original Message -
From: "Rafael" <[EMAIL PROTECTED]>
To: <[EMAIL PROTECTED]>
Sent: Sunday, April 11, 2004 3:12 AM
Subject: [obm-l] Re: [obm-l] Dúvida persistente!!!
Obrigado pelo elogio à figura, Qwert.

Na verdade, o que tornou a minha solução errada foi não ter somado quatro
vezes a área vermelha, pois cada uma acabou sendo subtraída duas vezes. 
Pelo
que vejo, descobrindo a área vermelha, teremos a área amarela (que foi a 
que
pretendi calcular) e a diferença da área do círculo menor (de raio x) com
esta área amarela é, precisamente, a área verde.

Descobrir essa área vermelha é que não me parece muito fácil...



<< FigColor.gif >>
<< result.gif >>
_
FREE pop-up blocking with the new MSN Toolbar – get it now! 
http://toolbar.msn.com/go/onm00200415ave/direct/01/
<><>

[obm-l] Re: [obm-l] Re: [obm-l] dúvida PARA FAELCCMM

2004-05-08 Thread Fernando Romualdo Braga



Se considerarmos apenas os cortes ADICIONAIS a 
partir do primeiro, a alternativa correta é a C- 6 e 25º, o total de cortes para 
se obter os 14 pedaços são 7.
 
- Original Message - 
From: TSD 
To: [EMAIL PROTECTED] 
Sent: Saturday, May 08, 2004 8:43 PM
Subject: [obm-l] Re: [obm-l] dúvida PARA FAELCCMM

a questão pergunta quantos cortes 
ADICIONAIS

  - Original Message - 
  From: 
  [EMAIL PROTECTED] 
  
  To: [EMAIL PROTECTED] 
  Sent: Saturday, May 08, 2004 7:01 
PM
  Subject: Re: [obm-l] dúvida
  Para 1 corte > 2 pedaços 2 
  cortes > 4 pedaços 3 cortes > 6 pedaços n cortes 
  > 2n pedaços Logo para 14 pedaços serao necessarios 7 cortes. 
  O angulo da fatia = 360/ 14 ~ 26 Alternativa B 
  Em uma mensagem de 8/5/2004 18:49:43 Hora padrão leste da 
  Am. Sul, [EMAIL PROTECTED] escreveu: 
  
  Para 1 corte > 2 pedaço 2 corte > 
4 pedaços 3 corte > 6 pedaços n corte > 2n pedaços 
Logo para 14 pedaços serao necessarios 7 cortes. O angulo da 
fatia = 360/ 14 ~ 26 Alternativa B Em uma mensagem 
de 8/5/2004 18:38:48 Hora padrão leste da Am. Sul, [EMAIL PROTECTED] 
escreveu: 
17 - Uma pizza de formato circular será cortada em 
  fatias pormeio de cortes quesão segmentos de reta que passam pelo"centro" 
  da pizza e medem um diâmetro. A figura a seguir mostra que, com um corte, 
  a pizza fica dividida em duas fatias iguais.( é uma circunferencia com um 
  traço diametro) Para se obter quatorze fatias iguais, a partir 
  do primeiro corte,serão necessários cortes adicionais na 
  pizza. Assinale o item que indica o número de 
  cortes adicionais que terão de ser feitos e o ângulo aproximado que cada 
  fatia, imaginada como um setor circular, terá: (A) 8 e 
  30o;(B) 7 e 26o ;(C) 6 e 25o;(D) 7 e 30o; (E) 8 e 26o. 


[obm-l] Re: [obm-l] Re: [obm-l] Dúvida "aparentemente" simples

2015-08-11 Thread Pedro José
Boa tarde!

Pela definição, simétrico ou oposto de um elemento a de um anel é o
elemento do anel que operado com a por + resulte 0.
Portanto o simétrico ou oposto de zero é zero.

Saudações,
PJMS.

Em 11 de agosto de 2015 12:02, Ralph Teixeira  escreveu:

> Acho que a convencao de quase todos eh que -0=0. Nao vejo problema de que
> o oposto de algo seja ele mesmo.
>
> 2015-08-11 11:27 GMT-03:00 Vanderlei Nemitz :
>
>> Trabalho com edição de material didático e outro dia, em conversa com um
>> autor de material do ensino médio, surgiu uma questão sobre a qual gostaria
>> de saber a opinião de vocês.
>>
>> O oposto de zero é zero ou não faz sentido falar dele?
>>
>> Por um lado, a solução da equação x = -x é x = 0, ou seja, poderíamos
>> dizer que -0 = 0 (o oposto de zero é zero).
>>
>> Por outro, trata-se de um único número (ele e seu oposto) e assim não faz
>> sentido falar dele.
>>
>> A pergunta foi motivada por outra:
>>
>> Para quais valores de x temos que sqrt(x^2) = -x?
>>
>> A resposta é x < 0 ou x<= 0?
>>
>>
>> Obrigado!
>>
>> Vanderlei
>>
>> --
>> Esta mensagem foi verificada pelo sistema de antivírus e
>> acredita-se estar livre de perigo.
>
>
>
> --
> Esta mensagem foi verificada pelo sistema de antivírus e
> acredita-se estar livre de perigo.
>

-- 
Esta mensagem foi verificada pelo sistema de antiv�rus e
 acredita-se estar livre de perigo.



[obm-l] Re: [obm-l] Re: [obm-l] Dúvida em Logaritmos

2016-06-07 Thread Daniel Rocha
Muito Obrigado (mais uma vez), Carlos !!!

Em 6 de junho de 2016 22:02, Carlos Gomes  escreveu:

> log[(sqrt 2)^(x-2)] = x ==>
> (x-2)log(sqrt 2) = x ==>
> x=2log(sqrt2)/(log(sqrt2)-1).
>
> Cgomes.
>
> Em 6 de junho de 2016 19:23, Daniel Rocha 
> escreveu:
>
>> Alguém poderia, por favor, solucionar o problema abaixo:
>>
>> Ache a solução real da equação:
>> log[(sqrt 2)^(x-2)] = x
>>
>> --
>> Esta mensagem foi verificada pelo sistema de antivírus e
>> acredita-se estar livre de perigo.
>
>
>
> --
> Esta mensagem foi verificada pelo sistema de antivírus e
> acredita-se estar livre de perigo.

-- 
Esta mensagem foi verificada pelo sistema de antiv�rus e
 acredita-se estar livre de perigo.



[obm-l] Re: [obm-l] Re: [obm-l] Dúvida num Enunciado

2018-04-25 Thread Luiz Antonio Rodrigues
Olá, Pedro!
Boa noite!
O resultado é esse mesmo.
Agora eu entendi o que o problema pede.
Muito obrigado!
Um abraço!
Luiz

On Wed, Apr 25, 2018, 8:29 PM Pedro José  wrote:

> Boa tarde!
> Realmente o enunciado está mal feito.
>
> Se |x+3| < r, não pode ser para todo o Real. Na verdade é x pertence a |R.
>
> x^2 -10x + 9 >0  ==> x pertence a A = (-oo, 1) U (9,oo)
>
> então temos que escolher r de modo que quando resolvamos |x + 3| < r,
> tenha x num subconjunto de A
>
> x < -3 ==> x+3 < 0 ==> -x -3 < r ==> r > x+3 Se r > 4 vai ter 1=< x =<9
> atendendo |x +3| <4 + delta. Portanto x <4
> então |x+3| < 4, conferindo
> x > -3 ==> x+3 <4  ==> x<1, atende.
> se x<-3 atende por hipótese. Mas se quiser conferir. -x - 3 < 4 : -x < 7:
> x >7, mas x <-3, não tem solução.
>
> x>=- 3 ==> x+3>=0 ==> x+3 < r. Se r >=4, existirá solução em [1,9].
>
> Portanto r pertence a (0,4)
>
> creio que seja isso.
>
> Saudações,
> PJMS.
>
>
>
> Em 25 de abril de 2018 16:41, Luiz Antonio Rodrigues <
> rodrigue...@gmail.com> escreveu:
>
>> Olá, pessoal!
>> Boa tarde!
>> Estou tentando fazer o exercício abaixo, mas o problema é que eu não
>> entendi o enunciado...
>>
>> Determine para quais valores de r (r>0) a implicação é verdadeira:
>> |x+3| x^2 - 10x + 9 > 0 para todo x real.
>>
>> Agradeço qualquer ajuda!
>> Um abraço!
>> Luiz
>>
>> --
>> Esta mensagem foi verificada pelo sistema de antivírus e
>> acredita-se estar livre de perigo.
>
>
>
> --
> Esta mensagem foi verificada pelo sistema de antivírus e
> acredita-se estar livre de perigo.

-- 
Esta mensagem foi verificada pelo sistema de antiv�rus e
 acredita-se estar livre de perigo.



[obm-l] Re: [obm-l] Re: [obm-l] Dúvida num Enunciado

2018-04-25 Thread Luiz Antonio Rodrigues
Olá, Claudio!
Boa noite!
Eu não havia percebido que o consequente é falso...
Preciso ficar mais atento!
Muito obrigado pela ajuda!
Um abraço!
Luiz

On Wed, Apr 25, 2018, 8:49 PM Claudio Buffara 
wrote:

> O consequente (x^2 - 10x + 9 > 0 para todo x real) é falso  (tome qualquer
> x no intervalo [1,9]).
>
> Logo, para a implicação ser verdadeira, o antecedente ( |x+3| < r ) deve
> ser falso, o que ocorre se e somente se r < 0.
>
> É mais ou menos a mesma coisa que (se 1 < 0, então 3+5 = 7), que é uma
> sentença verdadeira (Falso -> Falso é Verdadeiro).
>
> []s,
> Claudio.
>
> 2018-04-25 16:41 GMT-03:00 Luiz Antonio Rodrigues :
>
>> Olá, pessoal!
>> Boa tarde!
>> Estou tentando fazer o exercício abaixo, mas o problema é que eu não
>> entendi o enunciado...
>>
>> Determine para quais valores de r (r>0) a implicação é verdadeira:
>> |x+3| x^2 - 10x + 9 > 0 para todo x real.
>>
>> Agradeço qualquer ajuda!
>> Um abraço!
>> Luiz
>>
>> --
>> Esta mensagem foi verificada pelo sistema de antivírus e
>> acredita-se estar livre de perigo.
>
>
>
> --
> Esta mensagem foi verificada pelo sistema de antivírus e
> acredita-se estar livre de perigo.

-- 
Esta mensagem foi verificada pelo sistema de antiv�rus e
 acredita-se estar livre de perigo.



[obm-l] Re: [obm-l] Re: [obm-l] Dúvida num Enunciado

2018-04-25 Thread Bernardo Freitas Paulo da Costa
2018-04-25 20:41 GMT-03:00 Claudio Buffara :
> O consequente (x^2 - 10x + 9 > 0 para todo x real) é falso  (tome qualquer x
> no intervalo [1,9]).
>
> Logo, para a implicação ser verdadeira, o antecedente ( |x+3| < r ) deve ser
> falso, o que ocorre se e somente se r < 0.
>
> É mais ou menos a mesma coisa que (se 1 < 0, então 3+5 = 7), que é uma
> sentença verdadeira (Falso -> Falso é Verdadeiro).

Acho que há duas coisas.  Uma é a interpretação do enunciado.  Alguns
(eu me incluo, e o Pedro José também) vão ler como:

"Determine r > 0 tal que (para todo x real, |x+3| < r => x^2 - 10x + 9 > 0)."

A minha razão principal é porque o x aparece do lado esquerdo da
implicação, e portanto eu fico com a sensação que ele deveria também
estar quantificado.  Mas não é obrigatório.

Ao ler como você fez, a frase fica

"Determine r > 0 tal que [ |x+3| < r => (x^2 - 10x + 9 > 0 para todo x real) ]."

Nesta interpretação (que está mais próxima do texto original...), a
frase entre colchetes tem uma variável livre: o "x", que não está
quantificado.  Para enfatizar, como você mesmo separou o consequente,
ela não mudaria de valor se fosse

"Determine r > 0 tal que [ |x+3| < r => (A^2 - 10A + 9 > 0 para todo A real) ]."

Que continua com o "problema" de ter um "x" livre.  Daí, a proposição
entre colchetes tem um valor (verdadeiro/falso) que depende de x.
Assim, a frase completa "Determine r ..." também depende do valor de
x.  O problema fica bem diferente.  Primeiro, podemos simplificar o
enunciado para

"Determine r > 0 tal que [ |x+3| < r => FALSO ]."

Depois, pegando carona na sua solução, temos que ter o antecedente
falso, para que a afirmação entre colchetes seja verdadeira.  Ou seja:

"Determine r > 0 tal que [ |x+3| < r é FALSO ]."

Re-escrevendo, fica "Determine r > 0 tal que [ |x+3| >= r ].", o que
dá a solução: r <= |x+3|.  Repare que a solução está em função de x,
como esperado, já que o enunciado original também tinha um x livre.
Acho esta interpretação pouco plausível para um exercício, mas acho o
exercício de resolvê-la interessante ;-)

Abraços,
-- 
Bernardo Freitas Paulo da Costa

-- 
Esta mensagem foi verificada pelo sistema de antiv�rus e
 acredita-se estar livre de perigo.


=
Instru��es para entrar na lista, sair da lista e usar a lista em
http://www.mat.puc-rio.br/~obmlistas/obm-l.html
=


[obm-l] Re: [obm-l] Re: [obm-l] Dúvida num Enunciado

2018-04-25 Thread Bernardo Freitas Paulo da Costa
2018-04-25 20:20 GMT-03:00 Pedro José :
> Boa tarde!
> Realmente o enunciado está mal feito.
>
> Se |x+3| < r, não pode ser para todo o Real. Na verdade é x pertence a |R.
>
> x^2 -10x + 9 >0  ==> x pertence a A = (-oo, 1) U (9,oo)
>
> então temos que escolher r de modo que quando resolvamos |x + 3| < r, tenha
> x num subconjunto de A
>
> x < -3 ==> x+3 < 0 ==> -x -3 < r ==> r > x+3 Se r > 4 vai ter 1=< x =<9
> atendendo |x +3| <4 + delta. Portanto x <4
> então |x+3| < 4, conferindo
> x > -3 ==> x+3 <4  ==> x<1, atende.
> se x<-3 atende por hipótese. Mas se quiser conferir. -x - 3 < 4 : -x < 7: x
>>7, mas x <-3, não tem solução.
>
> x>=- 3 ==> x+3>=0 ==> x+3 < r. Se r >=4, existirá solução em [1,9].
>
> Portanto r pertence a (0,4)

Só um detalhe: r = 4 também serve: se |x+3| < 4, temos -7 < x < 1, que
está contido em A.

A minha forma preferida de resolver este exercício é gráfica:
desenhamos o conjunto A, depois tomamos P = -3, e traçamos um
intervalo simétrico em P de maior raio possível contido em A.  Dá r <=
4 ou r < 4 (no desenho, é difícil decidir entre o estrito ou não) e
daí tem que pensar um pouco para detectar se r = 4 serve.

Abraços,
-- 
Bernardo Freitas Paulo da Costa

-- 
Esta mensagem foi verificada pelo sistema de antiv�rus e
 acredita-se estar livre de perigo.


=
Instru��es para entrar na lista, sair da lista e usar a lista em
http://www.mat.puc-rio.br/~obmlistas/obm-l.html
=


[obm-l] Re: [obm-l] Re: [obm-l] Dúvida conceitual (equações)

2018-10-14 Thread Vanderlei Nemitz
Valeu, Pedro! Tomara que mais alguém emita sua opinião.
Um abraço!

Em dom, 14 de out de 2018 18:59, Pedro José  escreveu:

> Boa noite!
> Bom questionamento. Vou me posicionar na arquibancada.
> Minha posição é controversa. Se quer se levar em conta a repetição tem que
> se falar do produto das raízes, cada elevada a sua multiplicidade. No caso
> de soma, cada raiz multiplicada pela multiplicidade.
> Para esse exemplo, o conjunto solução é {1/2,-1} então o produto é -1/2.
> Em suma, não aceito n raízes iguais, mas sim uma raiz de multiplicidade n.
> Se quando queremos provar que algo é unico supomos a existência de dois e
> provamos que são iguais. Creio que seja contraditório dois ou nais iguais.
> Mas vamos observar as diversas posições, pois, creio que o assunto não
> seja pacífico.
> Saudações,
> PJMS
>
> Em Dom, 14 de out de 2018 06:33, Vanderlei Nemitz 
> escreveu:
>
>> Bom dia!
>> Na seguinte questão, que me foi apresentada por um aluno, a resposta
>> proposta é a alternativa C (1/2). Eu sempre pensei que apenas
>> considerávamos multiplicidades em equações polinomiais. Como essa é uma
>> equação exponencial, obtive a resposta B (-1/2). O que é correto pensar?
>>
>> O produto das raízes da equação 16.4^3x - 40.4^2x + 17.4^x - 2 = 0 é
>> igual a:
>> A) 1
>> B) - 0,5
>> C) 0,5
>> D) - 1
>> E) 0
>>
>> Muito obrigado!
>>
>> --
>> Esta mensagem foi verificada pelo sistema de antivírus e
>> acredita-se estar livre de perigo.
>
>
> --
> Esta mensagem foi verificada pelo sistema de antivírus e
> acredita-se estar livre de perigo.

-- 
Esta mensagem foi verificada pelo sistema de antiv�rus e
 acredita-se estar livre de perigo.



[obm-l] Re: [obm-l] Re: [obm-l] Dúvida conceitual (equações)

2018-10-15 Thread Pedro José
Boa tarde!
Artur, não sou contrário a multiplicidade da raiz. Porém, mesmo coma a
multiplicidade, a raiz continua sendo única.
Todavia,não há como negar, facilita sobremaneira as relações de Girard,
para soma e produto é fácil de ajeitar, mas quando passamos a somatório de
produtos dois a dois, três a três... ficaria complicado.

Saudações,
PJMS

Em seg, 15 de out de 2018 às 12:45, Artur Steiner <
artur.costa.stei...@gmail.com> escreveu:

> Isso de se considerar multiplicidades no número de raízes de um polinômio
> é uma convenção conveniente. Facilita muito no caso, por exemplo, das
> famosas relações de Girard. Elas só funcionam se considerarmos as
> multiplicidades. Em análise complexa há também vários teoremas relativos a
> funções analíticas que contam os zeros da função contando multiplicidades.
>
> É claro, por exemplo, que o conjunto de zeros (ou raízes) da função f(x) =
> x^3  é {0}. É uma única raiz com multiplicidade 3. Mas em muitas aplicações
> é mais conveniente supor que são 3 raízes iguais a 0.Sem esquecer que esta
> f só se anula para x = 0.
>
> Há muitas convenções convenientes na matemática. Por exemplo, embora a
> soma seja uma operação binária, convenciona-se que uma soma de uma única
> parcela é a própria parcela. Isto facilita muito.
>
> Artur Costa Steiner
>
> Em dom, 14 de out de 2018 06:33, Vanderlei Nemitz 
> escreveu:
>
>> Bom dia!
>> Na seguinte questão, que me foi apresentada por um aluno, a resposta
>> proposta é a alternativa C (1/2). Eu sempre pensei que apenas
>> considerávamos multiplicidades em equações polinomiais. Como essa é uma
>> equação exponencial, obtive a resposta B (-1/2). O que é correto pensar?
>>
>> O produto das raízes da equação 16.4^3x - 40.4^2x + 17.4^x - 2 = 0 é
>> igual a:
>> A) 1
>> B) - 0,5
>> C) 0,5
>> D) - 1
>> E) 0
>>
>> Muito obrigado!
>>
>> --
>> Esta mensagem foi verificada pelo sistema de antivírus e
>> acredita-se estar livre de perigo.
>
>
> --
> Esta mensagem foi verificada pelo sistema de antivírus e
> acredita-se estar livre de perigo.

-- 
Esta mensagem foi verificada pelo sistema de antiv�rus e
 acredita-se estar livre de perigo.



[obm-l] Re: [obm-l] Re: [obm-l] Dúvida e ajuda.

2022-04-08 Thread Pedro José
Grato a todos!
Já, já tenho de voltar ao trabalho.
Depois dou uma olhada.
Mas achei a demonstração usando casa de pombos, simples e prática.
Já que tem de haver um p/q com pp temos w=x+p/q,
onde x é a parte inteira de w/q, então pq e os restos só podem q-1, uma hora tem de
repetir e aí volta a sequência.
Mas saindo do trabalho dou uma olhada.
Mais uma vez, minha gratidão.

Cordialmente,
PJMS



Em sex., 8 de abr. de 2022 às 13:02, Claudio Buffara <
claudio.buff...@gmail.com> escreveu:

> A volta é fácil também: ao calcular a representação decimal de a/b (a e b
> naturais), nas divisões sucessivas por b só existem b-1 restos possíveis
> (resto = 0 em alguma etapa implica numa decimal finita) e, portanto, após
> não mais do que b-1 divisões, um resto vai se repetir, marcando o início de
> um novo período na representação decimal.
>
> Agora, suponha que  X =
> 0,123456789112233445566778899111222333444555666777888999... seja racional.
> Então existirão n e p naturais tais que, a partir da n-ésima casa decimal
> (1/10^n), os algarismos de X vão se repetir numa sequência com período p.
>
> Mas, pela lei de formação de X, vai existir uma sequência de n+p+1
> algarismos iguais a 1, e esta sequência vai começar após a n-ésima casa
> decimal.
> Ou seja, a sequência vai estar incluída na parte periódica da
> representação decimal de X.
> Mas como o período é p, isso implica que a parte periódica teria que
> ser 111..11 (p algarismos 1) ==> contradição à lei de formação de X.
>
> []s,
> Claudio.
>
>
> On Fri, Apr 8, 2022 at 11:17 AM Pedro José  wrote:
>
>> Bom dia!
>> Posso concluir que um número representado por uma infinidade de
>> algarismos decimais é racional se e somente se tem um período de repetições
>> desses algarismos?
>> A ida é fácil se tiver o período é racional.
>> Já a volta não sei se é verdade e se for há como provar?
>>
>> Meu objetivo primário é saber se:
>> 0,123456789112233445566778899111222333444555666777888999... é racional.
>> As reticências se referem ao aumento de mais um algarismo repetido a cada
>> sequência, ou seja a primeira aparição de 1 será 1, a 2a 11 a 3a 111 e
>> assim sucessivamente, o mesmo vale para os demais algarismos.
>>
>> Alguém poderia me ajudar?
>> Grato,
>> PJMS
>>
>> --
>> Esta mensagem foi verificada pelo sistema de antivírus e
>> acredita-se estar livre de perigo.
>
>
> --
> Esta mensagem foi verificada pelo sistema de antivírus e
> acredita-se estar livre de perigo.

-- 
Esta mensagem foi verificada pelo sistema de antiv�rus e
 acredita-se estar livre de perigo.



[obm-l] RE: [obm-l] Re: [obm-l] dúvida d e interpretação

2009-05-12 Thread Albert Bouskela
Olá Palmerim,

 

Obrigado pela citação!

 

Sua resposta está correta e didática. Não obstante, vou pedir-lhe um favor: 
acho que deveríamos parar de elucidar dúvidas tais como a que foi apresentada 
pelo Marcelo. Acredito que seja prudente preservar o propósito desta Lista: a 
discussão de problemas, que sejam, em tese, pelo menos em potencial, 
pertinentes às Olimpíadas de Matemática. Neste sentido, é de todo conveniente 
que a Lista continue voltada para o objetivo fixado pelo Prof. Nicolau 
Saldanha: apoiar a preparação de estudantes para essas Olimpíadas. Caso esta 
Lista passe a ficar poluída por questões do tipo 
“tire-suas-dúvidas-elementares-em-matemática-básica”, certamente vai afugentar 
aqueles participantes que aqui estão para contribuir com o propósito original 
deste fórum. Além disto, estudantes a exemplo do Marcelo, podem valer-se de 
outros fóruns na Internet, bem mais apropriados, para sanar as suas dúvidas em 
Matemática Básica. Podem, outrossim, contar com os seus professores. 

 

Albert Bouskela

bousk...@msn.com

 

From: owner-ob...@mat.puc-rio.br [mailto:owner-ob...@mat.puc-rio.br] On Behalf 
Of Palmerim Soares
Sent: Tuesday, May 12, 2009 11:32 AM
To: obm-l@mat.puc-rio.br
Subject: [obm-l] Re: [obm-l] dúvida de interpretação

 

Olá Marcelo

 

Numa fração os termos são necessariamente números inteiros. Mas uma fração pode 
representar inúmeras coisas: um número, uma divisão, uma RAZÃO etc. A Razão na 
verdade é uma comparação entre duas quantidades, feita por meio da divisão 
entre essas quantidades, as quais podem ser ou não números inteiros. Por 
exemplo, 2/5 é uma fração, e pode estar representando um número, uma divisão, e 
também uma RAZÃO, se estivermos, por exemplo, comparando a quantidade de 
mulheres numa festa em relação à quantidade de convidados. Neste caso, a RAZÃO 
2/5 quer dizer que de cada 5 convidados, 2 são do sexo feminino. Ou em outras 
palavras, já que 2/5 = 40/100, 40% dos convidados são do sexo feminino. Veja 
que 40/100 é uma fração, mas neste exemplo é também uma razão. Então, 40% é uma 
FRAÇÂO centesimal (denominador igual a 100) e também é uma taxa de comparação 
e, neste sentido, é uma razão. Mas, para os puristas, estaria errado dizer que 
RAIZ(5)/3 é uma fração porque o numerador é irracional e não inteiro; pode 
estar representando uma RAZÃO, mas não é uma fração.

 

Como diria o mestre Bouskela: Fui claro? :-)

 

Abraços 

Palmerim

 

 

 

2009/5/12 Marcelo Costa 

Seguinte: 

 

Pode-se afirmar que uma porcentagem é uma razão especial, uma razão em que o 
consequente é sempre igual a 100 ?!

Se sim, por ex., 25 % = 25/100 = ¼, não é ?! 

Posso ler então, como sendo razão de um para quatro. Está correto ?!

Nesse caso, são 5 partes no total (1 + 4). Onde está a confusão ou o erro de 
interpretação ?!

 

Acho que posso afirmar que uma porcentagem é uma fração (e não uma razão) em 
que o denominador é 100

 

Por ex,:

 

Fração ¼ significa uma parte em quatro. 

Razão ¼ significa uma parte para quatro, perfazendo um total de cinco partes...

 

Favor comentar

 

 

 






-- 
"Matemática é o alfabeto com o qual Deus escreveu o Universo"
Galileu Galilei




-- 
Palmerim



[obm-l] Re: [obm-l] Re: [obm-l] Re: [obm-l] dúvida

2002-10-01 Thread leonardo mattos

Ola Henrique,
Se cada questao vale o dobro da anterior estando a 1ªvalendo 1ponto,as 
outras valerao sempre "potencias de 2",correto?!Colocar 610 na base é uma 
maneira de observar quantas "potencias de 2" existem nele somadas.
610=1*2^9+0*2^8+0*2^7+1*2^6+1*2^5+0*2^4+0*2^3+0*2^2+1*2^1+0*2^0
 Um abraço,Leonardo


>From: "Henrique Branco" <[EMAIL PROTECTED]>
>Reply-To: [EMAIL PROTECTED]
>To: <[EMAIL PROTECTED]>
>Subject: [obm-l] Re: [obm-l] Re: [obm-l] dúvida
>Date: Mon, 30 Sep 2002 23:59:02 -0300
>
>Pergunta besta... Mas se 610 = (1001100010)_2, por que ele acertou as
>perguntas 2, 6, 7, 10?
>Desculpem pelo "nivel primario" da pergunta... Mas...
>Grato,
>Henrique.
>
>- Original Message -
>From: Rodrigo Villard Milet
>To: [EMAIL PROTECTED]
>Sent: Saturday, September 28, 2002 9:00 PM
>Subject: [obm-l] Re: [obm-l] dúvida
>
>Escreva 610 na base 2 : 610 = (1001100010)_2. Como sabemos que a
>representação na base 2 é única, ele acertou as perguntas 2,6,7 e 10.
>Villard
>
>
>-Mensagem original-
>De: Mário Pereira <[EMAIL PROTECTED]>
>Para: [EMAIL PROTECTED] <[EMAIL PROTECTED]>
>Data: Sábado, 28 de Setembro de 2002 11:22
>Assunto: [obm-l] dúvida
>
>Olá, se alguém puder, me dê uma dica:
>
>Em um jogo de televisão, um candidato deve responder a 10 perguntas. A
>primeira vale 1 ponto, a segunda vale 2 pontos, e assim, sucessivamente,
>dobrando sempre. O candidato responde a todas as perguntas e ganha os 
>pontos
>correspondentes às respostas que acertou, mesmo que erre algumas. Se o
>candidato obteve 610 pontos, quantas perguntas acertou?
>
>Obrigado,
>
>Mário.
>
>
>=
>Instruções para entrar na lista, sair da lista e usar a lista em
>http://www.mat.puc-rio.br/~nicolau/olimp/obm-l.html
>O administrador desta lista é <[EMAIL PROTECTED]>
>=




_
Tenha você também um MSN Hotmail, o maior webmail do mundo: 
http://www.hotmail.com/br

=
Instruções para entrar na lista, sair da lista e usar a lista em
http://www.mat.puc-rio.br/~nicolau/olimp/obm-l.html
O administrador desta lista é <[EMAIL PROTECTED]>
=



[obm-l] Re: [obm-l] Re: [obm-l] Re: [obm-l] dúvida

2002-10-01 Thread Eder

Seguindo o racionínio do Villard,veja que se representarmos na base 2 o
valor dos pontos ganhos em cada pergunta ,teremos sempre representações
diferentes.Teremos sempre um "1" e alguns zeros,ou apenas zeros (no caso de
repostas erradas).Nunca teremos duas pontuações com 1's na mesma
posição.Exemplos hipotéticos de pontuações:


0010 0100 0001,mas não aparecem 0110 ou 0010 novamente
O que isso tem de especial?

Na hora de somarmos as pontuações,os 1's aparecerão  na ordem das perguntas
respondidas corretamente!De fato,basta lembrar como se soma números
binários,como teremos sempre 1 "em cima de zero" ou "zero em cima de
zero"

Pois é,basta representar o resultado na base 2 e contar da direita para
esquerda,essa é a ordem das perguntas.Daí ele ter respondido as perguntas
2,6,7 e 10.Note que a representação na base 2 não compromete,ou melhor, ele
está por trás de tudo!Caso a pergunta 1 tivesse sido respondida corretamente
o candidato ganharia 1*2º=1 ponto,conforme expresso no enunciado.Espero ter
ajudado.

Eder

- Original Message -
From: Henrique Branco <[EMAIL PROTECTED]>
To: <[EMAIL PROTECTED]>
Sent: Monday, September 30, 2002 11:59 PM
Subject: [obm-l] Re: [obm-l] Re: [obm-l] dúvida


> Pergunta besta... Mas se 610 = (1001100010)_2, por que ele acertou as
> perguntas 2, 6, 7, 10?
> Desculpem pelo "nivel primario" da pergunta... Mas...
> Grato,
> Henrique.
>
> - Original Message -
> From: Rodrigo Villard Milet
> To: [EMAIL PROTECTED]
> Sent: Saturday, September 28, 2002 9:00 PM
> Subject: [obm-l] Re: [obm-l] dúvida
>
> Escreva 610 na base 2 : 610 = (1001100010)_2. Como sabemos que a
> representação na base 2 é única, ele acertou as perguntas 2,6,7 e 10.
> Villard
>
>
> -Mensagem original-
> De: Mário Pereira <[EMAIL PROTECTED]>
> Para: [EMAIL PROTECTED] <[EMAIL PROTECTED]>
> Data: Sábado, 28 de Setembro de 2002 11:22
> Assunto: [obm-l] dúvida
>
> Olá, se alguém puder, me dê uma dica:
>
> Em um jogo de televisão, um candidato deve responder a 10 perguntas. A
> primeira vale 1 ponto, a segunda vale 2 pontos, e assim, sucessivamente,
> dobrando sempre. O candidato responde a todas as perguntas e ganha os
pontos
> correspondentes às respostas que acertou, mesmo que erre algumas. Se o
> candidato obteve 610 pontos, quantas perguntas acertou?
>
> Obrigado,
>
> Mário.
>
>
> =
> Instruções para entrar na lista, sair da lista e usar a lista em
> http://www.mat.puc-rio.br/~nicolau/olimp/obm-l.html
> O administrador desta lista é <[EMAIL PROTECTED]>
> =

=
Instruções para entrar na lista, sair da lista e usar a lista em
http://www.mat.puc-rio.br/~nicolau/olimp/obm-l.html
O administrador desta lista é <[EMAIL PROTECTED]>
=



[obm-l] Re: [obm-l] Re: [obm-l] Re:[obm-l] dúvida??

2002-10-01 Thread Paulo Santa Rita

Ola Leonardo e demais
colegas desta lista ... OBM-L,

Oi Leonardo, tudo legal ?

Voce esta certo. O PRIMEIRO PULO ocorre quando ela BATE NO CHAO PELA 
PRIMEIRA VEZ, quando, portanto, fora largado da ALTURA INICIAL de 12 metros. 
Segue que a altura que ela atinge neste primeiro pulo e de
12*(2/3).

Esse ultimo valor que encontramos sera a base para se calcular qual a altura 
que ela atinge no segundo pulo, que sera : 12*[(2/3)^2]. E assim 
sucessivamente. No terceiro pulo ela atinge a altura de 12*[(2/3)^3], isto 
e,  32/9

Claramente que no N-esimo pulo ela atinge a altura de 12[(2/3)^N]

Quando ela bate no chao pela N-esima vez e porque ela ja deu N-1 pulos. em 
cada pulo e necessario considerar a SUBIDA E DESCIDA, logo :

Sn = 12 + 2*12*(2/3) + 2*12*(2/3)^2 + ... + 2*12*(2/3)^(N-1)

e o espaco que ela EFETIVAMENTE JA PERCORREU no momento em que toca no solo 
pela N-esima vez ! Em particular, ao tocar no solo pela terceira vez, tera 
efetivamente percorrido :

S3 = 12 + 2*12(2/3) + 2*12*(2/3)^2 = 116/3

Se nos quisessemos encontrar quanto ela teria percorrido ate parar bastaria 
somar a PG infinita. Isto daria :

S = 12 + PG infinita = 12 + [16/(1 - 2/3)] = 12 + 48 = 60

Isto tudo e muito simples, cai em vestibulares e constitui um conhecimento 
basico : Numa PG de termos a1, a2, ..., an se a razao "q" e tal que 
modulo(q) < 1 entao existe :

Lim Sn ( N indo pro infinito ) e Lim Sn = a1 /(1-q).

Em verdade, o passo crucial aqui e ver que se :

Sn = a1 + a1*q + a1*(q)^2 + ... + a1*[q^(N-1)]

Entao

(1-q)*Sn = a1*(1 - q^N) e como q^N -> 0 quando N->+INF segue que
(1-q)*Sn = a1.

Vemos que o polinomio P=f(q)=1-q foi muito util, pois tornou a consideracao 
do limite facil. Agora,

Observando que os expoente de "q" numa PG tradicional constituem uma PA 
ordinaria, isto e, uma PA de ordem 1, podemos imaginar uma "PG" de ordem P 
como aquela "serie geometrica" em que os expoentes de "q" constituem uma PA 
de ordem P. Assim,

a1, a1*q, a1*(q^3), a1*(q^6), ... ,a1*^[q^(N*(N-1)/2)], ...

Claramente que se modulo(q) < 1 a serie :

Sn= a1 + a1*q + a1*(q^3)+ a1*(q^6)+ ...+ a1*^[q^(N*(N-1)/2)]+ ...

converge. Sera que, assim como na serie geometrica, existe um polinomio 
p=f(q) tal que :

f(q)*Sn permita calcular com facilidade o LIM Sn ? Se sim, qual a cara ( a 
forma ) deste polinomio ?

Um Abraco
Paulo Santa Rita
3,1746,011002

>From: "leonardo mattos" <[EMAIL PROTECTED]>
>Reply-To: [EMAIL PROTECTED]
>To: [EMAIL PROTECTED]
>Subject: [obm-l] Re: [obm-l] Re:[obm-l] dúvida??
>Date: Tue, 01 Oct 2002 00:51:25 +
>
>  Ola,
>  Quanto a letra a).Pq nao seria 12*(2/3)^3?!Considerando que no 1ºpulo ela 
>alcança 12*2/3,2ºpulo 12*(2/3)^2 e no 3º12*(2/3)^3...
>   Um abraço,Leonardo
>
>
>>From: "Wagner" <[EMAIL PROTECTED]>
>>Reply-To: [EMAIL PROTECTED]
>>To: <[EMAIL PROTECTED]>
>>Subject: [obm-l] Re:[obm-l] dúvida?? Date: Mon, 30 Sep 2002 20:31:27 -0300
>>
>>Oi pessoal !
>>
>>
>>Mário wrote:
>>
>>Amigos de lista, peço ajuda:
>>
>>
>>"Uma bola pula cada vez que bate no chão 2/3 da altura de onde caiu.
>>Deixando-a cair da altura de 12 metros, pergunta-se:
>>a) qual será a altura do terceiro pulo?
>>b) Quanto percorreu ao bater no chão pela terceira vez?
>>
>>
>>  As alturas máximas a cada pulo estão em progressão geométrica de razão 
>>2/3 e termo inicial 12, assim como a distância percorrida a cada pulo 
>>também forma uma PG de razão 2/3 e termo inicial 24. Logo:
>>
>>a) x = 12.(2/3)^2 = 16/3 metros
>>b) Considerando que a pergunta seja quanto ela percorreu desde que é 
>>abandonada da altura inicial :
>>y = 24.((2/3)^3 - 1)/(2/3 -1) = 24.(-19/27)/(-1/3) = 24.3.19/27 = 152/3 
>>metros
>
>
>
>
>_
>Converse com seus amigos online, faça o download grátis do MSN Messenger: 
>http://messenger.msn.com.br
>
>=
>Instruções para entrar na lista, sair da lista e usar a lista em
>http://www.mat.puc-rio.br/~nicolau/olimp/obm-l.html
>O administrador desta lista é <[EMAIL PROTECTED]>
>=




_
MSN Photos é a maneira mais fácil e prática de editar e compartilhar sua 
fotos: http://photos.msn.com.br

=
Instruções para entrar na lista, sair da lista e usar a lista em
http://www.mat.puc-rio.br/~nicolau/olimp/obm-l.html
O administrador desta lista é <[EMAIL PROTECTED]>
=



[obm-l] Re: [[obm-l] Re: [obm-l] Dúvida de Matemática (Infinitos)]

2003-05-31 Thread Artur Costa Steiner


"Cláudio \(Prática\)" <[EMAIL PROTECTED]> wrote:
> Caro Thiago:
> 
> Não estou bem certo quanto aos infinitos potencial e atual (acho que são
> conceitos que têm mais a ver com filosofia do que com matemática), mas o
> exemplo a seguir pode ser relevante:
> 
> Considere o conjunto N, dos números naturais.
> 
> Por definição, N é um conjunto infinito (considere a bijeção f: N -->
P,
> onde P é o conjunto dos naturais pares, dada por:
> f(n) = 2n. Como P é um subconjunto próprio de N, concluímos que N é
> infinito)
> 
> Assim, N é um "infinito atual", ou seja, um conjunto infinito que já vem
> "pronto".
Estah me parecendo que "atual", aqui, eh uma traducao equivocada do "actual"
do Ingles.
Artur


=
Instruções para entrar na lista, sair da lista e usar a lista em
http://www.mat.puc-rio.br/~nicolau/olimp/obm-l.html
=


[obm-l] Re: [obm-l] Re: [[obm-l] Dúvida de Matemática (Infinitos)]

2003-05-31 Thread Carlos César de Araújo
Prezados Thiago,

Creio que o Artur forneceu as definições básicas que você pediu, exceto as
que se referem aos termos "infinito potencial" e "infinito atual". A
distinção entre

"infinito potencial" e "infinito atual"

remonta a Aristóteles. Foi ressucitada com a teoria dos conjuntos infinitos
de Cantor no século XIX e adquiriram força com o estudo dos paradoxos no
século XX. Há uma ENORME literatura sobre este tema em artigos e periódicos
devotados aos fundamentos da matemática. A distinção poderá parecer meio
"filosófica" ou "metafísica" num primeiro contato, mas pode ser entendida
mais facilmente observando como se utilizam esses dois termos. (A propósito,
Wittgenstein não estava certo ao dizer que o significado de um termo é o seu
uso?)

Os conjuntos infinitos de Cantor são "infinitos atuais" e isto foi o traço
ousado da sua teoria (para a época). Um "infinito atual" é aquele que pode
ser concebido como uma entidade "completa", "acabada": todos  os seus
elementos podem ser pensados num ato único, como já dados à nossa percepção
(ainda que não possamos "visualizá-los" simultaneamente). O "infinito atual"
é o único "infinito" conhecido pelos matemáticos de hoje (com exceção dos
que se dedicam ao estudo crítico dos fundamentos da matemática).

Entretanto, muitos objetaram a esse ponto de vista dizendo que, por exemplo,
os números naturais não podem ser concebidos como um conjunto infinito
"acabado". Segundo esses autores (Kronecker, Poincaré, Brower, etc), é
simplesmente um absurdo admitir que "todos" os números naturais podem ser
pensados coletivamente, como se pudéssemos colocá-los dentro de uma sacola à
qual damos o nome de IN. Para esses autores, a infinidade dos números
naturais é apenas "potencial": podemos sempre imaginar números maiores do
que um número dado previamente, mas não temos o direito de "considerar" ou
"pegar" TODOS os números naturais de uma só vez (como hoje estamos
acostumados a fazer por influência de Cantor). As várias complicações
envolvidas na teoria dos conjuntos de Cantor seriam uma prova (segundo esse
ponto de vista) de que o "infinito atual" é auto-contraditório.

Você ficará surpreso quando descobrir a ENORME quantidade de publicações que
foram devotados a este tema somente no século XX! O tom das discussões
requer algum preparo em filosofia (ontologia e epistemologia; a questão dos
universais; o nominalismo, etc.), lingüística (semântica, sintaxe,
significado, referentes, etc) e, claro, matemática! Quando tiver tempo,
enviarei a esta lista um resumo da bibliografia técnica com a qual estou
familiarizado. No momento, sugiro que leia o meu artigo na página

http://www.gregosetroianos.mat.br/historia.asp

De um ponto de vista puramente pragmático, você pode seguramente "ignorar"
essas incursões pela filosofia quando estiver estudando o infinito em cursos
regulares de matemática. O conceito de "infinito" que vigora nos livros de
Análise e Topologia Geral é (implicitamente) o de "infinito atual", sendo
esta a atitude que os matemáticos herdaram de Cantor.

Carlos César de Araújo
Matemática para Gregos & Troianos
www.gregosetroianos.mat.br
Belo Horizonte, MG

- Original Message -
From: "Artur Costa Steiner" <[EMAIL PROTECTED]>
To: <[EMAIL PROTECTED]>
Sent: Friday, May 30, 2003 10:55 AM
Subject: [obm-l] Re: [[obm-l] Dúvida de Matemática (Infinitos)]


> Oi Thiago
> Comecemos pelas definicoes de conjunto finito e infinito. Sao, de fato, o
que
> os termos sugerem, um conjunto finito tem um numero finito de elementos e,
no
> outro caso, infinitos elementos. Formalmente, dizemos que um conjunto eh
> finito se ele puder ser colocado em correspondencia biunivuca com um
> seguimento inicial do conjunto dos naturais. Dizemos que In eh um
seguimento
> inicial do conjunto dos naturais se, para algum natural n, tivermos In =
> {1,n}.  Logo, se um conjunto eh finito, existe uma bijecao dele sobre
In
> para algum natural n.
> Dizemos que um conjunto eh infinito se nao for finito, isto eh, se nao
houver
> nenhum n para o qual seja possivel encontrar uma bijecao do conjunto sobre
> In.
>
> Os termos enumerável e nao-enumeravel referem-se tambem a conjuntos.
Dizemos
> que um conjunto eh infinito enumeravel se houver uma bijecao dele sobre o
> conjunto N, dos naturais. O conjunto N eh portanto numeravel, pois hah uma
> bijecao dele sobre ele mesmo. Tambem numeravel eh o conjunto dos Z dos
> inteiros, pois podemos obter uma correspondencia biunivica da seguinte
forma
> 0 --- 1
> 1 --- 2
> -1 3
> 2  4
> -2  5
> ..
> n --- 2n
> -n ---2n+1
> ...
>
> O conjunto dos racionais eh numeravel (em qualquer livro de analise hah
esta
> prova). Unioes de colecoes numeraveis de conjuntos numeraveis sao
numeraveis
> (a prova disto, no caso geral, baseia-se em um axioma conhecido por Axioma
da
> Escolha). Produtos cartesianos finitos de conjuntos numeraveis sao
numeraveis,
> logo N^2 e Q^2 sao numeraveis.
>
> Um conjunto eh infinito nao enumeravel se nao houver uma bijecao dele
sobre N.
> 

[obm-l] Re: [obm-l] Re: [[obm-l] Dúvida de Matemática (Infinitos)]

2003-05-31 Thread Carlos César de Araújo
Prezados Thiago e Artur,

Apenas um detalhe quanto à definição de "conjunto finito", e que é RARAMENTE
percebido. Observe essas duas definições:

(1) Um conjunto A é finito se existe n em IN e uma bijeção f: {1,...,n}->A;
(2) Um conjunto A é finito se não existe função injetora f: A->A tal que
f(A)=A.

Embora a primeira definição de "finito" seja a mais natural, foi a segunda
(ou, mais precisamente, a sua negação) que marcou época na história do
pensamento matemático, pois revelou a natureza "paradoxal" do infinito
(atual): um conjunto A é INFINITO sse pode ser colocado em correspondência
biunívoca com um subconjunto próprio! A percepção deste fato foi destacada
como uma notável "descoberta" nos escritos de Bolzano, Dedekind, Cantor e
Peirce. Foi, para eles, o fato fundamental que permitiu uma autêntica
"ciência" do infinito.

Embora Cantor e Dedekind tenham reconhecido em (2) a marca verdadeira do
"infinito", seus tratamentos em artigos publicados foram diferentes. No seu
famoso livreto Was sind und Was sollen die Zahlen ?(1887), Dedekind COMEÇA
com a (negação da) definição (2). Já Cantor, em suas monografias de 1895 e
1897, parte de (1). Ambos tinham como não-problemático que (1) e (2) são
equivalentes, e ambos deram demonstrações dessa equivalência e ressaltaram o
seu papel crucial no estudo dos conjuntos infinitos.

O que demorou a ser percebido foi que a EQUIVALÊNCIA entre as definições (1)
e (2) depende de uma afirmação que causou grande consternação entre os
matemáticos no começo do século XX: o Axioma da Escolha. A prova de que
(1)=>(2) não coloca nenhuma dificuldade séria, mas provar que

(2) => (1)

requer o uso do Axioma da Escolha (mesmo em sua forma mais fraca, o "Axioma
da Escolha Enumerável"). Este fato não foi percebido por Cantor, e foi
"quase" descoberto por Dedekind (sem, contudo, dar-se conta dele por
inteiro).

Hoje, são raros os textos de Análise (pelo menos em português) que mostrem o
papel fundamental do Axioma da Escolha nas demonstrações (em topologia,
geometria diferencial, equações diferenciais, combinatória, etc.). Bem, mas
qual é o problema? Por que é importante ressaltar explicitamente os usos
desse axioma? Isto é assunto para uma outra ocasião, talvez outro lugar ...


Carlos César de Araújo
Matemática para Gregos & Troianos
www.gregosetroianos.mat.br
Belo Horizonte, MG


- Original Message -
From: "Artur Costa Steiner" <[EMAIL PROTECTED]>
To: <[EMAIL PROTECTED]>
Sent: Friday, May 30, 2003 10:55 AM
Subject: [obm-l] Re: [[obm-l] Dúvida de Matemática (Infinitos)]


> Oi Thiago
> Comecemos pelas definicoes de conjunto finito e infinito. Sao, de fato, o
que
> os termos sugerem, um conjunto finito tem um numero finito de elementos e,
no
> outro caso, infinitos elementos. Formalmente, dizemos que um conjunto eh
> finito se ele puder ser colocado em correspondencia biunivuca com um
> seguimento inicial do conjunto dos naturais. Dizemos que In eh um
seguimento
> inicial do conjunto dos naturais se, para algum natural n, tivermos In =
> {1,n}.  Logo, se um conjunto eh finito, existe uma bijecao dele sobre
In
> para algum natural n.
> Dizemos que um conjunto eh infinito se nao for finito, isto eh, se nao
houver
> nenhum n para o qual seja possivel encontrar uma bijecao do conjunto sobre
> In.
>
> Os termos enumerável e nao-enumeravel referem-se tambem a conjuntos.
Dizemos
> que um conjunto eh infinito enumeravel se houver uma bijecao dele sobre o
> conjunto N, dos naturais. O conjunto N eh portanto numeravel, pois hah uma
> bijecao dele sobre ele mesmo. Tambem numeravel eh o conjunto dos Z dos
> inteiros, pois podemos obter uma correspondencia biunivica da seguinte
forma
> 0 --- 1
> 1 --- 2
> -1 3
> 2  4
> -2  5
> ..
> n --- 2n
> -n ---2n+1
> ...
>
> O conjunto dos racionais eh numeravel (em qualquer livro de analise hah
esta
> prova). Unioes de colecoes numeraveis de conjuntos numeraveis sao
numeraveis
> (a prova disto, no caso geral, baseia-se em um axioma conhecido por Axioma
da
> Escolha). Produtos cartesianos finitos de conjuntos numeraveis sao
numeraveis,
> logo N^2 e Q^2 sao numeraveis.
>
> Um conjunto eh infinito nao enumeravel se nao houver uma bijecao dele
sobre N.
> Assim, o conjunto dos reais nao eh numeravel. O intervalo [0,1] tambem nao
eh
> numeravel. De modo geral, intervalos sobre a reta real nao sao numeraveis.
Os
> espacos vetoriais R^n nao sao numeraveis eo conjunto dos complexos tambem
nao
> eh.
>
> Alguns autores utilizam o termo numeravel tanto para conjuntos finitos
como
> para infinito numeraveis.
>
> Agora, com relacao aos termos infinito potencial e infinito atual, eu vou
> ficar devendo. Nunca ouvi estes conceitos antes, pelo menos nao com tais
> denominacoes.
> Um abraco
> Artur
>
> Thiago Luís Tezza <[EMAIL PROTECTED]> wrote:
> >
> >   Olá. Estou com uma dúvida sobre o que é:
> >
> >- Infinito enumerável;
> >- Infinito não-enumerável;
> >- Infinito potencial;
> >- Infinito atual;
> >

[obm-l] Re: [obm-l] Re: [obm-l] Dúvida de Matemática (Infinitos)

2003-05-31 Thread Thiago Luís Tezza
 Muito obrigado, pessoal...
 A conteúdo apresentado foi o que eu necessitava...
 Até,
 Thiago
_
MSN Hotmail, o maior webmail do Brasil.  http://www.hotmail.com
=
Instruções para entrar na lista, sair da lista e usar a lista em
http://www.mat.puc-rio.br/~nicolau/olimp/obm-l.html
=


[obm-l] RE: [obm-l] Re:[obm-l] RE: [obm-l] Dúvida

2004-06-24 Thread Paulo Santa Rita
Oi Claudio,
Se eu nao citei abeliano, foi esquecimento. O Teorema de Cauchy e assim:
"Seja G um grupo FINITO e ABELIANO. Se p e um primo que divide a ordem de G
entao existe um elemento g de G de ordem p".
Esta sua demonstracao ai embaixo e a do Kummer.
Um Abraco
Paulo Santa Rita
5,1356,240604
From: "claudio.buffara" <[EMAIL PROTECTED]>
Reply-To: [EMAIL PROTECTED]
To: "obm-l" <[EMAIL PROTECTED]>
Subject: [obm-l] Re:[obm-l]  RE: [obm-l] Dúvida
Date: Thu, 24 Jun 2004 12:43:59 -0300
Oi, Paulo:
Acho que esta sua demonstracao do teorema de Cauchy soh eh valida se G for 
abeliano, pois no fim, quando voce fala na projecao canonica p: G -> G/H, 
voce estah implicitamente supondo que G/H eh um grupo e, portanto, que H eh 
um subgrupo normal de G. Mas isso soh eh verdade para todo H se G for 
abeliano.

Por outro lado, existe uma demonstracao desse teorema que eh um dos meus 
exemplos favoritos de beleza matematica:

Seja G um grupo e p um primo que divide |G|.
Considere todos os produtos da forma x_1*x_2*...*x_p que sao iguais a "e", 
onde os x_i sao elementos nao necessariamente distintos de G.

Eh facil ver que existem |G|^(p-1) tais produtos pois, escolhendo-se 
livremente os valores de x_1, x_2, ..., x_(p-1), o valor de x_p fica 
unicamente determinado (igual ao inverso de x_1*x_2*...*x_(p-1))

Agora vamos dividir estes |G|^(p-1) produtos em classes de equivalencia de 
forma que dois produtos pertencem a uma mesma classe se e somente se um 
deles for uma permutacao circular do outro. Teremos dois casos a 
considerar:

Caso 1: todos os x_i sao iguais.
Nesse caso, a classe vai conter apenas um produto, pois x_1*x_2*...*x_p = 
a*a*...*a e existe apenas uma permutacao dos x_i.

Caso 2: pelo menos dois dos x_i sao distintos.
Nesse caso, a classe vai conter exatamente p produtos:
x_1*x_2*...*x_(p-1)*x_p;
x_2*x_3*... x_p*x_1;
x_3*x_4*...*x_1*x_2;
...
x_p*x_1*...x_(p-1)*x_(p-1).
Sejam N1 e N2 os numeros de classes de equivalencia de cada tipo.
Entao, teremos que:
numero de produtos =  1*N1 + p*N2 = |G|^(p-1).
Por hipotese, p | |G|^(p-1) e obviamente p | p*N2.
Logo, p | N1.
Alem disso, o produto e*e*...*e obviamente eh do tipo 1, de modo que N1 > 
0.

Ou seja, o numero N1 de produtos da forma a*a*...*a = a^p = e eh um 
multiplo positivo de p.
Em outras palavras, existem pelo menos p-1 elementos em G de ordem p.
Naturalmente, se a eh um tal elemento, entao , o subgrupo ciclico gerado 
por a, terah ordem p.

[]s,
Claudio.
De:[EMAIL PROTECTED]
Para:[EMAIL PROTECTED]
Cópia:
Data:Thu, 24 Jun 2004 14:20:38 +
Assunto:[obm-l] RE: [obm-l] Dúvida

> Ola Eder,
>
> Ok !
>
> Vamos fazer o seguinte. Vou provar um resultado classico que voce podera
> usar na solucao.
>
> TEOREMA DE CAUCHY : Se G e um grupo finito e "p" e um numero primo que
> divide
> a ordem de G entao existe um elemento "g" de G de ordem "p".
>
> PROVA : Vamos usar inducao sobre a ordem de G. Mais especificamente 
vamos
> mostrar que
> ( HIPOTESE DE INDUCAO ) se todos os grupos com ordem menor que G 
satisfazem
> o TEOREMA DE CAUCHY entao G satisfaz o TEOREMA DE CAUCHY.
>
> 1) Se a ordem de G for um numero primo, |G| = p, entao a prova e trivial 
e
> nem precisamos usar a hipotese de inducao, pois "p" sera o unico numero
> primo que pode dividir a ordem de G e se "g"
> for um elemento de G entao, pelo teorema de Lagrange, divide |G|, isto
> e, a ordem de
> "g" e "p". Assim, nao so um, mas todos os elementos de G ( com excecao 
da
> identidade ) tem
> ordem "p"
>
> 2) Se ordem de G nao for um numero primo, seja "p" um numero primo que
> divide a ordem de G.
> Tomando um elemento "g" pertencente a G, "g" diferente de "e", considere 
o
> subgrupo de G : H=. Existem duas possibilidades para H :
>
> PRIMEIRA : H e igual a G. Neste caso, G e ciclico com G=. Seja N=|G| e
> considere o elemento g^(N/p). Claramente que g^(N/p) pertence a G e 
ordem de
> g^(N/p) e "p". Assim,
> G tem um elemento de ordem "p" e acabou.
>
> SEGUNDA : H e diferente de G. Neste caso |H| < |G|.
>
> Se "p" divide |H|, pela HIPOTESE DE INDUCAO, existe "h" pertencente a H 
tal
> que ordem de "h" e "p". Como H e subconjunto de G segue que "h" e tambem
> elmento de G e, portanto, G tem um elemento de ordem "p" e acabou.
>
> Se "p" nao divide |H| ( mas "p" divide |G|, por hipotese ), pelo teorema 
de
> Lagrange |G|=|H|(G:H) teremos que "p" divide (G:H), isto e, "p" divide o
> indice de H em G. Como (G:H) =| G/H | e
> G/H| < |G|, pela HIPOTESE DE INDUCAO, existe um h_ ( h barra ) em G/H de
> ordem "p".
>
> Considere a projecao canonica :
>
>

[obm-l] RE: [obm-l] Re: [obm -l] Dúvida-Geometria ana lítica

2010-10-11 Thread marcone augusto araújo borges

Eu tambem estranhei essas medidas mas penso q o triângulo existe.A mediana de 
BC divide o triângulo ABC em dois de mesma àrea((12/2)=6 unidades).Seja D o pé 
da referida mediana.A área do triângulo ABD=(1/2)*4*3*senÂ=6.Dai,senÂ=1 e  é 
um ângulo reto,então BD=CD=5 implica BC=10.Nesse caso,a projeção de AC sobre a 
reta suporte de AB mede 4 e AC = raiz(52).Portanto,ABC seria obtusângulo,pois 
BC^2>AB^2+AC^2(100>16+52).Para testar,calculei a área de ABC usando as medidas 
dos seus lados e o resultado bateu:12 unidades.Quando perguntei não estava tão 
convicto e queria ver uma solução diferente tambem.

Obrigado pela sua atenção.
 


Date: Sun, 10 Oct 2010 16:57:11 -0700
From: eduardowil...@yahoo.com.br
Subject: [obm-l] Re: [obm-l] Dúvida-Geometria ana lítica
To: obm-l@mat.puc-rio.br





Pelo menos na geometria euclidiana esse triângulo inexiste: a altura relativa à 
AB vale 6, logo o pé da referida mediana está à uma altura de 3, logo sua 
medida tem que ser maior... 

  

[obm-l] Re: [obm-l] Re:[obm-l] Dúvida-Geometria ana lítica

2010-10-11 Thread Ralph Teixeira
Eu pensei dum jeito mais "desenho geométrico". No fundo, é a mesma solução
do Marcone acima, só pensada de outro jeito:

Faça uma figura com A, B e a reta y=6 onde mora C. Novamente, o ponto-chave
da questão é D, o médio de BC.

Como C está na reta y=6 e B=(4,0), o médio D está na reta y=3 (homotetia de
centro B e razão 1/2 aplicada à reta y=3).

Por outro lado, D está no círculo de centro A e raio 3. Intersectando este
círculo com a reta anterior, achamos as possíveis posições para D.

Neste problema em particular, a interseção destes 2 L.G.s é única, é o ponto
(0,3) (o círculo é tangente à reta y=3). Então D=(0,3), portanto C=(-4,6).
Como a primeira coordenada de C é negativa, o ângulo em A é obtuso, o
triângulo é obtusângulo.

Abraço,
 Ralph


[obm-l] Re: [obm-l] Re: [obm-l] Dúvida - OBM Nível Universitário

2011-01-24 Thread Hugo Fernando Marques Fernandes
Oi, Bruna.

Pois é, eu já tinha ouvido dizer isso e queria confirmar.
É uma pena, mas fazer o que? Regras são regras...

Talvez fosse o caso de criar uma categoria nova pra quem já tem diploma, né?

Obrigado pela resposta.

Hugo.

Em 24 de janeiro de 2011 02:20, Bruna Campos escreveu:

> PS.: E só até o quarto ano de graduação.
>
> Abraços!
>
> Em 23/01/11, Bruna Campos escreveu:
> > Hugo, que eu saiba não pode. Só pode participar quem não tem diploma
> > de curso superior :(
> >
> >
> >
> >
> > Em 20/01/11, Hugo Fernando Marques Fernandes
> > escreveu:
> >> Boa noite.
> >>
> >> Acabo de ser aprovado para o curso de matemática da UERJ e tenho uma
> >> dúvida
> >> em relação à OBM de nível universitário. Sendo esta minha segunda
> >> graduação,
> >> ou seja, possuindo um outro diploma de nível superior, ainda assim
> >> poderei
> >> participar?
> >>
> >> Grato por sua atenção.
> >>
> >> Hugo F. M. Fernandes.
> >>
> >
>
> =
> Instruções para entrar na lista, sair da lista e usar a lista em
> http://www.mat.puc-rio.br/~obmlistas/obm-l.html
> =
>


[obm-l] RE: [obm-l] Re: [obm-l] dúvida - teoria dos números

2011-08-05 Thread Rhilbert Rivera

Pequeno Teorema de Fermat: a^(p-1) ==1(mod p), se mdc(a,p)=1.

Como 47 é primo e mdc(2,47) =1, então  2^46 ==1 (mod 47). É claro que podemos 
dizer ( de acordo com as propriedades das potências nas  congruências) que 
2^23==1 (mod 47), o que nos leva 2^23 -1 ==0(mod 47).

Date: Sun, 31 Jul 2011 16:00:56 -0700
From: jeffma...@yahoo.com.br
Subject: [obm-l] Re: [obm-l] dúvida - teoria dos números
To: obm-l@mat.puc-rio.br

Tente pensar no pequeno Teorema de Newton, ou se preferir use congruencias.abs
De: Marco Antonio Leal 
Para: obm-l@mat.puc-rio.br
Enviadas: Quinta-feira, 28 de Julho de 2011 21:39
Assunto: [obm-l] dúvida - teoria dos números






Não estou conseguindo uma prova satisfatória para o seguinte exercício:

 

prove que 2 ^23  -  1 é divisivel por 47.

 
   


  

[obm-l] Re: [obm-l] Re: [obm-l] Re: [obm-l] Dúvida

2011-11-02 Thread Joao Maldonado



Não tem como ser isso não cara
Traduz isso aí que   não dá pra entender



O que poderia ser é
Mostre que  qualquer que seja o número  racional e positivo  a/b com a e  b 
inteiros  primos entre si,  é válido   que f(a/b) =  f(1)^(a/b)    



Tudo bem,  vamos dizer  que é isto, mas qual a regra para  a função f(x)??

Não são  todas as funções f(x)  que  satisfazem tal  afirmação  (aliás,  a 
única   que consegui pensar até agora é  f(x) = a^x

f(x) = x²,  f(4/7) = 16/49 !=   1^(4/7) = 1

[]'s

João






Date: Wed, 2 Nov 2011 17:17:59 -0200
Subject: [obm-l] Dúvida
From: klebe...@gmail.com
To: obm-l@mat.puc-rio.br

Olá grupo,
Estou me enrolando nesta prova.

Mostre q ∀ nº a/b>0, MDC(a,b) = 1, 
é válido: f(a/b) = f(1)^a/b .

-- 
Kleber.



De: Kleber Bastos 
Para: obm-l@mat.puc-rio.br
Enviadas: Quarta-feira, 2 de Novembro de 2011 22:21
Assunto: [obm-l] Re: [obm-l] Re: [obm-l] Dúvida


É isso mesmo:
Mostrar que ∀ nº racional a/b>0, M.D.C.(a,b)=1 é válida a sentença: 
f(a/b)=f(1)^a/b ( f(1) elevado a a/b)


Em 2 de novembro de 2011 20:57, Victor Hugo Rodrigues 
 escreveu:

Como assim? Acho que falta algo aí.
>
>
>Em 2 de novembro de 2011 17:17, Kleber Bastos  escreveu:
>
>
>Olá grupo,
>>Estou me enrolando nesta prova.
>>
>>Mostre q ∀ nº a/b>0, MDC(a,b) = 1, 
>>é válido: f(a/b) = f(1)^a/b
.
>>
>>-- 
>>Kleber.
>>
>


-- 
Kleber B. Bastos

Re: [obm-l] Re: [obm-l] Re:[obm-l] dúvida (Q uadriláteros)

2006-05-05 Thread cleber vieira
Valeu claudio, a idéia de fazer DO = OX  e daí provar que X coincide com H foi um xeque-mate no problema, parabéns e muito obrigado pela sua resolução.  Abraços  Cleber 
		 
Abra sua conta no Yahoo! Mail - 1GB de espaço, alertas de e-mail no celular e anti-spam realmente eficaz. 

[obm-l] Re: [obm-l] Re: [obm-l] dúvida - poblema das casas

2004-01-30 Thread Eduardo Azevedo
Boa obsevação. Agora ficou moleza!

Obrigado Nicolau e Arthur,

Abraco

-Eduardo



- Original Message - 
From: "Nicolau C. Saldanha" <[EMAIL PROTECTED]>
To: <[EMAIL PROTECTED]>
Sent: Wednesday, January 28, 2004 2:14 PM
Subject: [obm-l] Re: [obm-l] dúvida - poblema das casas


> On Wed, Jan 28, 2004 at 01:32:13PM -0200, Eduardo Azevedo wrote:
> > Tava fazendo esse problema das casas a um tempo atras:
> >
> > http://acm.uva.es/p/v1/138.html
> >
> > Ele se resume a encontrar inteiros 0 < k < n.  E a soma dos números
antes de
> > k tem que ser igual a soma dos números de k+1 até n. Por exemplo 1 e 1
ou 6 e
> > 8, ou 71631910824649559 e 101302819786919521.
>
> Reescreva isso como
>
> n(n+1)/2 = 2*(k(k-1)/2) + k
>
> ou, depois de um pouco de álgebra,
>
> (2n + 1)^2 - 2 (2k)^2 = 1
>
> Esta é uma modificação mínima da equação de Pell.
> A equação de Pell usual é:
>
> x^2 - a y^2 = 1
>
> onde a é um inteiro, no nosso caso 2.
> As soluções da equação de Pell estão em bijeção natural
> com os elementos de norma 1 de
> Z[sqrt(2)] = {x + y sqrt(2); x, y em Z}.
> A norma de x + y sqrt(2) é x^2 - 2 y^2. Os elementos de norma 1
> são exatamente +- as potências inteiras de 3 + 2 sqrt(2).
> A partir daí não é muito difícil tirar a forma geral das soluções
> do seu problema e demonstrar as suas observações experimentais.
>
> Você pode ler sobre a equação de Pell em qq livro de teoria dos
> números. Acho que já saiu um artigo na Eureka também.
>
> []s, N.
> =
> Instruções para entrar na lista, sair da lista e usar a lista em
> http://www.mat.puc-rio.br/~nicolau/olimp/obm-l.html
> =

=
Instruções para entrar na lista, sair da lista e usar a lista em
http://www.mat.puc-rio.br/~nicolau/olimp/obm-l.html
=


[obm-l] Re: [obm-l] Re: [obm-l] Re: [obm-l] dúvida

2004-02-28 Thread Rafael
Tarcio,

O enunciado não dá margem a interpretar que se tratem de grupos formados
para os quais a ordem de escolha importa. ("Dispomos de 10 livros diferentes
e queremos organizar grupos de três livros. O número de grupos diferentes
que podemos formar é igual a...?", observe que os "grupos" não foram
definidos, não se sabe se são grupos de livros que possuem a mesma capa, que
fazem parte de uma mesma coleção, absolutamente nada. Só se sabe que são
livros diferentes e que os grupos possuem três deles.) O Nicolau, aliás, deu
um exemplo bastante consistente sobre isso. Mas não é a primeira vez que
vejo enunciados interpretados de forma incorreta por quem resolveu. Certa
vez, aconteceu comigo quando estava estudando permutações caóticas, o que
realmente foi um caos, pois era um dos primeiros exercícios que eu resolvia.
A única sugestão é procurar bons livros para começar, a coleção do Iezzi
possui um livro excelente de Combinatória (volume 5), e depois que você
tiver os conceitos bem claros já saberá quando algo é incoerente.

Aproveitando a oportunidade, alguns dias atrás, você enviou novamente aquele
problema de capital que decuplicou, e nenhuma das alternativas está correta
para o enunciado dado. Por curiosidade, calculei aplicando juros simples e
juros compostos, comparando com a alternativa correta que você havia dito
(12/7% a.m.). Veja:

Seja C o capital envolvido, a juros simples, teremos:

10C = C(1+i*7*12) ==> i = 0,10714285714...

A juros compostos, teremos:

10C = C(1+i)^(7*12) ==> i = 0,02779088522...

Porém, 12/7% = 0,01714285714...

E, ainda assim, há uma diferença de 1% (aprox.) entre a resposta dada como
correta e a taxa de juros, considerando que fossem compostos, e não simples
como dito no enunciado.

Viu só?


Abraços,

Rafael de A. Sampaio




- Original Message -
From: "Tarcio Santiago" <[EMAIL PROTECTED]>
To: <[EMAIL PROTECTED]>
Sent: Saturday, February 28, 2004 8:16 PM
Subject: [obm-l] Re: [obm-l] Re: [obm-l] dúvida


AMIGO RAFAEL OBRIGADO POR SUA AJUDA. A RESPOSTA É 720, MAS EU ACHAVA QUE O
CERTO ERA 120, POIS O LIVRO  O TRIO DE LIVROS A,Be C é igual a B,CeA . estou
errado?
a questão dar 120 ou 720 ?
estou encucado!!?

=
Instruções para entrar na lista, sair da lista e usar a lista em
http://www.mat.puc-rio.br/~nicolau/olimp/obm-l.html
=


[obm-l] Re:[obm-l] Re: [obm-l] Re:[obm-l] dúvida

2004-04-25 Thread rickufrj
-- Início da mensagem original ---

  De: [EMAIL PROTECTED]
Para: [EMAIL PROTECTED]
  Cc: 
Data: Sun, 25 Apr 2004 10:10:46 -0300
 Assunto: [obm-l] Re: [obm-l] Re:[obm-l] dúvida

> No caso, não entendi o porque do i*(raiz de 1998), 
visto que ao elevarmos ao
> quadrado,  i^2= -1 e a expressão seria -1998
> 
> Creio que apenas (raiz de 1998) seja mais correto.
> 
> Porém a questão não deve ser apenas isto, a e b 
devem pertencer a algum
> conjunto específico como os Inteiros...
> 
> Abraços,
> Rossi
> 
É verdade Rossi , acho que ontem a noite eu estava 
meio 'grog' ... rs
Mas , tenta fazer essa questão por calculo .Tipo , é 
obvio que a diferença mínima é 0 , mas quando vc faz 
isso formalmente , aparece i na raiz . É bem 
estranho , na hora que eu fiz eu não consegui ver o pq 
disso.
Abraço
Luiz H. 

 
__
Acabe com aquelas janelinhas que pulam na sua tela.
AntiPop-up UOL - É grátis!
http://antipopup.uol.com.br/



=
Instruções para entrar na lista, sair da lista e usar a lista em
http://www.mat.puc-rio.br/~nicolau/olimp/obm-l.html
=


[obm-l] Re: [obm-l] Re: [obm-l] Dúvida Análise(tannery's theorem)

2015-05-05 Thread Israel Meireles Chrisostomo
Obrigado a ambos, as suas respostas são ambas interessantes.Em particular
quero agradecer ao Ralph, que mesmo depois de eu o contrapor em
argumentos(que por sinal eram infundados) em uma outa pergunta, mesmo assim
respondeu com paciência minha dúvida

Em 5 de maio de 2015 10:40, Ralph Teixeira  escreveu:

> Não sei se entendi a pergunta também... Mas *talvez* esse seja um exemplo
> bom...
>
> Considere a sequencia dupla a(k,n) (onde k,n=1,2,3,...) dada por:
>
> a(k,n) = 1/k se n<=k
> a(k,n) = 0 se n>k
>
> Ou seja, mais explicitamente, colocando k fixo e variando n em cada linha:
>
> a(1,n): 1,0,0,0,0,0,0,..
> a(2,n): 1/2, 1/2, 0, 0, 0, 0, ...
> a(3,n): 1/3, 1/3, 1/3, 0, 0, 0, 0,...
> a(4,n): 1/4, 1/4, 1/4, 1/4, 0, 0, 0, 0,...
>
> Então S(k)=sum(n=1 a +Inf) a(k,n) = 1 (soma da linha), e portanto lim
> (k->Inf) S(k) = 1 também.
>
> Porém se você fixar n e tomar primeiro o limite k->Inf, verá que lim
> (k->Inf) a(k,n) = 0. Então a soma dos limites dos a(k,n) é 0 -- não deu 1!
>
> Em suma: a soma (infinita) dos limites não é o limite da soma (infinita).
>
> Era algo assim que você queria?
>
> Abraço, Ralph.
>
> 2015-05-05 1:40 GMT-03:00 Israel Meireles Chrisostomo <
> israelmchrisost...@gmail.com>:
>
>> Olá tenho um dúvida de análise seja a_k(n) um termo dependente de n e a_k
>> o resultado do limite lim n->inf a_k(n)=a_k, se |Sa_k(n)-Sa_k|<épsilon, com
>> épsilon maior que zero então, isto significa dizer que lim
>> n->inf  Sa_k(n)=S a_k(em que S está no lugar de sigma e representa a soma
>> da série)?Se a resposta é sim, alguém poderia me explicar o pq?No caso, eu
>> não deveria ter épsilon próximo de zero?Tipo assim, para concluir que ambas
>> se são "iguais", se alguém puder me ajudar, é que estou lendo sobre o
>> Tannery's Theorem para séries
>>
>>
>> --
>> Esta mensagem foi verificada pelo sistema de antivírus e
>> acredita-se estar livre de perigo.
>
>
>
> --
> Esta mensagem foi verificada pelo sistema de antivírus e
> acredita-se estar livre de perigo.
>

-- 
Esta mensagem foi verificada pelo sistema de antiv�rus e
 acredita-se estar livre de perigo.



[obm-l] Re: [obm-l] Re: [obm-l] Re: [obm-l] dúvida

2015-07-09 Thread Israel Meireles Chrisostomo
Obrigado Ralph

Em 9 de julho de 2015 12:37, Ralph Teixeira  escreveu:

> Vamos generalizar para R^n: com a noção usual (Euclideana) de comprimento,
> o comprimento do segmento que liga (x1,x2,...,xn) a (y1,y2,...,yn) é:
>
> d=raiz((y1-x1)^2+(y2-x2)^2+...+(yn-xn)^2)
>
> Esta é a noção usual de distância entre dois pontos -- confira que é o que
> você conhece na reta (n=1) e no plano (n=2).
>
> Abraço, Ralph.
>
> 2015-07-09 10:27 GMT-03:00 Pedro José :
>
>> Bom dia!
>>
>> E o segmento???
>>
>> Em 8 de julho de 2015 21:48, Israel Meireles Chrisostomo <
>> israelmchrisost...@gmail.com> escreveu:
>>
>>> Como posso encontrar o comprimento de um segmento de reta no espaço
>>> tridimensional?Considere a origem da reta no ponto (x_0,y_0,z_0) e o final
>>> da reta no ponto (x_1,y_1,z_1)
>>>
>>> --
>>> Esta mensagem foi verificada pelo sistema de antivírus e
>>> acredita-se estar livre de perigo.
>>
>>
>>
>> --
>> Esta mensagem foi verificada pelo sistema de antivírus e
>> acredita-se estar livre de perigo.
>
>
>
> --
> Esta mensagem foi verificada pelo sistema de antivírus e
> acredita-se estar livre de perigo.
>

-- 
Esta mensagem foi verificada pelo sistema de antiv�rus e
 acredita-se estar livre de perigo.



[obm-l] Re: [obm-l] Re: [obm-l] Dúvida em Geometria Plana

2016-06-02 Thread Daniel Rocha
Muito Obrigado, Carlos !!!

Em 2 de junho de 2016 18:54, Carlos Gomes  escreveu:

> Seja x a medida do ângulo BAC. Como o triângulo APQ é isosceles de base
> AP, segue q a medida do ângulo APQ também é x. Note que o ângulo BQP é
> externo ao triângulo APQ, portanto, mede x+x=2x. Agora como o triângulo BQP
> é isosceles de base BQ, segue que o ângulo PBQ também mede 2x. Por fim note
> que o ângulo BPC é externo ao triângulo  ABP, portanto mede x+2x=3x...como
> o triângulo BCP também é isosceles de base PC, segue que o ângulo PCB
> também mede 3x...como o triângulo ABC é isosceles, segue que o ângulo ABC
> também mede 3x, o que revela q o ângulo PCB mede x. Assim, no triângulo BCP
> temos que
> x+3x+3x=π   ==>x=π/7.
> Em 2 de jun de 2016 18:32, "Daniel Rocha" 
> escreveu:
>
>> Olá a todos,
>>
>> Alguém poderia, por favor, apresentar os cálculos corretos da seguinte
>> questão:
>>
>> Considere um triângulo ABC isósceles de base BC, e os pontos P e Q tais
>> que P pertence a AC e Q pertence a AB. Se BC=BP=PQ=QA, a medida do ângulo
>> do vértice A, em radianos, é:
>>
>> GABARITO: Pi/7.
>>
>> Eu agradeço a quem apresentar os cálculos corretos.
>>
>> --
>> Esta mensagem foi verificada pelo sistema de antivírus e
>> acredita-se estar livre de perigo.
>
>
> --
> Esta mensagem foi verificada pelo sistema de antivírus e
> acredita-se estar livre de perigo.

-- 
Esta mensagem foi verificada pelo sistema de antiv�rus e
 acredita-se estar livre de perigo.



[obm-l] Re: [obm-l] Re: [obm-l] Dúvida em Geometria Espacial

2016-06-08 Thread Daniel Rocha
Muito Obrigado, Grande Carlos !!!

Em 8 de junho de 2016 20:13, Carlos Gomes  escreveu:

> Vc pode fazer assim:
>
> área total = 60 ==> 2.pi.r^2+2.pi.r.h=60  ==>h=(60-2.pi.r^2)/(2.pi.r)   (*)
> Por outro lado o volume é
>
> V=pi.r^2.h
>
> substituindo a expressão (*) do h , segue que
>
> V=60r-2.pi.r^3
>
> Fazendo dV/dr=0 (derivada igual a zero para achar os pontos críticos),
> segue que
>
> 0=60-6.pir^2  ==> r=sqtr(10/pi) (é ponto de máximo, pois
> d^2V/dr^2(sqtr(10/pi))<0
>
> substituindo com esse valor de r na expressão (*), segue que
> h=2.sqrt(10/pi).
>
> O que revela que r/h=sqtr(10/pi) / 2.sqtr(10/pi)=1/2.
>
> Cgomes.
>
> Em 8 de junho de 2016 19:17, Daniel Rocha 
> escreveu:
>
>> Alguém poderia, por favor, solucionar a questão abaixo:
>>
>> Uma lata de forma cilíndrica, com tampa, deve ser construída com 60 cm^2
>> de folha de alumínio. Se r é o raio da base e h é a altura da lata que
>> proporcionam o volume máximo, então o valor de r/h é:
>>
>> GABARITO: 1/2
>>
>> --
>> Esta mensagem foi verificada pelo sistema de antivírus e
>> acredita-se estar livre de perigo.
>
>
>
> --
> Esta mensagem foi verificada pelo sistema de antivírus e
> acredita-se estar livre de perigo.

-- 
Esta mensagem foi verificada pelo sistema de antiv�rus e
 acredita-se estar livre de perigo.



Re: [obm-l] Re: [obm-l] Re: [obm-l] Dúvida conceitual (equações)

2018-10-15 Thread Claudio Buffara
Qual a soma das raizes de (2^x - 8)^3 = 0?
Se a equação acima fosse apresentada como:
2^(3x) - 24*2^(2x) + 192*2^x - 512 = 0,
isso mudaria sua resposta?

Enviado do meu iPhone

Em 15 de out de 2018, à(s) 00:29, Vanderlei Nemitz  
escreveu:

> Valeu, Pedro! Tomara que mais alguém emita sua opinião.
> Um abraço!
> 
> Em dom, 14 de out de 2018 18:59, Pedro José  escreveu:
>> Boa noite!
>> Bom questionamento. Vou me posicionar na arquibancada. 
>> Minha posição é controversa. Se quer se levar em conta a repetição tem 
>> que se falar do produto das raízes, cada elevada a sua multiplicidade. No 
>> caso de soma, cada raiz multiplicada pela multiplicidade.
>> Para esse exemplo, o conjunto solução é {1/2,-1} então o produto é -1/2.
>> Em suma, não aceito n raízes iguais, mas sim uma raiz de multiplicidade n.
>> Se quando queremos provar que algo é unico supomos a existência de dois e 
>> provamos que são iguais. Creio que seja contraditório dois ou nais iguais.
>> Mas vamos observar as diversas posições, pois, creio que o assunto não 
>> seja pacífico. 
>> Saudações, 
>> PJMSÂ 
>> 
>> Em Dom, 14 de out de 2018 06:33, Vanderlei Nemitz  
>> escreveu:
>>> Bom dia!
>>> Na seguinte questão, que me foi apresentada por um aluno, a resposta 
>>> proposta é a alternativa C (1/2). Eu sempre pensei que apenas 
>>> considerávamos multiplicidades em equações polinomiais. Como essa é uma 
>>> equação exponencial, obtive a resposta B (-1/2). O que é correto pensar?
>>> 
>>> O produto das raízes da equação 16.4^3x - 40.4^2x + 17.4^x - 2 = 0 é 
>>> igual a:
>>> A) 1
>>> B) - 0,5
>>> C) 0,5
>>> D) - 1
>>> E) 0
>>> 
>>> Muito obrigado!
>>> 
>>> -- 
>>> Esta mensagem foi verificada pelo sistema de antivírus e 
>>> acredita-se estar livre de perigo.
>> 
>> -- 
>> Esta mensagem foi verificada pelo sistema de antivírus e 
>> acredita-se estar livre de perigo.
> 
> -- 
> Esta mensagem foi verificada pelo sistema de antivírus e 
> acredita-se estar livre de perigo.

-- 
Esta mensagem foi verificada pelo sistema de antiv�rus e
 acredita-se estar livre de perigo.



Re: [obm-l] Re: [obm-l] Re: [obm-l] Dúvida conceitual (equações)

2018-10-15 Thread Kevin Felipe Kühl Oliveira
Exatamente nisso que estava pensando. Se fizessemos 4^x = y teriamos uma 
equação polinomial de grau 3, ai fica mais evidente a existência de múltiplas 
raizes.

Abraços

Kevin Kühl
On 15 Oct 2018 07:25 -0300, Claudio Buffara , wrote:
> Qual a soma das raizes de (2^x - 8)^3 = 0?
> Se a equação acima fosse apresentada como:
> 2^(3x) - 24*2^(2x) + 192*2^x - 512 = 0,
> isso mudaria sua resposta?
>
> Enviado do meu iPhone
>
> Em 15 de out de 2018, à(s) 00:29, Vanderlei Nemitz  
> escreveu:
>
> > Valeu, Pedro! Tomara que mais alguém emita sua opinião.
> > Um abraço!
> >
> > > Em dom, 14 de out de 2018 18:59, Pedro José  
> > > escreveu:
> > > > Boa noite!
> > > > Bom questionamento. Vou me posicionar na arquibancada.Â
> > > > Minha posição é controversa. Se quer se levar em conta a repetição 
> > > > tem que se falar do produto das raízes, cada elevada a sua 
> > > > multiplicidade. No caso de soma, cada raiz multiplicada pela 
> > > > multiplicidade.
> > > > Para esse exemplo, o conjunto solução é {1/2,-1} então o produto é 
> > > > -1/2.
> > > > Em suma, não aceito n raízes iguais, mas sim uma raiz de 
> > > > multiplicidade n.
> > > > Se quando queremos provar que algo é unico supomos a existência de 
> > > > dois e provamos que são iguais. Creio que seja contraditório dois ou 
> > > > nais iguais.
> > > > Mas vamos observar as diversas posições, pois, creio que o assunto 
> > > > não seja pacífico.Â
> > > > Saudações,Â
> > > > PJMSÂ
> > > >
> > > > > Em Dom, 14 de out de 2018 06:33, Vanderlei Nemitz 
> > > > >  escreveu:
> > > > > > Bom dia!
> > > > > > Na seguinte questão, que me foi apresentada por um aluno, a 
> > > > > > resposta proposta é a alternativa C (1/2). Eu sempre pensei que 
> > > > > > apenas considerávamos multiplicidades em equações polinomiais. 
> > > > > > Como essa é uma equação exponencial, obtive a resposta B (-1/2). 
> > > > > > O que é correto pensar?
> > > > > >
> > > > > > O produto das raízes da equação 16.4^3x - 40.4^2x + 17.4^x - 2 = 
> > > > > > 0 é igual a:
> > > > > > A) 1
> > > > > > B) - 0,5
> > > > > > C) 0,5
> > > > > > D) - 1
> > > > > > E) 0
> > > > > >
> > > > > > Muito obrigado!
> > > > > >
> > > > > > --
> > > > > > Esta mensagem foi verificada pelo sistema de antivírus e
> > > > > > acredita-se estar livre de perigo.
> > > >
> > > > --
> > > > Esta mensagem foi verificada pelo sistema de antivírus e
> > > > acredita-se estar livre de perigo.
> >
> > --
> > Esta mensagem foi verificada pelo sistema de antivírus e
> > acredita-se estar livre de perigo.
>
> --
> Esta mensagem foi verificada pelo sistema de antiv�rus e
> acredita-se estar livre de perigo.

-- 
Esta mensagem foi verificada pelo sistema de antiv�rus e
 acredita-se estar livre de perigo.



[obm-l] Re: [obm-l] Re: [obm-l] dúvida sobre a OBMU

2019-01-20 Thread Pedro Soares
Combinatória aproveita bastante.
E pra exemplificar o que pode ter em comum, esse ano o problema 6 do Nível
U também estava na prova do nível 3 (não sei o número do problema)

On Sat, 19 Jan 2019 at 09:42, Anderson Torres 
wrote:

> Em sáb, 12 de jan de 2019 às 16:41, Luiz Kv
>  escreveu:
> >
> > Olá, boa tarde, tudo bom ?
> >
> > Gostaria de saber quais conteúdos caem na OBMU diferentes do nível 3 da
> OBM
>
> Acho que, bem, tudo! Dificilmente tem algo que se aproveite
> diretamente. No máximo Combinatória, já vi uma questão de Combinatória
> que poderia ser facilmente dada para o Nível 3.
>
> > Quais as melhores fontes para estudar ?
>
> As provas antigas, as provas da IMC e provas de Universitárias ao
> redor do mundo, como a Putnam dos EUA.
>
> E algumas revistas internacionais :)
>
> > Existe idade máxima para a OBMU ?
>
> Não exatamente, Acho que só cobram ser estudante universitário.
> Algumas competições internacionais cobram idade, porém.
>
>
> >
> > Obrigado, até mais :)
> >
> > --
> > Esta mensagem foi verificada pelo sistema de antivírus e
> > acredita-se estar livre de perigo.
>
> --
> Esta mensagem foi verificada pelo sistema de antivírus e
>  acredita-se estar livre de perigo.
>
>
> =
> Instru�ões para entrar na lista, sair da lista e usar a lista em
> http://www.mat.puc-rio.br/~obmlistas/obm-l.html
> =
>

-- 
Esta mensagem foi verificada pelo sistema de antiv�rus e
 acredita-se estar livre de perigo.



[obm-l] Re: [obm-l] Re: [obm-l] Re: [obm-l] Dúvida

2019-05-24 Thread Anderson Torres
Em dom, 19 de mai de 2019 às 13:24, Pedro José 
escreveu:

> Bom dia!
> Anderson,
> obrigado. Porém faltou-me saber se os entendimentos anteriores estão
> corretos.
>

O texto não tinha nenhum glossário para ajudar, ou uma referência do
gênero? Alguns bons livros de Teoria dos Números, em especial os grossões
como o do Hua Loo-Keng, costumam dedicar a primeira folha a convenções.

Eu até suspeito que esses teus entendimentos estejam corretos - dado que
são bem parecidos com outros já consagrados pelo uso - mas não bato o
martelo porque não tenho o contexto.



> Grato,
> PJMS
>
> Em sáb, 18 de mai de 2019 13:27, Anderson Torres <
> torres.anderson...@gmail.com escreveu:
>
>>
>>
>> Em sex, 17 de mai de 2019 às 10:49, Pedro José 
>> escreveu:
>>
>>> Bom dia!
>>>
>>> Tenho uma dúvida sobre os simbolismos, que aparecem recorrentemente, em
>>> artigos sobre teoria dos números, mas que não encontro a definição :
>>> Z[i]/(α) - Entendi como o conjunto das classes de equivalências mod α
>>> em Z{i}
>>> Z[i]/(α)* - Entendi como as classes de equivalência mod α em Z[i], que
>>> são inversíveis.
>>>
>>> Tentando compreender uma demonstração de que todos os números que não
>>> podem ser escritos da forma 4^k(8m+7) com k,m>=0, aceitam ser escritos como
>>> a soma de três parcelas, todas quadrados, me deparei com  [image:
>>> image.png] . O que significa?
>>>
>>
>> Algo como números da forma a+b*sqrt(m) onde a e b são racionais.
>>
>>
>>
>>>
>>> Os outros entendimentos estão corretos?
>>>
>>> Saudações,
>>> PJMS
>>>
>>>
>>> --
>>> Esta mensagem foi verificada pelo sistema de antivírus e
>>> acredita-se estar livre de perigo.
>>
>>
>> --
>> Esta mensagem foi verificada pelo sistema de antivírus e
>> acredita-se estar livre de perigo.
>
>
> --
> Esta mensagem foi verificada pelo sistema de antivírus e
> acredita-se estar livre de perigo.

-- 
Esta mensagem foi verificada pelo sistema de antiv�rus e
 acredita-se estar livre de perigo.



[obm-l] Re: [obm-l] Re: [obm-l] Dúvida basica equação polar

2019-09-02 Thread Gabriel Lopes
Caro Ralf, obrigado pela resposta.Para mim ficou confuso pq pensei que a
parametrização do círculo se daria colocando como referencia o novo centro
do mesmo. Quando penso em circulos diferentes , por exemplo residindo em
apenas um quadrante tenho dificuldade de imaginar varrendo todos os pontos
. Vou refletir sobre esses casos pois parecem ser obtidos como vc disse de
fato.

Att.Gabriel

Em Seg, 2 de set de 2019 18:04, Ralph Teixeira  escreveu:

> Bom, vale a pena fazer uma figura primeiro... Fez? Note como este circulo
> estah nos primeiro e quarto quadrantes apenas.
>
> Entao suponho que voce fez as contas e descobriu que r=2cos(teta). No
> quarto quadrante vale -pi/2 onde pi/2 terceiro quadrante). Por isso que -pi/2
> Para ser mas exato, o que aconteceria na equacao r=2cos(teta) para teta
> entre pi/2 e 3pi/2... Tipo, experimente pensar em teta=pi para fazer um
> exemplo. Jogando na equacao, ficaria r=2cos(pi)=-2?!?
>
> Aqui ha duas opcoes:
> a) Alguns livros vao insistir que r>=0 sempre. Neste caso, fica claro que
> pi/2 cosseno fica negativo.
> b) Alguns outros livros sao mais "liberais" e permitem r<0 -- a
> interpretacao seria que quando r eh negativo voce anda na reta que forma
> angulo teta com o eixo x NO SENTIDO OPOSTO. Por exemplo, r=-2 e teta=pi eh
> de fato o ponto (2,0) (voce anda na direcao NEGATIVA do eixo x, mas voce
> anda -2, entao acaba andando para a DIREITA duas unidades). Neste caso,
> colocar teta=pi dah um ponto no circulo sim senhor! Mas, mesmo assim, eu
> usaria apenas -pi/2 ponto (2,0) JAH APARECEU com teta=0, e nao vejo porque conta-lo duas vezes
> (e, dependendo da aplicacao, voce NAO QUER contar cada ponto duas vezes).
>
> Abraco, Ralph.
>
> On Mon, Sep 2, 2019 at 4:55 PM Gabriel Lopes  wrote:
>
>> Boa tarde, tenho uma duvida básica da representação em equação polar do
>> círculo  (x-1)^2 +y^2= 1.
>>
>> Pq os intervalo de teta é de -pi/2 a pi/2 e nao de 0 a 2pi?
>>
>> --
>> Esta mensagem foi verificada pelo sistema de antivírus e
>> acredita-se estar livre de perigo.
>
>
> --
> Esta mensagem foi verificada pelo sistema de antivírus e
> acredita-se estar livre de perigo.

-- 
Esta mensagem foi verificada pelo sistema de antiv�rus e
 acredita-se estar livre de perigo.



[obm-l] Re: [obm-l] Re: [obm-l] Re: [obm-l] Dúvida

2019-12-05 Thread Bernardo Freitas Paulo da Costa
Oi Pedro e Pedro, e demais colegas da OBM-L

Eu também nunca lera a definição de elipses através da razão entre as
distâncias.  Achei interessante, porque talvez permita "interpolar"
entre elipses, parábolas e hipérboles.  Mas até hoje, todas as
definições que eu vira de elipses (inclusive a da soma das distâncias)
incluíram círculos.  Mas, como talvez tenha desejado indicar o Pedro
Fonini, o importante é *para que serve a definição*.  No caso das
elipses, é muitas vezes importante incluir os círculos com elas, por
exemplo para o teorema de álgebra linear que ele citou.  Talvez o caso
da definição por razão das distâncias indique um outro caminho, mas aí
minha impressão é que o caso seria que a reta diretriz está no
infinito para os círculos (o que, mais uma vez, reforça a unidade das
cônicas no plano projetivo).

Acho que o mais comum, *hoje em dia*, é definir elipses de forma a
incluir os círculos.  Talvez isto invalide a definição que você deu
via razões, que passa a ser um teorema apenas para as elipses com dois
focos distintos.  Mas, por outro lado, permite generalizar de forma
mais natural outros teoremas para os quais a inclusão dos círculos
como elipses simplifique o enunciado.  Talvez você prefira a definição
por razões, mas voltando às origens das cônicas, onde a "classe" é
determinada pela posição relativa do plano secante com relação ao cone
gerador, acredito que a inclusão dos círculos junto com as elipses
seja totalmente razoável.

Cônicas suaves me parecem um assunto avançado; esta terminologia mesmo
já faz pensar em funções (infinitamente) diferenciáveis, etc, típicas
o ensino superior.  Do ponto de vista da geometria algébrica, uma
cõnica é definida como zeros de um polinômio P(x,y) de grau dois,
então esta "definição" não pode servir para separar quem seja suave e
quem não seja.  Se há uma diferença entre um círculo e duas retas que
se intersectam, não é pela regularidade da função que os define
implicitamente: num caso é x^2 + y^2 = 1, no outro, xy = 0.  O que
acaba servindo é a definição de "variedade suave".  E daí eu estaria
puxando mais ainda para temas universitários...

On Wed, Dec 4, 2019 at 9:33 PM Pedro José  wrote:
>
> Boa noite!
> As retas são cônicas degeneradas. Mas são cônicas.
> Definição de cônica :   Dada duas retas g,l concorrentes (cuja interseção é 
> {V} no |R3 que não sejam perpendiculares e um plano Pi. A interseção desse 
> plano com o cone K, reto de vértice V e eixo l , obtido pela rotação da reta 
> g ao redor do ponto V é uma cônica. Podemos ter uma reta, duas retas ou um 
> ponto como cônicas degeneradas.
> Você poderia até ter mencionado o conjunto vazio que não é uma cônica. 
> x^2+y^2=-1.
> Mas na verdade, eu não me expressei com rigor, o que queria dizer é que se 
> escrevermos a função quadrática F(x,y)= 0, que represente a cônica 
> (degenerada ou não) F(x,y) é suave? Ou as cônicas suaves devem ser não 
> degeneradas apenas?
> Outrossim, discordo do seu argumento "...geralmente é mais útil que as 
> definições dos objetos importantes não excluam os casos particulares.."
> Geralmente não é o balizador e sim a definição.
> 1 não é primo. Pois define-se que um primo deve ter dois divisores positivos 
> e 1 só possui um. Poderia argumentar, na sua linha, os dois divisores 
> coincidentes (os que afirmam é divisível por si e pela unidade)
> O quadrado por definição está claro que é retângulo.
> A definição da elipse é de que a soma das distâncias a dois pontos fixos (e 
> não um) é constante. Aí tem a forçação de se considerar dois como um só. Não 
> existe dois pontos coincidentes. Se são dois são distintos. Podemos 
> representar algo de várias maneiras mas se são iguais é só um, representado 
> de várias maneiras. Qual o cardinal do conjunto de focos de uma elipse, no 
> caso de você aceitar a elipse com um único foco?
> Como é a prova que só existe um vazio. Por hipótese há mais de um vazio, 
> vazio1 e vazio2 e no fim chega-se a conclusão que vazio1 = vazio2 e portanto 
> absurdo.Ora, podemos ter vazios coincidentes.
> Amigo, você afirma: "Nunca vi ninguém definir elipse de uma forma que exclua 
> os círculos."
> Você nem se deu ao trabalho de ler a minha nota, antes de comentar, ou então 
> me corrija se o círculo atende à:
> Lugar geométrico do plano em que a razão entre a distância de um ponto ao 
> foco direito e a distância entre esse ponto e uma reta (diretriz direita) é 
> constante e menor que 1 e igual a excentricidade da cônica.
> Como a excentricidade da circunferência é zero, teríamos que ter um ponto 
> fixo em que a distância de cada ponto da circunferência até esse ponto fosse 
> zero. E se na definição tem foco direito está implícito que há um esquerdo. 
> Vale a definição para foco esquerdo. Só atenderia se considerarmos o ponto 
> como uma circunferência de raio zero. E só para esse caso e ainda aceitarmos 
> que quando há só um foco ele tanto é direito quanto esquerdo. Grato pelos 
> comentários. Mas as dúvida persistem.
>
> Saudações

[obm-l] Re: [obm-l] Re: [obm-l] Dúvida de Combinatoria e PUTNAM

2002-08-01 Thread Eduardo Casagrande Stabel

From: "Paulo Santa Rita" <[EMAIL PROTECTED]>
>
> Ola Leonardo,
> Tudo Legal ?
>
> O problema esta mal formulado... Para ver isso, suponha N=3. Pelo
enunciado
> do problema deveriamos formar N/2=1.5 grupos ! Um absurdo, pois neste
> contexto nao tem sentido falar em FRACAO DE GRUPO !

Paulo,

a correção mais natural é a seguinte:

Tendo n-casais (marido e mulher) de quantas maneiras diferentes pode-se
formar n grupos de tal forma q em que cada grupo contenha 2 pessoas,ou
seja,não importa a ordem.Uma maneira seria
[(H1,M1),(H2,M2),(H3,M3)...,(Hn,Mn)].

Acho que esse problema nada tem a ver com permutações caóticas.

Um detalhe.
Se quisermos separar as pessoas em pares (sem que sejam casais), há
2n!/[n!*2^n].
Se quisermos separar as pessoas em casais, há n! maneiras.
Está certo isso? Você concorda?

Duda.


=
Instruções para entrar na lista, sair da lista e usar a lista em
http://www.mat.puc-rio.br/~nicolau/olimp/obm-l.html
O administrador desta lista é <[EMAIL PROTECTED]>
=



[obm-l] Re:[obm-l] Re: [obm-l] Dúvida de Combinatoria e PUTNAM

2002-08-02 Thread rafaelc.l


  Ei Santa Rita, vc disse umas coisas que me interessou
muito na última mensagem. Onde consigo este livro( e
outros) do morgado? é que estou me preparando para o IME,
ultilizo o IEZZI, mas não são todas questões que consigo
resolver, como a das estradas que caiu no ano passado por
exemplo.
Outra coisa é esse texto de preparação para olimpíadas
universitárias. Como posso obtê-lo?



obrigado


__
AcessoBOL, só R$ 9,90! O menor preço do mercado!
Assine já! http://www.bol.com.br/acessobol


Ola Leonardo,
Tudo Legal ?

O problema esta mal formulado... Para ver isso, suponha N=3. Pelo enunciado 
do problema deveriamos formar N/2=1.5 grupos ! Um absurdo, pois neste 
contexto nao tem sentido falar em FRACAO DE GRUPO !

Todavia, da sua segunda pergunta ( O que Permutacao Caotica ) e possivel 
deduzir o que voce quer ...

Tendo N-casais ( marido e mulher )de quantas maneiras diferentes pode-se 
formar N grupos, cada grupo com duas pessoas, de formar que em nenhum dos 
grupos um marido fique com sua mulher ?

A solucao deste problema e uma das aplicacoes das permutacoes caoticas.

Considere 3 letras, ABC, e suponha que esta disposicao inicial seja a 
posicao natural ( ou referencial ) de cada uma das letras : TODA PERMUTACAO 
DESTAS TRES LETRAS NA QUAL NENHUMA LETRA OCUPA SUA POSICAO NATURAL E DITA 
UMA PERMUTACAO CAOTICA.

Assim, sao exemplos de permutacoes caoticas :
BCA, CAB

Nao sao exemplos de permutacoes caoticas :
ACB ... ( pois a letra "A" esta na posicao natural )
CBA ... ( pois a letra "B" esta na posicao natural )

E importante perceber que :

O conceito de permutacao caotica depende de uma permutacao considerada 
natural ou referencial, em relacao a qual decidimos se uma outra permutacao 
dada e caotica ou nao.

Quem primeiro abordou as permutacoes caoticas foi Nicolau Bernoulli, que 
propos o problema a Euler. Este ultimo se interessou pela questao e a 
resolveu de uma outra forma, diferente da de Nicolau. Se chamaramos de PC(N) 
o numero de permutacoes caoticas de N objetos, dois a dois distintos, e 
possivel provar que :

PC(N) = N!(1/2!  -   1/3!  + ... +-  1/N!)

A titulo de exemplo, com 3 letras nos podemos formar :

PC(3) = 3!(1/2! - 1/3!) = 3 - 1 = 2 permutacoes caoticas

Bom, agora que voce sabe o que e permutacao caotica e conhece a formula com 
que as calculamos, voce pode resolver a reformulacao do seu problema. 
Imagine a disposicao :

H1,H2,H3,...,Hn
M1,M2,M3,...,Mn

Ou seja, cada homem sobre a sua mulher, o que vamos considerar a disposicao 
natural ( e legal ! ). Qualquer permutacao caotica dos homens fara com que 
nao existe ao menos um grupo onde um homem estara sobre a sua mulher, ou 
seja, a formulacao correta de seu problema se resolve calculando-se o numero 
de permutacoes caoticas.

Finalmente, e importante registrar que o nosso muito estimado Prof Morgado, 
membro desta lista, publicou um livro de Analise Combinatoria onde ele 
aborda este tema e muitos outros mais, tais como os Lemas de Kaplanski. Em 
minha opiniao, este livro e o que ha de melhor sobre o tema, aqui no Brasil. 
E acredito que todo estudante que deseja entender corretamente este tema 
basta estudar por este livro.

Um Abraco a Todos
Paulo Santa Rita
5,1515,010802

Em Tempo : Acabo de receber uma mensagem do Prof Jonh Scholes no qual ele me 
autoriza traduzir e divulgar todo o material PUTNAM. Vou fazer as traducoes 
e disponibilizar para todos, como material de treinamento e incentivo para 
as olimpiadas universitarias.









>From: "leonardo mattos" <[EMAIL PROTECTED]>
>Reply-To: [EMAIL PROTECTED]
>To: [EMAIL PROTECTED]
>Subject: [obm-l] Dúvida de Combinatoria
>Date: Thu, 01 Aug 2002 03:42:09 +
>
>
>
>Tendo n-casais(marido e mulher)de quantas maneiras diferentes pode-se 
>formar n/2 grupos de tal forma q em que cada grupo contenha 2 pessoas,ou 
>seja,não importa a ordem.Uma maneira seria 
>[(H1,M1),(H2,M2),(H3,M3)...,(Hn,Mn)].
>
>O que é permutação caótica?
>
>
>
>
>
>_
>Converse com seus amigos online, faça o download grátis do MSN Messenger: 
>http://messenger.msn.com.br
>
>=
>Instruções para entrar na lista, sair da lista e usar a lista em
>http://www.mat.puc-rio.br/~nicolau/olimp/obm-l.html
>O administrador desta lista é <[EMAIL PROTECTED]>
>=




_
Converse com seus amigos online, faça o download grátis do MSN Messenger: 
http://messenger.msn.com.br

=
Instruções para entrar na lista, sair da lista e usar a lista em
http://www.mat.puc-rio.br/~ni

[obm-l] Re: [obm-l] RE: [obm-l] RE: [obm-l] Dúvida Indução

2012-05-17 Thread Ralph Teixeira
Cuidado: ao passar de n=k para n=k+1 no Passo de Inducao... o ultimo
termo "era" 3n-1, agora eh 3(n+1)-1=3n+2 -- nao eh questao de "somar
um no termo", eh "trocar n por n+1".

Abraco,
 Ralph

2012/5/17 Thiago Bersch :
> Então eu estava tentando fazer mas parava no mesmo ponto, fazia
> 2+5+8+...+(3n-1)+[(3n-1)+1], chegando aí eu me perco
>
> 
> From: joao_maldona...@hotmail.com
> To: obm-l@mat.puc-rio.br
> Subject: [obm-l] RE: [obm-l] Dúvida Indução
> Date: Mon, 14 May 2012 15:24:47 -0300
>
> Vamos dizer que para n respeite a formula
> Logo 2+4+6+...+2n=n.(n+1)
> Somando 2n+2
> 2+4+6+...+(2n+2=n(n+1)+2n+2=(n+1)(n+2) que respeita a formula
> Logo se vale para n, vale para n+1
> Como vale para 1, vale para 2, e  entao para 3, 4, 5...
> Vale para qualquer natural
>
> Tente fazer o segundo agora
> []s Joao
>
> 
> From: thiago_t...@hotmail.com
> To: obm-l@mat.puc-rio.br
> Subject: [obm-l] Dúvida Indução
> Date: Mon, 14 May 2012 01:09:39 -0300
>
>  2 + 4 + . . . + 2n.
>  2 + 5 + 8 + . . . + (3n-1).
> Bem eu sei que o primeiro irá dar n(n+1) e o segundo n(3n+1)/2
> O que em si eu não entendi o resultado
> O primeiro eu tentei fazer assim:
> 2+4...+2n
> +
> n+2n+(2n+1),  e fiquei parado nisso e o segunda também, gostaria de uma
> explicação passo-a-passo pois não entendo.

=
Instruções para entrar na lista, sair da lista e usar a lista em
http://www.mat.puc-rio.br/~obmlistas/obm-l.html
=


[obm-l] Re: [obm-l] Re:[obm-l] RE: [obm-l] Dúvida (urgente)

2003-11-26 Thread Rogerio Ponce
Olá Osvaldo,
para encontrar a interseção de f() com uma reta , você está precisando 
calcular sucessivas interseções da mesma função f() com uma 
circunferência...
É original, mas acho que não faz muito sentido, certo ?

Abraços,
Rogério.




From: "Osvaldo" <[EMAIL PROTECTED]>

Está certo, a circunferencia tem raio f(X0).

Estou tentando desenvolvendo um metodo numerico para
calcular as raizes de uma funcao continua de modo que
necessite de MUITO MENOS interacoes com relacao as
necessarias usando o famoso metodo de newton (usando
derivacoes). Para isto pego um pto. da funcao e traco um
circunferencia de raio f(Xo), tocando no eixo dos X.
Encontrando as coodenadas de todas as  interseccoes de f
com a circunferencias pego a de abssissa menor e traco
outra circunferencia, agora com centro em (x1,f(x1))
tocando o eixo dos x novamente e assim sucessivamente,
porem tenho que encontrar x1=F(xo,f(xo)), onde F é a
funcao a ser determinada, e x1 NAO pode estar em funcao
de f(x1).
Fiz inumeros testes para este processo, funcionou!!!,
mais nao consigo provar analiticamente.
Na sua resposta

> f(x1)= f(x0) + sqrt(f(x0)^2 - (x1-x0)^2) ou
> f(x1)= f(x0) - sqrt(f(x0)^2 - (x1-x0)^2
x1 esta em funcao de f(x1), que nao é a funcao que
queria.
Agradeco sua ajuda.









> Osvaldo,
>
> Nao sei se entendi direito, me corriga se eu estiver er
rado.
>
> Considere dois pontos P1 e P2 tais que:
>
> P1: (X0,F(X0))  -
  Centro da Circunferencia (Why ??? Faca um desenho)
> P2: (X1,F(X1))  -
  Ponto de intersecao de f com a circunferencia.
>
> Note, a circunferencia tem que ter centro P1 e o raio d
ela tem que ser f(X0)
> segundo a descricao do problema
(Faca um desenho). Alem disso, se f
> intersecta a circunferencia em P2, entao temos que ter:
>
> |P1-P2| = Raio da circunferencia = f(X0)
>
> (x1-x0)^2 + (f(x1)-f(x0))^2 = [f(x0)] ^2
>
> [f(x1) - f(x0)]^2 = f(x0)^2 - (x1-x0)^2
>
> f(x1)= f(x0) + sqrt(f(x0)^2 - (x1-x0)^2) ou
> f(x1)= f(x0) - sqrt(f(x0)^2 - (x1-x0)^2).
>
>
> Regards,
>
> Leandro
> Los Angeles, CA.
>
>
> -Original Message-
> From: [EMAIL PROTECTED] [mailto:owner-obm-
[EMAIL PROTECTED] On
> Behalf Of Osvaldo
> Sent: Wednesday, November 26, 2003 2:06 PM
> To: lista de discussao de matematica
> Subject: [obm-l] Dúvida (urgente)
>
> Olá pessoal, tenho um problema que tenho tentado
> solucionar mas tá dificil. Muitos tem me dito que é
> impossível, mas eu insisto.
>
> O problema é o seguinte:
>
> "Seja f uma função contínua em seu domínio. Sabe-
se que
> ela passa pelo centro de uma circunferência que é
> tangente ao eixo dos X na abscissa Xo. A função não é
> necessariamente bijetora e seja X1 a abscissa de uma da
s
> intersecções de f com a circunferencia em questão.
>
> O problema é determinar f(X1) EM TERMOS DE Xo E/OU F
> (Xo)." - Paradigma de Labaki-Osvaldo
>
>
>
>
> Eu substitui X1 na equação da circunferência e dirivei-
a
> com relação a X1 duas vezes consecutivas obtendo, assim
,
> uma expressão para a derivada segunda em X1 da função
> dada em termos de Xo e de f
(Xo). Daí teria que encontrar
> as raízes desta equação diferenciavél, mais não consegu
i
> encontrar f(0) nem f'(0), o que complica mais.
>
>
>
> Atenciosamente,
>
> Osvaldo Mello Sponquiado FEIS - UNESP
> Usuário em GNU/Linux
> Futuro Engenheiro Eletricista
>
>
>
>
> ___
___
> Acabe com aquelas janelinhas que pulam na sua tela.
> AntiPop-up UOL - É grátis!
> http://antipopup.uol.com.br/
>
>
>
> ===
==
> Instruções para entrar na lista, sair da lista e usar a
 lista em
> http://www.mat.puc-rio.br/~nicolau/olimp/obm-l.html
> ===
==
>
> ===
==
> Instruções para entrar na lista, sair da lista e usar a
 lista em
> http://www.mat.puc-rio.br/~nicolau/olimp/obm-l.html
> ===
==
>
Atenciosamente,

Osvaldo Mello Sponquiado FEIS - UNESP
Usuário em GNU/Linux




__
Acabe com aquelas janelinhas que pulam na sua tela.
AntiPop-up UOL - É grátis!
http://antipopup.uol.com.br/


=
Instruções para entrar na lista, sair da lista e usar a lista em
http://www.mat.puc-rio.br/~nicolau/olimp/obm-l.html
=
_
MSN Messenger: converse com os seus amigos online.  
http://messenger.msn.com.br

=
Instruções para entrar na lista, sair da lista e usar a lista em
http://www.mat.puc-rio.br/~nicolau/olimp/obm-l.html
=


[obm-l] RE: [obm-l] Re: [obm-l] RE: [obm-l] Dúvida Simples!!!

2004-01-13 Thread Paulo Santa Rita
Ola Pessoal,

Eu cometi um erro de digitacao e uma de minhas respostas e, em funcao disso, 
vou falar um pouco mais sobre estas coisas, bastante conhecidas :

Se Y=F(X) e uma funcao e queremos mostrar que ela e injetiva, nos fazemos :
x1 # x2  =>  F(x1) # F(x2)   - aqui, o simbolo "#", siginifica : e diferente 
de.

Pode-se provar isso negando a tese, o que da : F(x1)=F(x2)  =>  x1=x2.
No caso da funcao Y=2x-5, nos podiamos por :
x1=x2  <=>  2*x1=2*x2  <=> 2*x1 - 5 = 2*x2 - 5 <=> F(x1)=F(x2)
A implicacao X1=x2 => F(x1)=F(x2) e desnecessaria, pois, sendo F uma funcao, 
um elemento qualquer do dominio nao pode ter mais de uma imagem no 
contra-dominio. Todavia, quando nos escrevemos, pensamos em que esta lendo e 
pode ser que a dupla implicacao torne a sequencia de raciocinios mais clara, 
sobretudo pra principiantes e foi justamente o que eu queria fazer, mas 
coloquei => onde deveria ter colocado <=>.

Assim, e certo fazer :
x1=x2  <=>  2*x1=2*x2  <=> 2*x1 - 5 = 2*x2 - 5 <=> F(x1)=F(x2)
Como seria certo fazer :
F(x1)=F(x2) => 2*X1-5=2*x2 - 5 => 2*x1=2*x2 => x1 = x2
Quando nos usamos isso, estamos, em verdade, usando o fato :

A => B  <=> ~B => ~A
Ou seja : Provar : x1 # X2 => F(x1) # F(x2)  - Funcao injetiva
E equivalente a provar : F(x1) = F(x2) => X1 = X2.
Um Abraco a Todos
Paulo Santa Rita
3,0956,130104
From: "Paulo Santa Rita" <[EMAIL PROTECTED]>
Reply-To: [EMAIL PROTECTED]
To: [EMAIL PROTECTED]
Subject: [obm-l] Re: [obm-l] RE: [obm-l] Dúvida Simples!!!
Date: Tue, 13 Jan 2004 11:16:36 +
MIME-Version: 1.0
X-Originating-IP: [200.142.58.18]
X-Originating-Email: [EMAIL PROTECTED]
X-Sender: [EMAIL PROTECTED]
Received: from mc1-f30.hotmail.com ([64.4.50.37]) by mc1-s2.hotmail.com 
with Microsoft SMTPSVC(5.0.2195.6824); Tue, 13 Jan 2004 03:25:55 -0800
Received: from saci.mat.puc-rio.br ([139.82.27.51]) by mc1-f30.hotmail.com 
with Microsoft SMTPSVC(5.0.2195.6824); Tue, 13 Jan 2004 03:25:01 -0800
Received: from saci.mat.puc-rio.br (localhost [127.0.0.1])by 
saci.mat.puc-rio.br (8.12.8/8.12.8) with ESMTP id i0DBG7xF025951for 
<[EMAIL PROTECTED]>; Tue, 13 Jan 2004 09:16:07 -0200
Received: (from [EMAIL PROTECTED])by saci.mat.puc-rio.br 
(8.12.8/8.12.8/Submit) id i0DBG7Jp025949for obm-l-MTTP; Tue, 13 Jan 2004 
09:16:07 -0200
Received: from hotmail.com (sea2-f29.sea2.hotmail.com [207.68.165.29])by 
saci.mat.puc-rio.br (8.12.8/8.12.8) with ESMTP id i0DBG5xF025946for 
<[EMAIL PROTECTED]>; Tue, 13 Jan 2004 09:16:06 -0200
Received: from mail pickup service by hotmail.com with Microsoft SMTPSVC; 
Tue, 13 Jan 2004 03:16:36 -0800
Received: from 200.142.58.18 by sea2fd.sea2.hotmail.msn.com with HTTP;Tue, 
13 Jan 2004 11:16:36 GMT
X-Message-Info: o8IIVuzO8A0xt1jbTtkAABvddtGJF13ACLo6v1RyJA0=
Message-ID: <[EMAIL PROTECTED]>
X-OriginalArrivalTime: 13 Jan 2004 11:16:36.0631 (UTC) 
FILETIME=[B5614A70:01C3D9C6]
Precedence: bulk
Return-Path: [EMAIL PROTECTED]

Ola Fabio e demais colegas
desta lista ... OBM-L,
Nao precisa inverter, basta usar o sinal <=> no lugar de =>, que e o que eu 
queria fazer.

From: Fábio Dias Moreira <[EMAIL PROTECTED]>
Reply-To: [EMAIL PROTECTED]
> 1) f(x)=2x-5 ( Dominio : R, Contra-Dominio : R )
>
> Suponha que x1=x2. Entao :
> 2*x1 = 2*x2   =>   2*x1 - 5 = 2*x2 - 5   =>   f(x1)=f(x2)
> Portanto : x1=x2 => f(x1)=f(x2) => funcao injetiva.
> [...]
A implicação x=y => f(x)=f(y) vale para qualquer função trivialmente.  A
implicação que prova a injetividade é f(x)=f(y) => x=y (ou, naturalmente, 
a
sua contrapositiva).

De qualquer forma, basta inverter a cadeia de implicações acima.

[]s,

- --
Fábio "ctg \pi" Dias Moreira
-BEGIN PGP SIGNATURE-
Version: GnuPG v1.2.3 (GNU/Linux)
iD8DBQFAAvaualOQFrvzGQoRAudzAJwKZmwKUbGWJepRhwJbXgzpRl+lhQCcDHUe
lmPqaHD0ss5v6t63HcZqlVE=
=aZJ1
-END PGP SIGNATURE-
=
Instruções para entrar na lista, sair da lista e usar a lista em
http://www.mat.puc-rio.br/~nicolau/olimp/obm-l.html
=
_
MSN Hotmail, o maior webmail do Brasil.  http://www.hotmail.com
=
Instruções para entrar na lista, sair da lista e usar a lista em
http://www.mat.puc-rio.br/~nicolau/olimp/obm-l.html
=
_
MSN Messenger: converse com os seus amigos online.  
http://messenger.msn.com.br

=
Instruções para entrar na lista, sair da lista e usar a lista em
http://www.mat.puc-rio.br/~nicolau/olimp/obm-l.html
=


[obm-l] Re: [obm-l] Re: [obm-l] Re: [obm-l] Dúvida persistente!!!

2004-04-13 Thread Rafael
Auggy,

Independentemente das contas, a criatividade na construção dos triângulos é
magnífica. Lendo o link, vi que o Cláudio já havia pensado no cálculo da
área por integral e teve uma idéia muito melhor em relação à posição dos
eixos, com origem em B em vez de A.

Enfim, apesar de trabalhoso, é um problema bonito.


Cláudio,

Parabéns por ambas as soluções!


Abraços,

Rafael de A. Sampaio





- Original Message -
From: "Qwert Smith" <[EMAIL PROTECTED]>
To: <[EMAIL PROTECTED]>
Sent: Tuesday, April 13, 2004 5:41 PM
Subject: RE: [obm-l] Re: [obm-l] Re: [obm-l] Dúvida persistente!!!


Sai na geometria mas da umas contas chatas.

na primeira figura:
---
(area em amarelo)  = (area do circulo menor) - 2*(area em verde)

(area em vermelho) = (area do quadrado) -
1/4*{(area do circulo maior) + [(area do quadrado)-(area circulo menor)]} -
(area em verde)
---

na segunda figura:
---
(area em laranja) = (setor circular PBQ) - 2*(triangulo PBO - area em azul)
(area em verde) = (setor circular POQ) - (area em laranja)
--

Como todos os lados do triangulo sao conhecidos (em funcao do lado do
quadrado).  Agulos e areas sao questao de braco.

A descricao da figura 2 vc encontra no link

http://www.mat.puc-rio.br/~nicolau/olimp/obm-l.200310/msg00574.html

em uma menssagem do grande Claudio BUffara que ainda teve paciencia
montruosa de me explicar em off o problema.

Em meu email anterior eu tinha feito confusao e atribuido a mensagem do link
acima a outro fera, o Paulo Santa Rita que mandou uma mensagem sobre "lua
algebrica", que tb vale a pena conferir.
E' muito genio pra keep track.

Valeu Super Buffara!

[]s,
Auggy



=
Instruções para entrar na lista, sair da lista e usar a lista em
http://www.mat.puc-rio.br/~nicolau/olimp/obm-l.html
=


[obm-l] Re:[obm-l] RE: [obm-l] Re:[obm-l] dúvida chara!

2004-05-23 Thread Osvaldo
Desculpe-me se fui parcial Dr., porém equivoquei-me ao 
ler o enunciado da questão. Eu apenas fiz os calculos 
para os números inteiros e não naturais, ou seja, 
inclui algumas possibilidades a mais. 
Obrigado pela observação!





> Olá colegas da lista,
> 
>   Apesar da resolução apresentada pelo Osvaldo 
ter seguido um possível
> raciocínio correto para resolver esta questão, a 
análise dele está
> incompleta porque omite alguns passos muito 
importantes, o que pode nos
> levar a encontrar soluções inválidas. Neste problema 
especificamente, a
> resposta encontrada está correta, porém, se 
modificarmos o valor da
> diferença de quadrados de 27 para outro valor, então 
a resolução dele pode
> nos levar a resultados errados.
> 
>   A análise que eu apresento a seguir 
corresponde a uma crítica de
> caráter construtivo com relação à resolução 
apresentada pelo Osvaldo. O
> objetivo desta análise não é depreciar a resolução 
do Osvaldo, mas sim de
> mostrar que é necessário sermos rigorosos nas 
resoluções de problemas de
> Matemática para não chegarmos a resultados 
incorretos. Muitas vezes podemos
> encontrar uma resposta correta para uma questão 
resolvendo-a de maneira
> errada.
> 
>   Na resolução apresentada abaixo, considere 
que "=>" significa
> "implica" e ">=" significa "maior ou igual a".
> 
> 
> QUESTÃO ORIGINAL:
> 
> "A diferença entre os quadrados de dois números 
naturais é 27. UM dos
> possíveis valores do quadrado da soma desses dois 
números:
> a)529
> b)625
> c)729
> d)841"
> 
> 
> RESOLUÇÃO POSSÍVEL:
> 
> Sejam x e y os dois números naturais, então devemos 
ter:
> x^2 - y^2 = 27 <=> (x + y)(x - y) = 27
> 
> Adotando a = x + y e b = x - y, teremos:
> a.b = 27 (i) (Observe que o produto de a e b é 
positivo)
> Resolvendo o sistema de equações nas variáveis x e 
y, podemos encontrar x e
> y em função de a e b:
> a + b = (x + y) + (x - y) <=> a + b = 2x <=> x = (a 
+ b)/2 (ii)
> a - b = (x + y) - (x - y) <=> a - b = 2y <=> y = (a -
 b)/2 (iii)
> 
> Como x e y são naturais, então x >= 0 e y >= 0. 
Portanto:
> x + y >= 0 + 0 => a >= 0. De acordo com a igualdade 
(i), a não pode ser 0,
> logo a > 0 (iv)
> Como a.b > 0 (i) e a > 0 (iv), então b > 0 (v)
> y >= 0 => -y <= 0 => y >= 0 e 0 >= -y => y >= -y => 
x + y >= x - y =>
> a >= b (vi)
> Por (v) e (vi), concluímos que: a >= b > 0 (vii)
> 
> Sendo assim, devemos encontrar a e b inteiros tais 
que sejam satisfeitas as
> seguintes condições:
> a.b = 27 (ii)
> a >= b > 0 (vii)
> x = (a + b)/2 (ii) seja um número natural.
> y = (a - b)/2 (iii) seja um número natural.
> 
> Analisando os divisores de 27, podemos concluir que 
existem apenas dois
> pares de valores de a e b que satisfazem as 
condições (ii) e (vii):
> (a = 27 e b = 1) ou (a = 9 e b = 3)
> 
> Para a = 27 e b = 1:
> x = (27 + 1)/2 = 14 é um número natural.
> y = (27 - 1)/2 = 13 é um número natural.
> Portanto, x = 14 e y = 13 é uma solução possível.
> 
> Para a = 9 e b = 3:
> x = (9 + 3)/2 = 6 é um número natural.
> y = (9 - 3)/2 = 3 é um número natural.
> Portanto, x = 6 e y = 3 é uma solução possível.
> 
> Possíveis valores para (x + y)^2:
> (x + y)^2 = (14 + 13)^2 = 27^2 = 729
> (x + y)^2 = (6 + 3)^2 = 9^2 = 81
> 
> Resposta: Alternativa c
> 
> 
> Observação: Pode parecer que os passos apresentados 
para deduzir as
> condições são desnecessários, mas são eles que 
garantem a validade das
> soluções encontradas.
> 
> 
> EXPLICAÇÃO DO MOTIVO DA RESOLUÇÃO APRESENTADA PELO 
OSVALDO SER INCOMPLETA:
> 
> Na resolução são apresentados 4 valores possíveis 
para a e b (a,b):
> {(1,27),(3,9),(9,3),(27,1)}. Porém, (1,27) e (3,9) 
não satisfazem a condição
> (vii): a >= b > 0. Portanto, somente os pares (9,3) 
e (27,1) correspondem a
> possíveis valores para a e b, restando apenas 
verificar se eles produzem
> valores naturais para x e y. Logo, na lista de 
valores apresentados para
> (x+y)^2 = a^2, {1, 9, 81, 729}, não poderia aparecer 
os valores 1 = 1^2 e
> nem 9 = 3^2. Além disto, não há garantia de que 81 = 
9^2 e 729 = 27^2
> correspondem a valores de a e b válidos, pois os 
valores de x e y não são
> calculados para verificar se eles são naturais, como 
foi descrito no
> enunciado do problema. Portanto, os valores de a e b 
encontrados poderiam
> não ser válidos. Neste problema específico, os 
valores de a e b encontrados
> são válidos, logo a resposta encontrada está 
correta. A seguir, eu apresento
> uma variação deste problema que mostra de maneira 
concreta que a resolução
> apresentada pelo Osvaldo pode apresentar resultados 
errados. Para se ter uma
> idéia apenas 1 resultado, dos 6 encontrados, é 
correto!
> 
> 
> 
> QUESTÃO MODIFICADA:
> 
> "A diferença entre os quadrados de dois números 
naturais é 68. UM dos
> possíveis valores do quadrado da soma desses dois 
números:
> a)16
> b)289
> c)1156
> d)4624"
> 
> 
> RESOLUÇÃO DO OSVALDO ALTERADA PARA A VERSÃO 
MODIFICADA DA QUESTÃO:
> 
> sejam x e y tais numeros, dai temos que
> x^2-y^2=68
> 
> (x+y)(x-y)=68
> 
> 
> a=x+y

[obm-l] Re: [obm-l] Re: [obm-l] Dúvida sobre a Obm U

2016-07-25 Thread Raul Alves
Também tenho interesse na OBMU, e a 1ª fase tá chegando.
Se algum professor puder organizar algum material de apoio, seria de grande
ajuda

Em 25 de julho de 2016 10:09, Otávio Araújo 
escreveu:

>
>
> Pois é, se algum professor com experiência em olimpíadas, como o Nicolau
> por exemplo, respondesse minha pergunta seria de grande ajuda
>
> Em 24 de jul de 2016, às 23:25, Israel Meireles Chrisostomo <
> israelmchrisost...@gmail.com> escreveu:
>
> Boa pergunta, eu também tenho interesse em participar da OBM U e gostaria
> de umas dicas
>
> Em 16 de julho de 2016 13:29, Otávio Araújo 
> escreveu:
>
>> Galera, gostaria que vocês me dessem dicas de o que estudar, como
>> estudar e por quais livros e materiais estudar para a prova da Obm nível
>> universitário...
>> Estou muito interessado em participar, mas fico meio confuso por onde
>> estudar...
>> Por favor me ajudem
>> --
>> Esta mensagem foi verificada pelo sistema de antivírus e
>> Â acredita-se estar livre de perigo.
>>
>>
>> =
>> Instruções para entrar na lista, sair da lista e usar a lista em
>> http://www.mat.puc-rio.br/~obmlistas/obm-l.html
>> =
>>
>
>
> --
> Esta mensagem foi verificada pelo sistema de antivírus e
> acredita-se estar livre de perigo.
>
>
> --
> Esta mensagem foi verificada pelo sistema de antivírus e
> acredita-se estar livre de perigo.
>



-- 
*Raul Lima Alves*

*Estagiário na Aton Engenharia*
*Estudante de Engenharia de Computação - UFBA*
*Telefone: (71) 9103-0878*
*Facebook: *https://www.facebook.com/raul.alves.161
*LinkedIn*: https://br.linkedin.com/in/raul-alves-8b090228


-- 
Esta mensagem foi verificada pelo sistema de antiv�rus e
 acredita-se estar livre de perigo.



[obm-l] Re: [obm-l] Re: [obm-l] Dúvida sobre a Obm U

2016-07-25 Thread Carlos Victor
 

Oi Otávio, 

Você já viu a Revista Matemática Universitária da SBM ? 

Em 25/07/2016 10:09, Otávio Araújo escreveu: 

> Pois é, se algum professor com experiência em olimpíadas, como o Nicolau por 
> exemplo, respondesse minha pergunta seria de grande ajuda 
> 
> Em 24 de jul de 2016, às 23:25, Israel Meireles Chrisostomo 
>  escreveu:
> 
> Boa pergunta, eu tambÃ(c)m tenho interesse em participar da OBM U e gostaria 
> de umas dicas 
> 
> Em 16 de julho de 2016 13:29, Otávio Araújo  
> escreveu:
> Galera, gostaria que vocês me dessem dicas de o que estudar, como estudar 
> e por quais livros e materiais estudar para a prova da Obm nível 
> universitário...
> Estou muito interessado em participar, mas fico meio confuso por onde 
> estudar...
> Por favor me ajudem
> --
> Esta mensagem foi verificada pelo sistema de antivírus e
> Â acredita-se estar livre de perigo.
> 
> =
> Instruções para entrar na lista, sair da lista e usar a lista em
> http://www.mat.puc-rio.br/~obmlistas/obm-l.html [1]
> =
> 
> -- 
> Esta mensagem foi verificada pelo sistema de antivírus e 
> acredita-se estar livre de perigo.

-- 
Esta mensagem foi verificada pelo sistema de antivrus e 
acredita-se estar livre de perigo. 

Links:
--
[1] http://www.mat.puc-rio.br/~obmlistas/obm-l.html

-- 
Esta mensagem foi verificada pelo sistema de antiv�rus e
 acredita-se estar livre de perigo.



[obm-l] RES: [obm-l] Re: [obm-l] Re: [obm-l] Dúvida com questão

2008-09-16 Thread Bouskela
E provar que x^2+x+1 é maior do que 0 é muito fácil:
 
1] x^2+x+1 não tem raízes reais. Logo a parábola x^2+x+1 está totalmente
acima OU ("ou" exclusivo) totalmente abaixo do eixo X. Como 0^2+0+1>0 ,
então a parábola está totalmente acima do eixo X .
 
[EMAIL PROTECTED]
[EMAIL PROTECTED]   

 


  _  

De: [EMAIL PROTECTED] [mailto:[EMAIL PROTECTED] Em nome
de João Luís
Enviada em: terça-feira, 16 de setembro de 2008 13:01
Para: obm-l@mat.puc-rio.br
Assunto: [obm-l] Re: [obm-l] Re: [obm-l] Dúvida com questão


Sim, é verdade. Ficou incompleto mesmo.
 
O que acontece é que eu quis enfatizar que, independentemente da
incompletude do enunciado, a bicondicional dada será falsa. E, com isso,
acabei me esquecendo do sinal do termo quadrático.
 
Obrigado pela observação, Bouskela.

- Original Message - 
From: Bouskela <mailto:[EMAIL PROTECTED]>  
To: obm-l@mat.puc-rio.br 
Sent: Tuesday, September 16, 2008 11:46 AM
Subject: Re: [obm-l] Re: [obm-l] Dúvida com questão

João Luís:
 
Sua solução está correta! Entretanto repare que ela (sua solução) está
correta apenas porque "x^2+x+1" é positivo (maior do que "0") para qualquer
que seja "x" real. É, então, necessário, no âmbito da sua solução,
demonstrar isto:
 
x^2+x+1 > 0 para qualquer que seja "x" real.
 
Sds.,
AB 


2008/9/16 João Luís <[EMAIL PROTECTED]>


Bom, faltou um símbolo de desigualdade no primeiro membro (antecedente)
dessa bicondicional: [(x^2+x+1)/(x-2)] 3. Mas de qualquer forma, a
afirmativa é falsa, já que o sinal da desigualdade vai mudar de sentido
conforme o sinal do termo de primeiro grau (x-2): se (x-2) < 0, a segunda
desigualdade será o contrário da primeira; se (x-2) > 0, será igual à
primeira
 
Um abraço,
 

João Luís.

- Original Message - 
From: Robÿe9rio Alves <mailto:[EMAIL PROTECTED]>  
To: obm-l@mat.puc-rio.br 
Sent: Tuesday, September 16, 2008 10:05 AM
Subject: [obm-l] Dúvida com questão


A afirmação: para todo x real, x diferente de 2,[( x^2+x+1) / (x - 2 )]
3  <=> x^2+x+1 > 3.(x - 2 ) é verdaeira ou falsa ? Justifique.
 
Como resolver ???
 

  _  

Novos endereços, o Yahoo! que você conhece. Crie um email novo
<http://br.rd.yahoo.com/mail/taglines/mail/*http://br.new.mail.yahoo.com/add
resses>  com a sua cara @ymail.com <http://ymail.com/>  ou @rocketmail.com
<http://rocketmail.com/> .




-- 
Saudações,
AB
[EMAIL PROTECTED]
[EMAIL PROTECTED]




[obm-l] Re: [obm-l] Re: [obm-l] Dúvida em questão de Racio cínio

2009-02-03 Thread Ralph Teixeira
Fiz como o João Luís falou, deu certo: faça um diagrama de Venn,
preencha "de dentro para fora". Chamando os conjuntos de A, B e C,
(chamo "e" de interseção, "ou" de união):

A e B e C = (A e B) e (A e C) = {Cão} (usando II e IV)
A e B e (não C) = (A e B) - (A e B e C) = {Boi} (II)
A e (não B) e C = (A e C) - (A e B e C) = {Ove} (IV)
(não A) e B e C = (B e C) - (A e B e C) = {Cav} (III)
(não A) e B e (não C) = tudo - (A ou C) = {Por} (I e V)
(não A) e (não B) e C = tudo - (A ou B) = {Chi, Coe} (I e VI)
A e (não B) e (não C) = o que sobrou = {Gat, Gal}

Resposta (c). Tá certo.

Abraço,
  Ralph

2009/2/3 João Luís :
> Não fiz ainda, mas creio que é uma simples questão de considerar cada
> fazenda como sendo um conjunto cujos elementos são o tipo de animal que é
> criado lá. Então afirmativas como "os animais comuns às fazendas Alfa e Beta
> são somente cães e bois" ,  quer dizer que a intersecção dos conjuntos Alfa
> e Beta é o conjunto {cães, bois}.
>
> Monte o diagrama de Venn para 3 conjuntos, vá preenchendo as regiões com as
> informações de I a VI, que eu acho que sai fácil.
>
> Uma pergunta:de onde é essa questão? vestibular? concurso? que instituição?
> e que ano?
>
> Um abraço,
>
> João Luís.
>
> - Original Message -
> From: Marcos Xavier
> To: OBM
> Sent: Monday, February 02, 2009 4:31 PM
> Subject: [obm-l] Dúvida em questão de Raciocínio
>
> Amigos, estou com dificuldade na seguinte questão:
>
> Em um país, há três fazendas: Alfa, Beta e Gama. Sabe-se que nessas fazendas
> criam-se somente animais, e também que,
>
> I.se reunirmos os animais das três fazendas, teremos porcos,
> galinhas, cães, gatos, bois, ovelhas, cavalos, chinchilas e coelhos;
>
> II.   os animais comuns às fazendas Alfa e Beta são somente cães e bois;
>
> III.  os animais comuns às fazendas Beta e Gama são somente cães e
> cavalos;
>
> IV. os animais comuns às fazendas Alfa e Gama são somente cães e
> ovelhas;
>
> V.  se reunirmos os animais presentes nas fazendas Alfa e Gama, então
> ficaremos com galinhas, cães, gatos, bois, ovelhas, cavalos, chinchilas e
> coelhos; e
>
> VI. reunirmos os animais presentes nas fazendas Alfa e Beta, então
> teremos porcos, cães, gatos, bois, ovelhas, cavalos e galinhas.
>
> Logo, pode-se afirmar que:
>
> A)  a fazenda Alfa abriga apenas bois, cães e. ovelhas;
>
> B)  a fazenda Beta abriga apenas bois, cães e cavalos;
>
> C)  a fazenda Beta abriga bois, cães, porcos e cavalos;
>
> D)  as outras fazendas juntas abrigam um menor número de espécies de
> animais que a fazenda Alfa;
>
> E)  bois e galinhas vivem apenas na fazenda Alfa.
>
> A resposta é Letra (C).
>
> Agradeço a todos.
>
> Marcos.
>
> 
> Receba GRÁTIS as mensagens do Messenger no seu celular quando você estiver
> offline. Conheça o MSN Mobile! Crie já o seu!

=
Instruções para entrar na lista, sair da lista e usar a lista em
http://www.mat.puc-rio.br/~obmlistas/obm-l.html
=


[obm-l] Re: [obm-l] Re: [obm-l] Dúvida em questão de R aciocínio

2009-02-03 Thread João Luís
Obrigado Nehab
  - Original Message - 
  From: Carlos Nehab 
  To: obm-l@mat.puc-rio.br 
  Sent: Tuesday, February 03, 2009 11:27 AM
  Subject: Re: [obm-l] Re: [obm-l] Dúvida em questão de Raciocínio


  Oi, João,

  Foi numa prova da ANPAD.  Em geral boas questões...

  http://www.anpadcurso.com/provas_anteriores/rl/PROVA_TESTE_ANPAD_RL-fev_08.pdf

  Abraços,
  Nehab


  João Luís escreveu: 
Não fiz ainda, mas creio que é uma simples questão de considerar cada 
fazenda como sendo um conjunto cujos elementos são o tipo de animal que é 
criado lá. Então afirmativas como "os animais comuns às fazendas Alfa e Beta 
são somente cães e bois" ,  quer dizer que a intersecção dos conjuntos Alfa e 
Beta é o conjunto {cães, bois}.

Monte o diagrama de Venn para 3 conjuntos, vá preenchendo as regiões com as 
informações de I a VI, que eu acho que sai fácil.

Uma pergunta:de onde é essa questão? vestibular? concurso? que instituição? 
e que ano?

Um abraço,

João Luís.
  - Original Message - 
  From: Marcos Xavier 
  To: OBM 
  Sent: Monday, February 02, 2009 4:31 PM
  Subject: [obm-l] Dúvida em questão de Raciocínio


  Amigos, estou com dificuldade na seguinte questão:




  Em um país, há três fazendas: Alfa, Beta e Gama. Sabe-se que nessas 
fazendas criam-se somente animais, e também que,


  I.se reunirmos os animais das três fazendas, teremos porcos, 
galinhas, cães, gatos, bois, ovelhas, cavalos, chinchilas e coelhos;

  II.   os animais comuns às fazendas Alfa e Beta são somente cães e 
bois;

  III.  os animais comuns às fazendas Beta e Gama são somente cães e 
cavalos; 

  IV. os animais comuns às fazendas Alfa e Gama são somente cães e 
ovelhas; 

  V.  se reunirmos os animais presentes nas fazendas Alfa e Gama, então 
ficaremos com galinhas, cães, gatos, bois, ovelhas, cavalos, chinchilas e 
coelhos; e

  VI. reunirmos os animais presentes nas fazendas Alfa e Beta, então 
teremos porcos, cães, gatos, bois, ovelhas, cavalos e galinhas.


  Logo, pode-se afirmar que:


  A)  a fazenda Alfa abriga apenas bois, cães e. ovelhas;

  B)  a fazenda Beta abriga apenas bois, cães e cavalos;

  C)  a fazenda Beta abriga bois, cães, porcos e cavalos;


  D)  as outras fazendas juntas abrigam um menor número de espécies de 
animais que a fazenda Alfa;

  E)  bois e galinhas vivem apenas na fazenda Alfa.




  A resposta é Letra (C).




  Agradeço a todos.




  Marcos.




--
  Receba GRÁTIS as mensagens do Messenger no seu celular quando você 
estiver offline. Conheça o MSN Mobile! Crie já o seu! 
  = 
Instruções para entrar na lista, sair da lista e usar a lista em 
http://www.mat.puc-rio.br/~obmlistas/obm-l.html 
=

[obm-l] RE: [obm-l] Re: [obm-l] Re: [obm-l] Dúvida de vestibular

2003-02-27 Thread João Gilberto Ponciano Pereira
Este é um problema interessante! Mas acho que faltou dizer que as cidades em
questão fazem parte do mesmo país, ou seja, a cidade A pertence a um país C
se existe pelo menos uma estrada que vá de A para alguma cidade pertencente
a C.

Acho que a solução é mais ou menos assim:

A "pegadinha" é provar que se existe um caminho que vai de Cn para Cm, então
existe o caminho de volta de Cm para Cn. Prova: seja Pi o "peso" do número
de estradas da cidade Ci, que é o número de estradas que entram mais o
número de estradas que saem. Supondo que o viajante pega o carro de Cm e vai
para Cn passando por algumas cidades. Como ele saiu de Cm, o peso Pm será 1.
Para as outras cidades, o peso é 2i, pois ele chegou e saiu. Para Cn, o peso
é 2i+1, pois ele chegou.

Veja que as cidades devem ter Pi par, pois o número de estradas que chegam é
o mesmo das que saem. Logo, o ciclo de volta sempre terminará em Cm.

Suponha agora um país com 2 cidades A e B. Se A chega a B, temos que B chega
a A, e o número de mapas é igual. Acrescentando uma cidade C no país, ela
será ligada em A, sem perda de generalidade. Ora, se A chega em C e B chega
em A, então B chega em C e, pela volta, C chega em B. Logo, por indução, vc
consegue chegar em qualquer cidade de um determinado país com N cidades, o
número de mapas possíveis é N-1.

-Original Message-
From: Nicolau C. Saldanha [mailto:[EMAIL PROTECTED]
Sent: Thursday, February 27, 2003 1:41 PM
To: [EMAIL PROTECTED]
Subject: [obm-l] Re: [obm-l] Re: [obm-l] Dúvida de vestibular


Eu [ainda] não sei resolver o problema do Okakome mas...

On Wed, Feb 26, 2003 at 04:39:33PM -0300, Domingos Jr. wrote:
> > Oi Pessoal,
> >   Estava estudando análise combinatória por uma
> > apostila de um curso pré-vestibular, e encontrei o
> > seguinte problema, que achei interessante, mas minha
> > solução foi muito longa, e não sei se está certa,
> > porque tinha muitos casos. Se estivesse num
> > vestibular, o que faria?
> >   Num país, as estradas ligam duas cidades e são de
> > mão única (pode haver mais de uma estrada entre duas
> > cidades). O número de estradas que partem de cada
> > cidade é igual ao número de estradas que chegam nessa
> > cidade. Um mapa da cidade C é um conjunto de rotas
> > que: 1) levam C a cada uma das outras cidades do país,
> > sem passar por uma cidade mais de uma vez. 2) Se uma
> > rota parte de C a D passando por E, então a rota que
> > vai de C a E coincide com o começo da rota de C a D.
> > Prove que o número de mapas da cidade C é igual ao
> > número de mapas de qualquer outra cidade.
> 
> Acho que esse enunciado não está completo:
> - se existe uma cidade com nenhuma estrada partindo ou chegando possui um
> número de rotas 0 e não vai ser igual as demais, até aí é um caso idiota
que
> pode ser excluído do problema.

Neste caso não existe nenhum mapa pois é impossível ligar C a X,
a cidade sem estradas. O problema fica correto pois o número
de mapas é sempre 0.

> - se existem conjuntos de cidades "disjuntos" ou seja cidades de um
conjunto
> A não possuem rota nenhuma para as cidades do conjunto B e vice-versa:
> 
> C1 <---> C2 ( 1 estrada de C1 -> C2 e outra de C2 -> C1  )
> C3 <==> C4 ( 2 estradas de C3 -> C4 e duas de C4 -> C3 )
> 
> neste caso temos que o número de rotas de N(C1) = N(C2) = 1, mas N(C3) =
> N(C4) = 2.

Também neste caso o número de mapas é sempre 0.

> além disso, há um trecho que eu considero confuso:
> 
> "2) Se uma rota parte de C a D passando por E, então a rota que vai de C a
E
> coincide com o começo da rota de C a D"
> nessa situação parece que a rota de C a E deve ser única, mas podem haver
> outras rotas de C até E sem que uma cidade seja visitada mais de uma
vez...
> mesmo que sempre fossem escolhidos os caminhos que passem por menos
cidades
> isso poderia ocorrer já que é permitido haver mais de uma estrada ligando
> duas cidades.

Acho que ajuda considerar o caso em que todas as estradas são de mão dupla:
neste caso um mapa nada mais é do que uma árvore maximal e o problema
fica trivial.

[]s, N.
=
Instruções para entrar na lista, sair da lista e usar a lista em
http://www.mat.puc-rio.br/~nicolau/olimp/obm-l.html
O administrador desta lista é <[EMAIL PROTECTED]>
=
=
Instruções para entrar na lista, sair da lista e usar a lista em
http://www.mat.puc-rio.br/~nicolau/olimp/obm-l.html
O administrador desta lista é <[EMAIL PROTECTED]>
=


  1   2   >